NURS 415 Exam 1

¡Supera tus tareas y exámenes ahora con Quizwiz!

Which statement by a newly delivered woman indicates that she knows what to expect regarding her menstrual activity after childbirth? a. "My first menstrual cycle will be lighter than normal and then will get heavier every month thereafter." b. "My first menstrual cycle will be heavier than normal and will return to my prepregnant volume within three or four cycles." c. "I will not have a menstrual cycle for 6 months after childbirth." d. "My first menstrual cycle will be heavier than normal and then will be light for several months after."

"My first menstrual cycle will be heavier than normal and will return to my prepregnant volume within three or four cycles." Rationale: "My first menstrual cycle will be heavier than normal and will return to my prepregnant volume within three or four cycles" is an accurate statement and indicates her understanding of her expected menstrual activity. She can expect her first menstrual cycle to be heavier than normal, which occurs by 3 months after childbirth, and the volume of her subsequent cycles will return to prepregnant levels within three to four cycles.

Maternity nurses can enhance communication among health care providers by using the SBAR technique. The acronym SBAR stands for what? a. Situation, background, assessment, recommendation b. Situation, baseline, assessment, recommendation c. Subjective, background, analysis, recommendation d. Subjective, background, analysis, review

A. Situation, background, assessment, recommendation Rationale: SBAR is an easy-to-remember, useful, and concrete mechanism for communicating important information that requires a clinician's immediate attention. Baseline is not discussed as part of SBAR. Subjective and analysis are not specific to the SBAR acronym. Subjective, analysis, and review are not specific to the SBAR acronym.

Which hormone is responsible for the maturation of mammary gland tissue? a. Estrogen b. Testosterone c. Prolactin d. Progesterone

Answer: Progesterone Rationale: Progesterone causes maturation of the mammary gland tissue, specifically acinar structures of the lobules. Estrogen increases the vascularity of the breast tissue. Testosterone has no bearing on breast development. Prolactin is produced after birth and released from the pituitary gland; it is produced in response to infant suckling and an emptying of the breasts.

What are the most common causes for subinvolution of the uterus? a. Postpartum hemorrhage and infection b. Multiple gestation and postpartum hemorrhage c. Uterine tetany and overproduction of oxytocin d. Retained placental fragments and infection

Answer: Retained placental fragments and infection Rationale: Subinvolution is the failure of the uterus to return to a nonpregnant state. The most common causes of subinvolution are retained placental fragments and infection. Subinvolution may be caused by an infection and result in hemorrhage. Multiple gestations may cause uterine atony, resulting in postpartum hemorrhaging. Uterine tetany and overproduction of oxytocin do not cause subinvolution.

A recently delivered mother and her baby are at the clinic for a 6-week postpartum checkup. Which response by the client alerts the nurse that psychosocial outcomes have not been met? a. The woman excessively discusses her labor and birth experience. b. The woman feels that her baby is more attractive and cleverer than any others. c. The woman refers to the baby as "sweetie" since she has not given the baby a name yet. d. The woman has a partner or family members who react very positively about the baby.

Answer: The woman refers to the baby as "sweetie" since she has not given the baby a name yet. Rationale: If the mother is having difficulty naming her new infant, it may be a signal that she is not adapting well to parenthood. Other red flags include a refusal to hold or feed the baby, a lack of interaction with the infant, and becoming upset when the baby vomits or needs a diaper change. A new mother who is having difficulty is unwilling to discuss her labor and birth experience. An appropriate nursing diagnosis might be Need for health teaching, related to a long, difficult labor or unmet expectations of birth. A mother who is willing to discuss her birth experience is making a healthy personal adjustment. The mother who is not coping well finds her baby unattractive and messy. She may also be overly disappointed in the baby's sex. The client might voice concern that the baby reminds her of a family member whom she does not like. Having a partner and/or other family members react positively is an indication that this new mother has a good support system in place. This support system helps reduce anxiety related to her new role as a mother.

In which situation would the nurse be called on to stimulate the fetal scalp? a. As part of fetal scalp blood sampling b. In response to tocolysis c. In preparation for fetal oxygen saturation monitoring d. To elicit an acceleration in the fetal heart rate (FHR)

Answer: To elicit an acceleration in the fetal heart rate (FHR) Rationale: The scalp can be stimulated using digital pressure during a vaginal examination. Fetal scalp blood sampling involves swabbing the scalp with disinfectant before a sample is collected. The nurse stimulates the fetal scalp to elicit an acceleration of the FHR. Tocolysis is relaxation of the uterus. Fetal oxygen saturation monitoring involves the insertion of a sensor.

After an emergency birth, the nurse encourages the woman to breastfeed her newborn. What is the primary purpose of this activity? a. To facilitate maternal-newborn interaction b. To stimulate the uterus to contract c. To prevent neonatal hypoglycemia d. To initiate the lactation cycle

Answer: To stimulate the uterus to contract Rationale: Stimulation of the nipples through breastfeeding or manual stimulation causes the release of oxytocin and prevents maternal hemorrhage. Breastfeeding facilitates maternal-newborn interaction, but it is not the primary reason a woman is encouraged to breastfeed after an emergency birth. The primary intervention for preventing neonatal hypoglycemia is thermoregulation. Cold stress can result in hypoglycemia. The woman is encouraged to breastfeed after an emergency birth to stimulate the release of oxytocin, which prevents hemorrhaging. Breastfeeding is encouraged to initiate the lactation cycle, but it is not the primary reason for this activity after an emergency birth.

What is the rationale for the administration of an oxytocic after expulsion of the placenta? a. To relieve pain b. To stimulate uterine contraction c. To prevent infection d. To facilitate rest and relaxation

Answer: To stimulate uterine contraction Rationale: Oxytocics stimulate uterine contractions, which reduce blood loss after the third stage of labor. Oxytocics are not used to treat pain, do not prevent infection, and do not facilitate rest and relaxation.

What is the primary difference between the labor of a nullipara and that of a multipara? a. Amount of cervical dilation b. Total duration of labor c. Level of pain experienced d. Sequence of labor mechanisms

Answer: Total duration of labor Rationale: In a first-time pregnancy, the descent is usually slow but steady; in subsequent pregnancies, the descent is more rapid, resulting in a shorter duration of labor. Cervical dilation is the same for all labors. The level of pain is individual to the woman, not to the number of labors she has experienced. The sequence of labor mechanisms is the same with all labors.

What is the most likely cause for variable fetal heart rate (FHR) decelerations? a. Altered fetal cerebral blood flow b. Umbilical cord compression c. Uteroplacental insufficiency d. Fetal hypoxemia

Answer: Umbilical cord compression Rationale: Variable FHR decelerations can occur at any time during the uterine contracting phase and are caused by compression of the umbilical cord. Altered fetal cerebral blood flow results in early decelerations in the FHR. Uteroplacental insufficiency results in late decelerations in the FHR. Fetal hypoxemia initially results in tachycardia and then bradycardia if hypoxia continues.

What are the various modes of heat loss in the newborn? (Select all that apply.) a. Perspiration b. Convection c. Radiation d. Conduction e. Urination

Answers: b. Convection c. Radiation d. Conduction Rationale: Convection, radiation, evaporation, and conduction are the four modes of heat loss in the newborn.

Many first-time parents do not plan on having their parents' help immediately after the newborn arrives. Which statement by the nurse is the most appropriate when counseling new parents regarding the involvement of grandparents? a. "You should tell your parents to leave you alone." b. "Grandparents can help you with parenting skills." c. "Grandparent involvement can be very disruptive to the family." d. "They are getting old. You should let them be involved while they can."

"Grandparents can help you with parenting skills." Rationale: Telling the parents that grandparents can help with parenting skills and therefore help preserve family traditions is the most appropriate response. Intergenerational help may be perceived as interference but telling the parents that their parents should be told to leave them alone is not therapeutic to the adaptation of the family. Telling the parents that grandparent involvement can be disruptive to the family is an invalid statement and not an appropriate nursing response. Regardless of age, grandparents can help with parenting skills and preserve family traditions.

A young woman arrives at the emergency department and states that she thinks she has been raped. She is sobbing and expresses disbelief that this could happen because the perpetrator was a very close friend. Which statement is most appropriate nursing response? a. "Rape is not limited to strangers and frequently occurs by someone who is known to the victim." b. "I would be very upset if my best friend did that to me; that is very unusual." c. "You must feel very betrayed. In what way do you think you might have led him on?" d. "This does not sound like rape. Didn't you just change your mind about having sex after the fact?"

"Rape is not limited to strangers and frequently occurs by someone who is known to the victim." Rationale: Acquaintance rape involves individuals who know one another. Sexual assault occurs when the trust of a relationship is violated. Victims may be less prone to recognize what is happening to them because the dynamics are different from those of stranger rape. It is not at all unusual for the victim to know and trust the perpetrator. Stating that the woman might have led the man to attack her indicates that the sexual assault was somehow the victim's fault. This type of mentality is not constructive. Nurses must first reflect on their own feelings and learn to be unbiased when dealing with victims. A statement of this type can be very psychologically damaging to the victim. Nurses must display compassion by first believing what the victim states. The nurse is not responsible for deciphering the facts involving the victim's claim.

Which information about variations in the infant's blood counts is important for the nurse to explain to the new parents? a. A somewhat lower-than-expected red blood cell count could be the result of a delay in clamping the umbilical cord. b. An early high white blood cell (WBC) count is normal at birth and should rapidly decrease. c. Platelet counts are higher in the newborn than in adults for the first few months. d. Even a modest vitamin K deficiency means a problem with the blood's ability to properly clot.

Ansswer: early high white blood cell (WBC) count is normal at birth and should rapidly decrease. Rationale: The WBC count is normally high on the first day of birth and then rapidly declines. Delayed cord clamping results in an increase in hemoglobin and the red blood cell count. The platelet count is essentially the same for newborns and adults. Clotting is sufficient to prevent hemorrhage unless the deficiency of vitamin K is significant.

Which infant response to cool environmental conditions is either not effective or not available to them? a. Constriction of peripheral blood vessels b. Metabolism of brown fat c. Increased respiratory rates d. Unflexing from the normal position

Answer : Unflexing from the normal position Rationale: The newborn's flexed position guards against heat loss, because it reduces the amount of body surface exposed to the environment. The newborn's body is able to constrict the peripheral blood vessels to reduce heat loss. Burning brown fat generates heat. The respiratory rate may rise to stimulate muscular activity, which generates heat.

Which condition is likely to be a psychologic consequence of continued physical and psychological abuse? Select all that apply. a. Substance abuse b. Posttraumatic stress disorder (PTSD) c. Eating disorders d. None of the above e. Bipolar disorder General anxiety

Answer: Rationale: Bipolar disorder is a specific illness (also known as manic depressive disorder) not related to abuse. Substance abuse is a common method of coping with long-term abuse. The abuser is also more likely to use alcohol and other chemical substances. PTSD is the most prevalent mental health sequela of long-term abuse. The traumatic event is persistently re-experienced through distress recollection and dreams. Eating disorders, depression, psychologic-physiologic illness, and anxiety reactions are all mental health problems associated with repeated abuse.

Which client is most likely to experience strong and uncomfortable afterpains? a. A woman who experienced oligohydramnios b. A woman who is a gravida 4, para 4-0-0-4 c. A woman who is bottle-feeding her infant d. A woman whose infant weighed 5 pounds, 3 ounces

Answer: A woman who is a gravida 4, para 4-0-0-4 Rationale: Afterpains are more common in multiparous women. In a woman who experienced polyhydramnios, afterpains are more noticeable because the uterus was greatly distended. Breastfeeding may cause the afterpains to intensify. In a woman who delivered a large infant, afterpains are more noticeable because the uterus was greatly distended.

The parents of a newborn ask the nurse how much the newborn can see. The parents specifically want to know what type of visual stimuli they should provide for their newborn. What information provided by the nurse would be most useful to these new parents? a. "Infants can see very little until approximately 3 months of age." b. "Clearest visual distance is 8 to 12 inches and they can distinguish patterns; preferring complex ones." c. "The infant's eyes must be protected. Infants enjoy looking at brightly colored stripes." d. "It's important to shield the newborn's eyes. Overhead lights help them see better."

Answer: "Clearest visual distance is 8 to 12 inches and they can distinguish patterns; preferring complex ones." Rationale: Telling the parents that their infant's clearest visual distance is 17 to 20 cm (8 to 12 inches) and can distinguish patterns is an accurate statement. Development of the visual system continues for the first 6 months of life. Visual acuity is difficult to determine, but they seem to prefer to look at complex patterns, regardless of the color. They prefer low illumination and withdraw from bright lights.

A new father is ready to take his wife and newborn son home. He proudly tells the nurse who is discharging them that within the next week he plans to start feeding the infant cereal between breastfeeding sessions. Which information should the nurse provide regarding this feeding plan? a. "Feeding solid foods before your son is 4 to 6 months old may decrease your son's intake of sufficient calories." b. "Feeding solid foods between breastfeeding sessions before your son is 4 to 6 months old will lead to an early cessation of breastfeeding." c. "Your feeding plan will help your son sleep through the night." d. "Feeding solid foods before your son is 4 to 6 months old will limit his growth."

Answer: "Feeding solid foods between breastfeeding sessions before your son is 4 to 6 months old will lead to an early cessation of breastfeeding." Rationale: The introduction of solid foods before the infant is 4 to 6 months of age may result in overfeeding and decreased intake of breast milk. The belief that feeding solid foods helps infants sleep through the night is untrue. The proper balance of carbohydrate, protein, and fat for an infant to grow properly is in the breast milk or formula.

The nurse is evaluating the new mother's knowledge about appropriate infant bottle feeding techniques. Which statement by the client reassures the nurse that correct learning has taken place? a. "Since reaching 2 weeks of age, I add rice cereal to my daughter's formula to ensure adequate nutrition." b. "I warm the bottle in my microwave oven." c. "I burp my daughter during and after the feeding and as needed." d. "I refrigerate any leftover formula for the next feeding."

Answer: "I burp my daughter during and after the feeding and as needed." Rationale: Most infants swallow air when fed from a bottle and should be given a chance to burp several times during and after the feeding. Solid food should not be introduced to the infant for at least 4 to 6 months after birth. A microwave should never be used to warm any food to be given to an infant. The heat is not distributed evenly, which may pose a risk of burning the infant. Any formula left in the bottle after the feeding should be discarded because the infant's saliva has mixed with it.

Which statement by the client indicates that she understands breast self-examination (BSE)? a. "I will examine both breasts in two different positions." b. "I will examine my breasts 1 week after my menstrual period starts." c. "I will examine only the outer upper area of the breast." d. "I will use the palm of the hand to perform the examination."

Answer: "I will examine my breasts 1 week after my menstrual period starts." Rationale: The woman should examine her breasts when hormonal influences are at their lowest level. The client should be instructed to use four positions: standing with arms at her sides, standing with arms raised above her head, standing with hands pressed against hips, and lying down. The entire breast needs to be examined, including the outer upper area. The client should use the sensitive pads of the middle three fingers.

The nurse guides a woman to the examination room and asks her to remove her clothes and put on an examination gown with the front open. The woman replies, "I have special undergarments that I do not remove for religious reasons." Which is the most appropriate response from the nurse? a. "You can't have an examination without removing all your clothes." b. "I'll ask the physician to modify the examination." c. "I'll explain the examination procedure, and then we can discuss how you can comfortably have your examination." d. "I have no idea how we can accommodate your beliefs."

Answer: "I'll explain the examination procedure, and then we can discuss how you can comfortably have your examination." Rationale: Explaining the examination procedure reflects cultural competence by the nurse and shows respect for the woman's religious practices. The nurse must respect the rich and unique qualities that cultural diversity brings to individuals. The examination can be modified to ensure that modesty is maintained. In recognizing the value of cultural differences, the nurse

A blind woman has arrived for an examination. She appears nervous and says, "I've never had a pelvic examination." What response from the nurse would be most appropriate? a. "Being visually impaired must be very anxiety producing." b. "Try to relax. I'll be very gentle, and I promise not to hurt you." c. "Your anxiety is common. I was anxious when I first had a pelvic examination." d. "I'll let you touch each instrument I'll be using as I tell you how it will be used."

Answer: "I'll let you touch each instrument I'll be using as I tell you how it will be used." Rationale: The client who is visually impaired needs to be oriented to the examination room and needs a full explanation of what the examination entails before the nurse proceeds. The statement regarding her visual disability and anxiety does not address her concerns. The nurse should openly and directly communicate with sensitivity. Women who have physical disabilities should be respected and involved in the assessment and physical examination to the full extent of their abilities. Telling the client that she will not be hurt does not reflect respect or sensitivity. Although anxiety may be common, the nurse should not discuss her own issues nor compare them to the client's concerns.

Which statement accurately describes an appropriate-for-gestational age (AGA) weight assessment? a. AGA weight assessment falls between the 25th and 75th percentiles for the infant's age. b. AGA weight assessment depends on the infant's length and the size of the newborn's head. c. AGA weight assessment falls between the 10th and 90th percentiles for the infant's age. d. AGA weight assessment is modified to consider intrauterine growth restriction (IUGR).

Answer: AGA weight assessment falls between the 10th and 90th percentiles for the infant's age Rationale: An AGA weight falls between the 10th and 90th percentiles for the infant's age. The AGA range is larger than the 25th and 75th percentiles. The infant's length and head size are measured, but these measurements do not affect the normal weight designation. IUGR applies to the fetus, not to the newborn's weight.

A client is in early labor, and her nurse is discussing the pain relief options she is considering. The client states that she wants an epidural "no matter what!" What is the nurse's best response? a. "I'll make sure you get your epidural." b. "You may only have an epidural if your physician allows it." c. "You may only have an epidural if you are going to deliver vaginally." d. "The type of analgesia or anesthesia used is determined, in part, by the stage of your labor and the method of birth."

Answer: "I'll make sure you get your epidural." Rationale: To avoid suppressing the progress of labor, pharmacologic measures for pain relief are generally not implemented until labor has advanced to the active phase of the first stage and the cervix is dilated approximately 4 to 5 cm. A plan of care is developed for each woman that addresses her particular clinical and nursing problems. The nurse collaborates with the primary health care provider and the laboring woman in selecting features of care relevant to the woman and her family. The decision whether to use an epidural to relieve labor pain is multifactorial. The nurse should not make a blanket statement guaranteeing the client one pharmacologic option over another until a complete history and physical examination has been obtained. A physician's order is required for pharmacologic options for pain management. However, expressing this requirement is not the nurse's best response. An epidural is an effective pharmacologic pain management option for many laboring women. It can also be used for anesthesia control if the woman undergoes an operative delivery.

A 25-year-old multiparous woman gave birth to an infant boy 1 day ago. Today her husband brings a large container of brown seaweed soup to the hospital. When the nurse enters the room, the husband asks for help with warming the soup so that his wife can eat it. What is the nurse's most appropriate response? a. "Didn't you like your lunch?" b. "Does your health care provider know that you are planning to eat that?" c. "What is that anyway?" d. "I'll warm the soup in the microwave for you."

Answer: "I'll warm the soup in the microwave for you." Rationale: Offering to warm the food shows cultural sensitivity to the dietary preferences of the woman and is the most appropriate response. Cultural dietary preferences must be respected. Women may request that family members bring favorite or culturally appropriate foods to the hospital. Asking the woman to identify her food does not show cultural sensitivity. Both remaining options demonstrate insensitivity.

A woman who is 39 weeks pregnant expresses fear about her impending labor and how she will manage. What is the nurse's ideal response? a. "Don't worry about it. You'll do fine." b. "It's normal to be anxious about labor. Let's discuss what makes you afraid." c. "Labor is scary to think about, but the actual experience isn't." d. "You can have an epidural. You won't feel anything." ANS: B "It's normal to be anxious about labor. Let's discuss what makes you afraid" is a statement that allows the woman to share her concerns with the nurse and is a therapeutic communication tool. "Don't worry about it. You'll do fine" negates the woman's fears and is not therapeutic. "Labor is scary to think about, but the actual experience isn't" negates the woman's fears and offers a false sense of security. To suggest that every woman can have an epidural is untrue. Several criteria must be met before an epidural is considered. Furthermore, many women still experience the feeling of pressure with an epidural.

Answer: "It's normal to be anxious about labor. Let's discuss what makes you afraid." Rationale: "It's normal to be anxious about labor. Let's discuss what makes you afraid" is a statement that allows the woman to share her concerns with the nurse and is a therapeutic communication tool. "Don't worry about it. You'll do fine" negates the woman's fears and is not therapeutic. "Labor is scary to think about, but the actual experience isn't" negates the woman's fears and offers a false sense of security. To suggest that every woman can have an epidural is untrue. Several criteria must be met before an epidural is considered. Furthermore, many women still experience the feeling of pressure with an epidural.

When the nurse is alone with a battered client, the client seems extremely anxious and says, "It was all my fault. The house was so messy when he got home, and I know he hates that." What is the most suitable response by the nurse? a. "No one deserves to be hurt. It's not your fault. How can I help you?" b. "What else do you do that makes him angry enough to hurt you?" c. "He will never find out what we talk about. Don't worry. We're here to help you." d. "You have to remember that he is frustrated and angry, so he takes it out on you."

Answer: "No one deserves to be hurt. It's not your fault. How can I help you?" Rationale: The nurse should stress that the client is not at fault. Asking what the client did to make her husband angry is placing the blame on the woman and would be an inappropriate statement. The nurse should not provide false reassurance. To assist the woman, the nurse should be honest. Often the batterer will find out about the conversation.

A first-time mother is concerned about the type of medications she will receive during labor. The client is in a fair amount of pain and is nauseated. In addition, she appears to be very anxious. The nurse explains that opioid analgesics are often used along with sedatives. How should the nurse phrase the rationale for this medication combination? a. "The two medications, together, reduce complications." b. "Sedatives enhance the effect of the pain medication." c. "The two medications work better together, enabling you to sleep until you have the baby." d. "This is what your physician has ordered for you."

Answer: "Sedatives enhance the effect of the pain medication." Rationale: Sedatives may be used to reduce the nausea and vomiting that often accompany opioid use. In addition, some ataractic drugs reduce anxiety and apprehension and potentiate the opioid analgesic affects. A potentiator may cause two drugs to work together more effectively, but it does not ensure zero maternal or fetal complications. Sedation may be a related effect of some ataractic drugs; however, sedation is not the goal. Furthermore, a woman is unlikely to be able to sleep through transitional labor and birth. Although the physician may have ordered the medication, "This is what your physician has ordered for you" is not an acceptable comment for the nurse to make.

A first-time father is changing the diaper of his 1-day-old daughter. He asks the nurse, "What is this brown, sticky stuff in her diaper?" What is the nurse's best response? a. "That's meconium, which is your baby's first stool. It's normal." b. "That's transitional stool." c. "That means your baby is bleeding internally." d. "Oh, don't worry about that. It's okay."

Answer: "That's meconium, which is your baby's first stool. It's normal." Rationale: Explaining what meconium is and that it is normal is an accurate statement and the most appropriate response. Transitional stool is greenish-brown to yellowish-brown and usually appears by the third day after the initiation of feeding. Telling the father that the baby is internally bleeding is not an accurate statement. Telling the father not to worry is not appropriate. Such responses are belittling to the father and do not teach him about the normal stool patterns of his daughter.

A new client and her partner arrive on the labor, delivery, recovery, and postpartum (LDRP) unit for the birth of their first child. The nurse applies the electronic fetal monitor (EFM) to the woman. Her partner asks you to explain what is printing on the graph, referring to the EFM strip. He wants to know what the baby's heart rate should be. What is the nurse's best response? a. "Don't worry about that machine; that's my job." b. "The baby's heart rate will fluctuate in response to what is happening during labor." c. "The top line graphs the baby's heart rate, and the bottom line lets me know how strong the contractions are." d. "Your physician will explain all of that later."

Answer: "The baby's heart rate will fluctuate in response to what is happening during labor." Rationale: Explaining what indicates a normal fetal heart rate (FHR) teaches the partner about fetal monitoring and provides support and information to alleviate his fears. Telling the partner not to worry discredits his feelings and does not provide the teaching he is requesting. Telling the partner that the graph indicates how strong the contractions are provides inaccurate information and does not address the partner's concerns about the FHR. The EFM graphs the frequency and duration of the contractions, not their intensity. Nurses should take every opportunity to provide teaching to the client and her family, especially when information is requested.

Which statement by the client will assist the nurse in determining whether she is in true labor as opposed to false labor? a. "I passed some thick, pink mucus when I urinated this morning." b. "My bag of waters just broke." c. "The contractions in my uterus are getting stronger and closer together." d. "My baby dropped, and I have to urinate more frequently now."

Answer: "The contractions in my uterus are getting stronger and closer together." Rationale: Regular, strong contractions with the presence of cervical change indicate that the woman is experiencing true labor. The loss of the mucous plug (operculum) often occurs during the first stage of labor or before the onset of labor, but it is not the indicator of true labor. Spontaneous rupture of membranes (ROM) often occurs during the first stage of labor, but it is not the indicator of true labor. The presenting part of the fetus typically becomes engaged in the pelvis at the onset of labor, but this is not the indicator of true labor.

While obtaining a detailed history from a woman who has recently immigrated from Somalia, the nurse realizes that the client has undergone female genital mutilation. What is the nurse's most appropriate response in this situation? a. "This is a very abnormal practice and rarely seen in the United States." b. "Are you aware of who performed this mutilation so that it can be reported to the authorities?" c. "We will be able to restore fully your circumcision after delivery." d. "The extent of your circumcision will affect the potential for complications."

Answer: "The extent of your circumcision will affect the potential for complications." Rationale: The extent of the circumcision is important. The client may experience pain, bleeding, scarring, or infection and may require surgery before childbirth. Although this practice is not prevalent in the United States, it is very common in many African and Middle Eastern countries for religious reasons. Mentioning that the practice is abnormal and rarely seen in the United States is culturally insensitive. The infibulation may have occurred during infancy or childhood; consequently, the client will have little to no recollection of the event. She would have considered this to be a normal milestone during her growth and development. The International Council of Nurses has spoken out against this procedure as harmful to a woman's health.

A primiparous woman is watching her newborn sleep. She wants him to wake up and respond to her. The mother asks the nurse how much he will sleep every day. What is an appropriate response by the nurse? a. "He will only wake up to be fed, and you should not bother him between feedings." b. "The newborn sleeps approximately 17 hours a day, with periods of wakefulness gradually increasing." c. "He will probably follow your same sleep and wake patterns, and you can expect him to be awake soon." d. "He is being stubborn by not waking up when you want him to. You should try to keep him awake during the daytime so that he will sleep through the night."

Answer: "The newborn sleeps approximately 17 hours a day, with periods of wakefulness gradually increasing." Rationale: Telling the woman that the newborn sleeps approximately 16 to 19 hours a day with periods of wakefulness that gradually increase is both accurate and the most appropriate response by the nurse. Periods of wakefulness are dictated by hunger, but the need for socializing also appears. Telling the woman that her infant is stubborn and should be kept awake during the daytime is an inappropriate nursing response.

Which statement indicates that a client requires additional instruction regarding breast self-examination (BSE)? a. "Yellow discharge from my nipple is normal if I'm having my period." b. "I should check my breasts at the same time each month, after my period." c. "I should also feel in my armpit area while performing my breast examination." d. "I should check each breast in a set way, such as in a circular motion."

Answer: "Yellow discharge from my nipple is normal if I'm having my period." Rationale: Discharge from the nipples requires further examination from a health care provider. The breasts should be checked at the same time each month. The armpit should also be examined. A circular motion is the best method during which to ascertain any changes in the breast tissue.

A father and mother are carriers of phenylketonuria (PKU). Their 2-year-old daughter has the condition. The couple tells the nurse that they are expecting a second baby. Because their daughter has PKU, they are certain that this baby will not be affected. Which response by the nurse is the most accurate? a. "Good planning. You need to take advantage of the odds that are in your favor." b. "I think you'd better first check with your physician." c. "You are both carriers; therefore, each baby has a 25% chance of being affected." d. "The ultrasound indicates a boy, and boys are not affected by PKU."

Answer: "You are both carriers; therefore, each baby has a 25% chance of being affected." Rationale: Each child conceived by this couple has a one-in-four chance of being affected with the PKU disorder. This couple still has an increased likelihood of having a child with PKU; having one child already with PKU does not guarantee that they will not have another. These parents need to discuss their options with their physician since the pregnancy has already occurred. However, an opportune time has presented itself for the couple to receive correct teaching about inherited genetic risks. No correlation exists between gender and inheritance of the disorder, because PKU is an autosomal recessive disorder.

A client is warm and asks for a fan in her room for her comfort. The nurse enters the room to assess the mother and her infant and finds the infant unwrapped in his crib with the fan blowing over him on high. The nurse instructs the mother that the fan should not be directed toward the newborn and that the newborn should be wrapped in a blanket. The mother asks why. How would the nurse respond? a. "Your baby may lose heat by convection, which means that he will lose heat from his body to the cooler ambient air. You should keep him wrapped and should prevent cool air from blowing on him." b. "Your baby may lose heat by conduction, which means that he will lose heat from his body to the cooler ambient air. You should keep him wrapped and should prevent cool air from blowing on him." c. "Your baby may lose heat by evaporation, which means that he will lose heat from his body to the cooler ambient air. You should keep him wrapped and should prevent cool air from blowing on him." d. "Your baby will easily get cold stressed and needs to be bundled up at all times."

Answer: "Your baby may lose heat by convection, which means that he will lose heat from his body to the cooler ambient air. You should keep him wrapped and should prevent cool air from blowing on him." Rationale: Convection is the flow of heat from the body surface to cooler ambient air. Because of heat loss by convection, all newborns in open bassinets should be wrapped to protect them from the cold. Conduction is the loss of heat from the body surface to cooler surfaces, not air, in direct contact with the newborn. Evaporation is a loss of heat that occurs when a liquid is converted into a vapor. In the newborn, heat loss by evaporation occurs as a result of vaporization of moisture from the skin. Cold stress may occur from excessive heat loss; however, this does not imply that the infant will become stressed if not bundled at all times. Furthermore, excessive bundling may result in a rise in the infant's temperature.

A woman gave birth to an infant boy 10 hours ago. Where does the nurse expect to locate this woman's fundus? a. 1 centimeter above the umbilicus b. 2 centimeters below the umbilicus c. Midway between the umbilicus and the symphysis pubis d. Nonpalpable abdominally

Answer: 1 centimeter above the umbilicus Rationale: The fundus descends approximately 1 to 2 cm every 24 hours. Within 12 hours after delivery the fundus may be approximately 1 cm above the umbilicus. By the sixth postpartum week the fundus is normally halfway between the symphysis pubis and the umbilicus. The fundus should be easily palpated using the maternal umbilicus as a reference point.

The nurse is assessing a full term, quiet, and alert newborn. What is the average expected apical pulse range (in beats per minute)? a. 80 to 100 b. 100 to 120 c. 120 to 160 d. 150 to 180

Answer: 120 to 160 Rationale: The average infant heart rate while awake is 120 to 160 beats per minute. The newborn's heart rate may be approximately 85 to 100 beats per minute while sleeping and typically a little higher than 100 to 120 beats per minute when alert but quiet. A heart rate of 150 to 180 beats per minute is typical when the infant cries.

A new mother asks the nurse when the "soft spot" on her son's head will go away. What is the nurse's best response, based upon her understanding of when the anterior frontal closes? a. 2 months b. 8 months c. 12 months d. 18 months

Answer: 18 months Rationale: The larger of the two fontanels, the anterior fontanel, closes by 18 months after birth. The posterior fontanel closes at 6 to 8 weeks. The remaining three options are too early for the anterior fontanel to close.

Parents who have not already done so need to make time for newborn follow-up of the discharge. According to the American Academy of Pediatrics (AAP), when should a breastfeeding infant first need to be seen for a follow-up examination? a. 2 weeks of age b. 7 to 10 days after childbirth c. 4 to 5 days after hospital discharge d. 48 to 72 hours after hospital discharge

Answer: 48 to 72 hours after hospital discharge Rationale: Breastfeeding infants are routinely seen by the pediatric health care provider clinic within 3 to 5 days after birth or 48 to 72 hours after hospital discharge and again at 2 weeks of age. Formula-feeding infants may be seen for the first time at 2 weeks of age.

At 1 minute after birth a nurse assesses an infant and notes a heart rate of 80 beats per minute, some flexion of extremities, a weak cry, grimacing, and a pink body but blue extremities. Which Apgar score does the nurse calculate based upon these observations and signs? a. 4 b. 5 c. 6 d. 7

Answer: 5 Rationale: Each of the five signs the nurse notes scores a 1 on the Apgar scale, for a total of 5. A score of 4 is too low for this infant. A score of 6 is too high for this infant. A score of 7 is too high for an infant with this presentation.

How many kilocalories per kilogram (kcal/kg) of body weight does a breastfed term infant require each day? a. 50 to 65 b. 75 to 90 c. 95 to 110 d. 150 to 200

Answer: 95 to 110 Rationale: For the first 3 months, the infant needs 110 kcal/kg/day. At ages 3 to 6 months, the requirement is 100 kcal/kg/day. This level decreases slightly to 95 kcal/kg/day from 6 to 9 months and increases again to 100 kcal/kg/day until the baby reaches 12 months.

A nulliparous woman has just begun the latent phase of the second stage of her labor. The nurse should anticipate which behavior? a. A nulliparous woman will experience a strong urge to bear down. b. Perineal bulging will show. c. A nulliparous woman will remain quiet with her eyes closed between contractions. d. The amount of bright red bloody show will increase.

Answer: A nulliparous woman will remain quiet with her eyes closed between contractions. Rationale: The woman is able to relax and close her eyes between contractions as the fetus passively descends. The woman may be very quiet during this phase. During the latent phase of the second stage of labor, the urge to bear down is often absent or only slight during the acme of the contractions. Perineal bulging occurs during the transition phase of the second stage of labor, not at the beginning of the second stage. An increase in bright red bloody show occurs during the descent phase of the second stage of labor.

Screening for critical congenital heart disease (CCHD) was added to the uniform screening panel in 2011. The nurse has explained this testing to the new mother. Which action by the nurse related to this test is correct? a. Screening is performed when the infant is 12 hours of age. b. Testing is performed with an electrocardiogram. c. Oxygen (O2) is measured in both hands and in the right foot. d. A passing result is an O2 saturation of ≥95%.

Answer: A passing result is an O2 saturation of ≥95%. Rationale: Screening is performed when the infant is between 24 and 48 hours of age. The test is performed using pulse oximetry technology. O2 is measured in the right hand and one foot. A passing result is an O2 saturation of ≥95% with a ≤3% absolute difference between upper and lower extremity readings.

Part of the health assessment of a newborn is observing the infant's breathing pattern. What is the predominate pattern of newborn's breathing? a. Abdominal with synchronous chest movements b. Chest breathing with nasal flaring c. Diaphragmatic with chest retraction d. Deep with a regular rhythm

Answer: Abdominal with synchronous chest movements Rationale: In a normal infant respiration, the chest and abdomen synchronously rise and infant breaths are shallow and irregular. Breathing with nasal flaring is a sign of respiratory distress. Diaphragmatic breathing with chest retraction is also a sign of respiratory distress.

Which term is an accurate description of the process by which people retain some of their own culture while adopting the practices of the dominant society? a. Acculturation b. Assimilation c. Ethnocentrism d. Cultural relativism

Answer: Acculturation Rationale: Acculturation is the process by which people retain some of their own culture while adopting the practices of the dominant society. This process takes place over the course of generations. Assimilation is a loss of cultural identity. Ethnocentrism is the belief in the superiority of one's own culture over the cultures of others. Cultural relativism recognizes the roles of different cultures.

A new mother states that her infant must be cold because the baby's hands and feet are blue. This common and temporary condition is called what? a. Acrocyanosis b. Erythema toxicum neonatorum c. Harlequin sign d. Vernix caseosa

Answer: Acrocyanosis Rationale: Acrocyanosis, or the appearance of slightly cyanotic hands and feet, is caused by vasomotor instability, capillary stasis, and a high hemoglobin level. Acrocyanosis is normal and intermittently appears over the first 7 to 10 days after childbirth. Erythema toxicum neonatorum (also called erythema neonatorum) is a transient newborn rash that resembles flea bites. The harlequin sign is a benign, transient color change in newborns. One half of the body is pale, and the other one half is ruddy or bluish-red with a line of demarcation. Vernix caseosa is a cheeselike, whitish substance that serves as a protective covering for the newborn.

Nurses can help their clients by keeping them informed about the distinctive stages of labor. Which description of the phases of the first stage of labor is accurate? a. Latent: Mild, regular contractions; no dilation; bloody show; duration of 2 to 4 hours b. Active: Moderate, regular contractions; 4- to 7-cm dilation; duration of 3 to 6 hours c. Lull: No contractions; dilation stable; duration of 20 to 60 minutes d. Transition: Very strong but irregular contractions; 8- to 10-cm dilation; duration of 1 to 2 hours

Answer: Active: Moderate, regular contractions; 4- to 7-cm dilation; duration of 3 to 6 hours Rationale: The active phase is characterized by moderate, regular contractions; 4- to 7-cm dilation; and a duration of 3 to 6 hours. The latent phase is characterized by mild-to-moderate and irregular contractions; dilation up to 3 cm; brownish-to-pale pink mucus, and a duration of 6 to 8 hours. No official "lull" phase exists in the first stage. The transition phase is characterized by strong- to-very strong and regular contractions; 8- to 10-cm dilation; and a duration of 20 to 40 minutes.

Which description of the phases of the first stage of labor is most accurate? a. Latent: mild, regular contractions; no dilation; bloody show b. Active: moderate, regular contractions; 4 to 7 cm dilation c. Lull: no contractions; dilation stable d. Transition: very strong but irregular contractions; 8 to 10 cm dilation

Answer: Active: moderate, regular contractions; 4 to 7 cm dilation Rationale: The active phase is characterized by moderate and regular contractions, 4 to 7 cm dilation, and duration of 3 to 6 hours. The latent phase is characterized by mild-to-moderate and irregular contractions, dilation up to 3 cm, brownish-to-pale pink mucus, and duration of 6 to 8 hours. No official "lull" phase exists in the first stage. The transition phase is characterized by strong to very strong and regular contractions, 8 to 10 cm dilation, and duration of 20 to 40 minutes.

In evaluating the level of a pregnant woman's risk of having a low-birth-weight (LBW) infant, which factor is the most important for the nurse to consider? a. African-American race b. Cigarette smoking c. Poor nutritional status d. Limited maternal education

Answer: African-American Rationale: The rise in the overall LBW rates were due to increases in LBW births to non-Hispanic black women (13.35%) and Hispanic women (7.21%); non-Hispanic black infants are almost twice as likely as non-Hispanic white infants to be of LBW and to die in the first year of life. Race is a nonmodifiable risk factor. Cigarette smoking is an important factor in potential infant mortality rates, but it is not the most important. Additionally, smoking is a modifiable risk factor. Poor nutrition is an important factor in potential infant mortality rates, but it is not the most important. Additionally, nutritional status is a modifiable risk factor. Maternal education is an important factor in potential infant mortality rates, but it is not the most important. Additionally, maternal education is a modifiable risk factor.

Which statement regarding the postpartum uterus is correct? a. At the end of the third stage of labor, the postpartum uterus weighs approximately 500 g. b. After 2 weeks postpartum, it should be abdominally nonpalpable. c. After 2 weeks postpartum, it weighs 100 g. d. Postpartum uterus returns to its pre-pregnancy size by 6 weeks postpartum.

Answer: After 2 weeks postpartum, it should be abdominally nonpalpable. Rationale: The uterus does not return to its original size. At the end of the third stage of labor, the uterus weighs approximately 1000 g. After 2 weeks postpartum, the uterus weighs approximately 350 g. The normal self-destruction of excess hypertrophied tissue accounts for the slight increase in uterine size after each pregnancy.

Which statement describes a key finding of the Human Genome Project? a. Humans produce one protein per gene. b. All human beings are 99.9% identical at the deoxyribonucleic acid (DNA) level. c. The Human Genome Project has not yet been able to translate the accumulating raw research into anything medically practical. d. Humans have more genes than other species.

Answer: All human beings are 99.9% identical at the deoxyribonucleic acid (DNA) level. Rationale: The majority of the 0.1% genetic variations are found within and not among populations. Most human genes produce at least three proteins. The project's research has been very valuable in the identification of genes involved in disease and in the development of genetic testing. There are 20,500 genes in the human genome; scientists originally estimated more than 50,000 genes. Human genes are more efficient than the genes in other species, thereby increasing the human genetic complexity.

A 25-year-old gravida 1 para 1 who had an emergency cesarean birth 3 days ago is scheduled for discharge. As the nurse prepares her for discharge, she begins to cry. The nurse's next action should be what? a. Assess her for pain. b. Point out how lucky she is to have a healthy baby. c. Explain that she is experiencing postpartum blues. d. Allow her time to express her feelings.

Answer: Allow her time to express her feelings. Rationale: Although many women experience transient postpartum blues, they need assistance in expressing their feelings. Postpartum blues affects 50% to 80% of new mothers. An assumption that the client is in pain should not be made when, in fact, she may have no pain whatsoever. Making this assumption would be blocking communication and inappropriate in this situation. The client needs the opportunity to express her feelings first; client teaching can occur later.

What is the most likely cause for early decelerations in the fetal heart rate (FHR) pattern? a. Altered fetal cerebral blood flow b. Umbilical cord compression c. Uteroplacental insufficiency d. Spontaneous rupture of membranes

Answer: Altered fetal cerebral blood flow Rationale: Early decelerations are the fetus' response to fetal head compression; these are considered benign, and interventions are not necessary. Variable decelerations are associated with umbilical cord compression. Late decelerations are associated with uteroplacental insufficiency. Spontaneous rupture of membranes has no bearing on the FHR unless the umbilical cord prolapses, which would result in variable or prolonged bradycardia.

Which definition of an acceleration in the fetal heart rate (FHR) is accurate? a. FHR accelerations are indications of fetal well-being when they are periodic. b. FHR accelerations are greater and longer in preterm gestations. c. FHR accelerations are usually observed with breech presentations when they are episodic. d. An acceleration in the FHR presents a visually apparent and abrupt peak.

Answer: An acceleration in the FHR presents a visually apparent and abrupt peak. Rationale: Acceleration of the FHR is defined as a visually apparent abrupt (only to peak 30 seconds) increase in the FHR above the baseline rate. Periodic accelerations occur with uterine contractions and are usually observed with breech presentations. Episodic accelerations occur during fetal movement and are indications of fetal well-being. Preterm accelerations peak at 10 beats per minute above the baseline and last for at least 10 seconds.

A woman in labor is breathing into a mouthpiece just before the start of her regular contractions. As she inhales, a valve opens, and gas is released. She continues to inhale the gas slowly and deeply until the contraction starts to subside. When the inhalation stops, the valve closes. Which statement regarding this procedure is correct? a. The application of nitrous oxide gas is not often used anymore. b. An inhalation of gas is likely to be used in the second stage of labor, not during the first stage. c. An application of nitrous oxide gas is administered for pain relief. d. The application of gas is a prelude to a cesarean birth.

Answer: An application of nitrous oxide gas is administered for pain relief. Rationale: A mixture of nitrous oxide with oxygen in a low concentration can be used in combination with other nonpharmacologic and pharmacologic measures for pain relief. This procedure is still commonly used in Canada and in the United Kingdom. Nitrous oxide inhaled in a low concentration will reduce but not eliminate pain during the first and second stages of labor. Nitrous oxide inhalation is not generally used before a caesarean birth. Nitrous oxide does not appear to depress uterine contractions or cause adverse reactions in the newborn.

While providing routine mother-baby care, which activities should the nurse encourage to facilitate the parent-infant attachment? a. The baby is able to return to the nursery at night so that the new mother can sleep. b. Routine times for care are established to reassure the parents. c. The father should be encouraged to go home at night to prepare for discharge of the mother and baby. d. An environment that fosters as much privacy as possible should be created.

Answer: An environment that fosters as much privacy as possible should be created. Rationale: Care providers need to knock before gaining entry. Nursing care activities should be grouped. Once the baby has demonstrated an adjustment to extrauterine life (either in the mother's room or the transitional nursery), all care should be provided in one location. This important principle of family-centered maternity care fosters attachment by offering parents the opportunity to learn about their infant 24 hours a day. One nurse should provide care to both mother and baby in this couplet care or rooming-in model. It is not necessary for the baby to return to the nursery at night. In fact, the mother will sleep better with the infant close by. Care should be individualized to meet the parents' needs, not the routines of the staff. Teaching goals should be developed in collaboration with the parents. The father or significant other should be permitted to sleep in the room with the mother. The maternity unit should develop policies that allow for the presence of significant others as much as the new mother desires.

Which statement concerning the third stage of labor is correct? a. The placenta eventually detaches itself from a flaccid uterus. b. An expectant or active approach to managing this stage of labor reduces the risk of complications. c. It is important that the dark, roughened maternal surface of the placenta appears before the shiny fetal surface. d. The major risk for women during the third stage is a rapid heart rate.

Answer: An expectant or active approach to managing this stage of labor reduces the risk of complications. Rationale: Active management facilitates placental separation and expulsion, reducing the risk of complications. The placenta cannot detach itself from a flaccid (relaxed) uterus. Which surface of the placenta comes out first is not clinically important. The major risk for women during the third stage of labor is postpartum hemorrhaging.

Which characteristic of a uterine contraction is not routinely documented? a. Frequency: how often contractions occur b. Intensity: strength of the contraction at its peak c. Resting tone: tension in the uterine muscle d. Appearance: shape and height ANS: D Uterine contractions are described in terms of frequency, intensity, duration, and resting tone. Appearance is not routinely charted.

Answer: Appearance: shape and height Rationale: Uterine contractions are described in terms of frequency, intensity, duration, and resting tone. Appearance is not routinely charted.

A woman gave birth 48 hours ago to a healthy infant girl. She has decided to bottle feed. During the assessment, the nurse notices that both breasts are swollen, warm, and tender on palpation. Which guidance should the nurse provide to the client at this time? a. Run warm water on her breasts during a shower. b. Apply ice to the breasts for comfort. c. Express small amounts of milk from the breasts to relieve the pressure. d. Wearing a loose-fitting bra to prevent nipple irritation.

Answer: Apply ice to the breasts for comfort. Rationale: Applying ice packs and cabbage leaves to the breasts for comfort is an appropriate intervention for treating engorgement in a mother who is bottle feeding. The ice packs should be applied for 15 minutes on and 45 minutes off to avoid rebound engorgement. A bottle-feeding mother should avoid any breast stimulation, including pumping or expressing milk. A bottle-feeding mother should continuously wear a well-fitted support bra or breast binder for at least the first 72 hours after giving birth. A loose-fitting bra will not aid lactation suppression. Furthermore, the shifting of the bra against the breasts may stimulate the nipples and thereby stimulate lactation.

Which intervention by the nurse would reduce the risk of abduction of the newborn from the hospital? a. Instructing the mother not to give her infant to anyone except the one nurse assigned to her that day b. Applying an electronic and identification bracelet to the mother and the infant c. Carrying the infant when transporting him or her in the halls d. Restricting the amount of time infants are out of the nursery

Answer: Applying an electronic and identification bracelet to the mother and the infant. Rationale: A measure taken by many facilities is to band both the mother and the baby with matching identification bracelets and band the infant with an electronic device that will sound an alarm if the infant is removed from the maternity unit. It is impossible for one nurse to be on call for one mother and baby for the entire shift; therefore, parents need to be able to identify the nurses who are working on the unit. Infants should always be transported in their bassinette for both safety and security reasons. All maternity unit nursing staff should have unique identification bracelets in comparison with the rest of the hospital. Infants should remain with their parents and spend as little time in the nursery as possible.

A laboring woman is reclining in the supine position. What is the most appropriate nursing action at this time? a. Ask her to turn to one side. b. Elevate her feet and legs. c. Take her blood pressure. d. Determine whether fetal tachycardia is present.

Answer: Ask her to turn to one side Rationale: The woman's supine position may cause the heavy uterus to compress her inferior vena cava, thus reducing blood return to her heart and reducing placental blood flow. Elevating her legs will not relieve the pressure from the inferior vena cava. If the woman is allowed to stay in the supine position and blood flow to the placental is reduced significantly, then fetal tachycardia may occur. The most appropriate nursing action is to prevent this from occurring by turning the woman to her side. Blood pressure readings may be obtained when the client is in the appropriate and safest position.

A woman tells the nurse that she thinks she has a vaginal infection, and has been using an over-the-counter cream for the past 2 days to treat it. How should the nurse initially respond? a. Determine when she first noticed the symptoms. b. Reassure the woman that using vaginal cream is not a problem for the examination. c. Ask the woman to describe the symptoms that indicate to her that she has a vaginal infection. d. Ask the woman to reschedule the appointment for the examination.

Answer: Ask the woman to describe the symptoms that indicate to her that she has a vaginal infection. Rationale: An important element of the health history and physical examination is the client's description of any symptoms she may be experiencing. The best response is for the nurse to inquire about the symptoms the woman is experiencing. While relevant, when the symptoms began is not as important as what the symptoms are. Women should not douche, use vaginal medications, or have sexual intercourse for 24 to 48 hours before obtaining a Pap test. Although the woman may need to reschedule a visit for her Pap test, her current symptoms should still be addressed.

When a nulliparous woman telephones the hospital to report that she is in labor, what intervention should the nurse prioritize? a. Instructing the woman to stay home until her membranes rupture. b. Emphasizing that food and fluid intake should stop. c. Arranging for the woman to come to the hospital for labor evaluation. d. Asking the woman to describe why she believes she is in labor.

Answer: Asking the woman to describe why she believes she is in labor.

The obstetric nurse is preparing the client for an emergency cesarean birth, with no time to administer spinal anesthesia, general anesthesia will be used. What is the greatest risk of administering general anesthesia to the client.? a. Respiratory depression b. Uterine relaxation c. Inadequate muscle relaxation d. Aspiration of stomach contents

Answer: Aspiration of stomach contents Rationale: Aspiration of acidic gastric contents with possible airway obstruction is a potentially fatal complication of general anesthesia. Respirations can be altered during general anesthesia, and the anesthesiologist will take precautions to maintain proper oxygenation. Uterine relaxation can occur with some anesthesia but can be monitored and prevented. Inadequate muscle relaxation can be improved with medication.

During a prenatal intake interview, the nurse is in the process of obtaining an initial assessment of a 21-year-old Hispanic client with limited English proficiency. Which intervention is the most important for the nurse to implement? a. Use maternity jargon to enable the client to become familiar with these terms. b. Speak quickly and efficiently to expedite the visit. c. Provide the client with handouts. d. Assess whether the client understands the discussion.

Answer: Assess whether the client understands the discussion. Rationale: Nurses contribute to health literacy by using simple, common words, avoiding jargon, and evaluating whether the client understands the discussion. Speaking slowly and clearly and focusing on what is important will increase understanding. Most client education materials are written at a level too high for the average adult and may not be useful for a client with limited English proficiency.

Which nursing intervention would result in an increase in maternal cardiac output? a. Change in position b. Oxytocin administration c. Regional anesthesia d. IV analgesic

Answer: Change in position Rationale: Maternal supine hypotension syndrome is caused by the weight and pressure of the gravid uterus on the ascending vena cava when the woman is in a supine position. This position reduces venous return to the woman's heart, as well as cardiac output, and subsequently reduces her blood pressure. The nurse can encourage the woman to change positions and to avoid the supine position. Oxytocin administration, regional anesthesia, and IV analgesic may reduce maternal cardiac output.

A woman who is pregnant for the first time is dilated 3 cm and having contractions every 5 minutes. She is groaning and perspiring excessively; she states that she did not attend childbirth classes. What is the optimal intervention for the nurse to provide initially? a. Notify the woman's health care provider. b. Administer the prescribed narcotic analgesic. c. Assure her that her labor will be over soon. d. Assist her with simple breathing and relaxation instructions.

Answer: Assist her with simple breathing and relaxation instructions Rationale: By reducing tension and stress, both focusing, and relaxation techniques will allow the woman in labor to rest and conserve energy for the task of giving birth. For those who have had no preparation, instruction in simple breathing and relaxation can be given in early labor and is often successful. The nurse can independently perform many functions in labor and birth, such as teaching and support. Pain medication may be an option for this client. However, the initial response of the nurse should include teaching the client about her options. The length of labor varies among individuals, but the first stage of labor is the longest. Providing false assurances will only cause the client more emotional distress. At 3 cm of dilation with contractions every 5 minutes, this woman has a significant amount of labor yet to experience.

An infant boy was delivered minutes ago. The nurse is conducting the initial assessment. Part of the assessment includes the Apgar score. When should the Apgar assessment be performed? a. Only if the newborn is in obvious distress b. Once by the obstetrician, just after the birth c. At least twice, 1 minute and 5 minutes after birth d. Every 15 minutes during the newborn's first hour after birth

Answer: At least twice, 1 minute and 5 minutes after birth Rationale: Apgar scoring is performed at 1 minute and at 5 minutes after birth. Scoring may continue at 5-minute intervals if the infant is in distress and requires resuscitation efforts. The Apgar score is performed on all newborns. Apgar score can be completed by the nurse or the birth attendant. The Apgar score permits a rapid assessment of the newborn's transition to extrauterine life. An interval of every 15 minutes is too long to wait to complete this assessment.

New parents express concern that because of the mother's emergency cesarean birth under general anesthesia, they did not have the opportunity to hold and bond with their daughter immediately after her birth. Which information should the nurse's response convey? a. Attachment, or bonding, is a process that occurs over time and does not require early contact. b. Time immediately after birth is a critical period for humans. c. Early contact is essential for optimal parent-infant relationships. d. These new parents should just be happy that the infant is healthy.

Answer: Attachment, or bonding, is a process that occurs over time and does not require early contact. Rationale: Attachment occurs over time and does not require early contact. Although a delay in contact does not necessarily mean that attachment is inhibited, additional psychologic energy may be necessary to achieve the same effect. The formerly accepted definition of bonding held that the period immediately after birth was critical for bonding to occur. Research since has indicated that parent-infant attachment occurs over time. A delay does not inhibit the process. Parent-infant attachment involves activities such as touching, holding, and gazing; it is not exclusively eye contact. Telling the parents that they should be happy that the infant is healthy is inappropriate; it may be received as derogatory and belittling.

The nurse is performing an initial assessment of a client in labor. What is the appropriate terminology for the relationship of the fetal body parts to one another? a. Lie b. Presentation c. Attitude d. Position

Answer: Attitude Rationale: Attitude is the relationship of the fetal body parts to one another. Lie is the relationship of the long axis (spine) of the fetus to the long axis (spine) of the mother. Presentation refers to the part of the fetus that enters the pelvic inlet first and leads through the birth canal during labor at term. Position is the relationship of the presenting part of the fetus to the four quadrants of the mother's pelvis.

After delivery, excess hypertrophied tissue in the uterus undergoes a period of self-destruction. What is the correct term for this process? a. Autolysis b. Subinvolution c. Afterpains d. Diastasis

Answer: Autolysis Rationale: Autolysis is caused by a decrease in hormone levels. Subinvolution is failure of the uterus to return to a nonpregnant state. Afterpains are caused by uterine cramps 2 to 3 days after birth. Diastasis refers to the separation of muscles.

Which newborn reflex is elicited by stroking the lateral sole of the infant's foot from the heel to the ball of the foot? a. Babinski b. Tonic neck c. Stepping d. Plantar grasp

Answer: Babinski Rationale: The Babinski reflex causes the toes to flare outward and the big toe to dorsiflex. The tonic neck reflex (also called the fencing reflex) refers to the posture assumed by newborns when in a supine position. The stepping reflex occurs when infants are held upright with their heel touching a solid surface and the infant appears to be walking. Plantar grasp reflex is similar to the palmar grasp reflex; when the area below the toes is touched, the infant's toes curl over the nurse's finger.

What is the rationale for the administration of vitamin K to the healthy full-term newborn? a. Most mothers have a diet deficient in vitamin K, which results in the infant being deficient. b. Vitamin K prevents the synthesis of prothrombin in the liver and must be administered by injection. c. Bacteria that synthesize vitamin K are not present in the newborn's intestinal tract. d. The supply of vitamin K in the healthy full-term newborn is inadequate for at least 3 to 4 months and must be supplemented.

Answer: Bacteria that synthesize vitamin K are not present in the newborn's intestinal tract. Rationale: Vitamin K is provided because the newborn does not have the intestinal flora to produce this vitamin for the first week. The maternal diet has no bearing on the amount of vitamin K found in the newborn. Vitamin K promotes the formation of clotting factors in the liver and is used for the prevention and treatment of hemorrhagic disease in the newborn. Vitamin K is not produced in the intestinal tract of the newborn until after microorganisms are introduced. By day 8, normal newborns are able to produce their own vitamin K.

Which statement regarding female sexual response is inaccurate? a. Women and men are more alike than different in their physiologic response to sexual arousal and orgasm. b. Vasocongestion is the congestion of blood vessels. c. Orgasmic phase is the final state of the sexual response cycle. d. Facial grimaces and spasms of the hands and feet are often part of arousal.

Answer: Orgasmic phase is the final state of the sexual response cycle. Rationale: The final state of the sexual response cycle is the resolution phase after orgasm. Men and women are surprisingly alike. Vasocongestion causes vaginal lubrication and engorgement of the genitals. Arousal is characterized by increased muscular tension (myotonia).

Which key point is important for the nurse to understand regarding the perinatal continuum of care? a. Begins with conception and ends with the birth b. Begins with family planning and continues until the infant is 1 year old c. Begins with prenatal care and continues until the newborn is 24 weeks old d. Refers to home care only

Answer: Begins with family planning and continues until the infant is 1 year old Rationale: The perinatal continuum of care begins with family planning and continues until the infant is 1 year old. It takes place both at home and in health care facilities. The perinatal continuum does not end with the birth. The perinatal continuum begins before conception and continues after the birth. Home care is one delivery component; health care facilities are another.

Due to the effects of cyclic ovarian changes in the breast, when is the best time for breast self-examination (BSE)? a. Between 5 and 7 days after menses ceases b. Day 1 of the endometrial cycle c. Midmenstrual cycle d. Any time during a shower or bath

Answer: Between 5 and 7 days after menses ceases Rationale: The physiologic alterations in breast size and activity reach their minimal level approximately 5 to 7 days after menstruation ceases. Therefore, BSE is best performed during this phase of the menstrual cycle. Day 1 of the endometrial cycle is too early to perform an accurate BSE. After the midmenstrual cycle, breasts are likely to become tender and increase in size, which is not the ideal time to perform BSE. Lying down after a shower or bath with a small towel under the shoulder of the side being examined is appropriate teaching for BSE. A secondary BSE may be performed while in the shower.

After birth, a crying infant may be soothed by being held in a position in which the newborn can hear the mother's heartbeat. This phenomenon is known as what? a. Entrainment b. Reciprocity c. Synchrony d. Biorhythmicity

Answer: Biorhythmicity Rationale: The newborn is in rhythm with the mother. The infant develops a personal biorhythm with the parents' help over time. Entrainment is the movement of a newborn in time to the structure of adult speech. Reciprocity is body movement or behavior that gives cues to the person's desires. These take several weeks to develop with a new baby. Synchrony is the fit between the infant's behavioral cues and the parent's responses.

Which statement related to fetal positioning during labor is correct and important for the nurse to understand? a. Position is a measure of the degree of descent of the presenting part of the fetus through the birth canal. b. Birth is imminent when the presenting part is at +4 to +5 cm below the spine. c. The largest transverse diameter of the presenting part is the suboccipitobregmatic diameter. d. Engagement is the term used to describe the beginning of labor.

Answer: Birth is imminent when the presenting part is at +4 to +5 cm below the spine. Rationale: The station of the presenting part should be noted at the beginning of labor to determine the rate of descent. Position is the relationship of the presenting part of the fetus to the four quadrants of the mother's pelvis; station is the measure of degree of descent. The largest diameter is usually the biparietal diameter. The suboccipitobregmatic diameter is the smallest, although one of the most critical. Engagement often occurs in the weeks just before labor in nulliparous women and before or during labor in multiparous women.

The unique muscle fibers that constitute the uterine myometrium make it ideally suited for what? a. Menstruation b. Birth process c. Ovulation d. Fertilization

Answer: Birth process Rationale: The myometrium is made up of layers of smooth muscle that extend in three directions. These muscles assist in the birth process by expelling the fetus, ligating blood vessels after birth, and controlling the opening of the cervical os. These muscle fibers have no contribution to the process of menstruation, ovulation or fertilization.

A nurse caring for a pregnant client should be aware that the U.S. birth rate shows what trend? a. Births to unmarried women are more likely to have less favorable outcomes. b. Birth rates for women 40 to 44 years of age are declining. c. Cigarette smoking among pregnant women continues to increase. d. Rates of pregnancy and abortion among teenagers are lower in the United States than in any other industrialized country.

Answer: Births to unmarried women are more likely to have less favorable outcomes. Rationale: LBW infants and preterm births are more likely because of the large number of teenagers in the unmarried group. Birth rates for women in their early 40s continue to increase. Fewer pregnant women smoke. Teen pregnancy and abortion rates are higher in the United States than in any other industrial country.

Which body part both protects the pelvic structures and accommodates the growing fetus during pregnancy? a. Perineum b. Bony pelvis c. Vaginal vestibule d. Fourchette

Answer: Bony pelvis Rationale: The bony pelvis protects and accommodates the growing fetus. The perineum covers the pelvic structures. The vaginal vestibule contains openings to the urethra and vagina. The area of thin, flat tissue called the fourchette is formed by the labia minor and is found underneath the vaginal opening.

A pregnant woman wants to breastfeed her infant; however, her partner is not convinced that there are any scientific reasons to do so. The nurse can give the couple printed information comparing breastfeeding and bottle feeding. Which statement regarding bottle feeding using commercially prepared infant formulas might influence their choice? a. Bottle feeding using a commercially prepared formula increases the risk that the infant will develop allergies. b. Bottle feeding helps the infant sleep through the night. c. Commercially prepared formula ensures that the infant is getting iron in a form that is easily absorbed. d. Bottle feeding requires that multivitamin supplements be given to the infant.

Answer: Bottle feeding using a commercially prepared formula increases the risk that the infant will develop allergies. Rationale: Exposure to cow's milk poses a risk of developing allergies, eczema, and asthma. Newborns should be fed during the night, regardless of the feeding method. Iron is better absorbed from breast milk than from formula. Commercial formulas are designed to meet the nutritional needs of the infant and to resemble breast milk. No supplements are necessary.

A client is experiencing back labor and reports intense pain in her lower back. Which measure provided by the woman's labor coach would best support this woman in labor? a. Counterpressure against the sacrum b. Pant-blow (breaths and puffs) breathing techniques c. Effleurage d. Conscious relaxation or guided imagery

Answer: Counterpressure against the sacrum Rationale: Counterpressure is steady pressure applied by a support person to the sacral area with the fist or heel of the hand. This technique helps the woman cope with the sensations of internal pressure and pain in the lower back. The pain management techniques of pant-blow, effleurage, and conscious relaxation or guided imagery are usually helpful for contractions per the gate-control theory.

The breastfeeding mother should be taught a safe method to remove the breast from the baby's mouth. Which suggestion by the nurse is most appropriate? a. Slowly remove the breast from the baby's mouth when the infant has fallen asleep and the jaws are relaxed. b. Break the suction by inserting your finger into the corner of the infant's mouth. c. A popping sound occurs when the breast is correctly removed from the infant's mouth. d. Elicit the Moro reflex to wake the baby and remove the breast when the baby cries.

Answer: Break the suction by inserting your finger into the corner of the infant's mouth. Rationale: Inserting a finger into the corner of the baby's mouth between the gums to break the suction avoids trauma to the breast. The infant who is sleeping may lose grasp on the nipple and areola, resulting in chewing on the nipple that makes it sore. A popping sound indicates improper removal of the breast from the baby's mouth and may cause cracks or fissures in the breast. Most mothers prefer the infant to continue to sleep after the feeding. Gentle wake-up techniques are recommended.

A breastfeeding woman develops engorged breasts at 3 days postpartum. What action will help this client achieve her goal of reducing the engorgement? a. Skip feedings to enable her sore breasts to rest. b. Avoid using a breast pump. c. Breastfeed her infant every 2-3 hours. d. Reduce her fluid intake for 24 hours.

Answer: Breastfeed her infant every 2-3 hours. Rationale: The mother should be instructed to attempt feeding her infant every 2-3 hours while massaging the breasts as the infant is feeding. Skipping feedings may cause further swelling and discomfort. If the infant does not adequately feed and empty the breast, then the mother may pump to extract the milk and relieve some of the discomfort. Dehydration further irritates swollen breast tissue

While discussing the societal impacts of breastfeeding, the nurse should be cognizant of the benefits and educate the client accordingly. Which statement as part of this discussion would be incorrect? a. Breastfeeding requires fewer supplies and less cumbersome equipment. b. Breastfeeding saves families money. c. Breastfeeding costs employers in terms of time lost from work. d. Breastfeeding benefits the environment.

Answer: Breastfeeding costs employers in terms of time lost from work. Rationale: Actually, less time is lost to work by breastfeeding mothers, in part because infants are healthier. Breastfeeding is convenient because it does not require cleaning or transporting bottles and other equipment. It saves families money because the cost of formula far exceeds the cost of extra food for the lactating mother. Breastfeeding uses a renewable resource; it does not need fossil fuels, advertising, shipping, or disposal.

Which statement regarding the nutrient needs of breastfed infants is correct? a. Breastfed infants need extra water in hot climates. b. During the first 3 months, breastfed infants consume more energy than formula-fed infants. c. Breastfeeding infants should receive 400 IU of oral vitamin D drops daily. d. Vitamin K injections at birth are not necessary for breastfed infants.

Answer: Breastfeeding infants should receive 400 IU of oral vitamin D drops daily. Rationale: Human milk contains only small amounts of vitamin D. All infants who are breastfed should receive 400 International Units of vitamin D each day. Neither breastfed nor formula-fed infants need to be fed water, not even in very hot climates. During the first 3 months, formula-fed infants consume more energy than breastfed infants and therefore tend to grow more rapidly. Vitamin K shots are required for all infants because the bacteria that produce it are absent from the baby's stomach at birth.

The nurse is explaining the benefits associated with breastfeeding to a new mother. Which statement by the nurse would provide conflicting information to the client? a. Women who breastfeed have a decreased risk of breast cancer. b. Breastfeeding is an effective method of birth control. c. Breastfeeding increases bone density. d. Breastfeeding may enhance postpartum weight loss.

Answer: Breastfeeding is an effective method of birth control. Rationale: Although breastfeeding delays the return of fertility, it is not an effective birth control method. Women who breastfeed have a decreased risk of breast cancer, an increase in bone density, and a possibility of faster postpartum weight loss.

A 62-year-old woman has not been to the clinic for an annual examination for 5 years. The recent death of her husband reminded her that she should come for a visit. Her family physician has retired, and she is going to see the women's health nurse practitioner for her visit. What should the nurse do to facilitate a positive health care experience for this client? a. Remind the woman that she is long overdue for her examination and that she should come in annually. b. Carefully listen and allow extra time for this woman's health history interview. c. Reassure the woman that a nurse practitioner is just as good as her old physician. d. Encourage the woman to talk about the death of her husband and her fears about her own death.

Answer: Carefully listen and allow extra time for this woman's health history interview. Rationale: The nurse has an opportunity to use reflection and empathy while listening, as well as ensure an open and caring communication. Scheduling a longer appointment time may be necessary because older women may have longer histories or may need to talk. A respectful and reassuring approach to caring for women older than age 50 years can help ensure that they continue to seek health care. Reminding the woman about her overdue examination, reassuring the woman that she has a good practitioner, and encouraging conversation about the death of her husband and her own death are not the best approaches.

The nurse has received a report regarding a client in labor. The woman's last vaginal examination was recorded as 3 cm, 30%, and -2. What is the nurse's interpretation of this assessment? a. Cervix is effaced 3 cm and dilated 30%; the presenting part is 2 cm above the ischial spines. b. Cervix is dilated 3 cm and effaced 30%; the presenting part is 2 cm above the ischial spines. c. Cervix is effaced 3 cm and dilated 30%; the presenting part is 2 cm below the ischial spines. d. Cervix is dilated 3 cm and effaced 30%; the presenting part is 2 cm below the

Answer: Cervix is dilated 3 cm and effaced 30%; the presenting part is 2 cm above the ischial spines. Rationale: The sterile vaginal examination is recorded as centimeters of cervical dilation, percentage of cervical dilation, and the relationship of the presenting part to the ischial spines (either above or below). For this woman, the cervix is dilated 3 cm and effaced 30%, and the presenting part is 2 cm above the ischial spines. The first interpretation of this vaginal examination is incorrect; the cervix is dilated 3 cm and is 30% effaced. However, the presenting part is correct at 2 cm above the ischial spines. The remaining two interpretations of this vaginal examination are incorrect. Although the dilation and effacement are correct at 3 cm and 30%, the presenting part is actually 2 cm above the ischial spines.

While evaluating an external monitor tracing of a woman in active labor, the nurse notes that the fetal heart rate (FHR) for five sequential contractions begins to decelerate late in the contraction, with the nadir of the decelerations occurring after the peak of the contraction. What is the nurse's first priority? a. Change the woman's position. b. Notify the health care provider. c. Assist with amnioinfusion d. Insert a scalp electrode.

Answer: Change the woman's position. Rationale: Late FHR decelerations may be caused by maternal supine hypotension syndrome. These decelerations are usually corrected when the woman turns onto her side to displace the weight of the gravid uterus from the vena cava. If the fetus does not respond to primary nursing interventions for late decelerations, then the nurse should continue with subsequent intrauterine resuscitation measures and notify the health care provider. An amnioinfusion may be used to relieve pressure on an umbilical cord that has not prolapsed. The FHR pattern associated with this situation most likely will reveal variable decelerations. Although a fetal scalp electrode will provide accurate data for evaluating the well-being of the fetus, it is not a nursing intervention that will alleviate late decelerations nor is it the nurse's first priority.

A nurse hears a primiparous woman talking to her son and telling him that his chin is just like his dad's. This statement is most descriptive of which process? a. Mutuality b. Synchrony c. Claiming d. Reciprocity ANS: C Claiming refers to the process by which the child is identified in terms of likeness to other family members. Mutuality occurs when the infant's behaviors and characteristics call forth a corresponding set of maternal behaviors and characteristics. Synchrony refers to the "fit" between the infant's cues and the parent's responses. Reciprocity is a type of body movement or behavior that provides the observer with cues.

Answer: Claiming

Which information regarding the procedures and criteria for admitting a woman to the hospital labor unit is important for the nurse to understand? a. Client is considered in active labor when she arrives at the facility with contractions. b. Client can have only her male partner or predesignated doula with her at assessment. c. Children are not allowed on the labor unit. d. Non-English speaking client must bring someone to translate.

Answer: Client is considered in active labor when she arrives at the facility with contractions Rationale: According to the Emergency Medical Treatment and Active Labor Act (EMTALA), a woman is entitled to active labor care and is presumed to be in true labor until a qualified health care provider certifies otherwise. A woman may have anyone she wishes present for her support. An interpreter must be provided by the hospital, either in person or by a telephonic service. Siblings of the new infant may be allowed at the delivery, depending on hospital policy and adequate preparation and supervision.

Which client would not be a suitable candidate for internal electronic fetal monitoring (EFM)? a. Client who still has intact membranes b. Woman whose fetus is well engaged in the pelvis c. Pregnant woman who has a comorbidity of obesity d. Client whose cervix is dilated to 4 to 5 cm

Answer: Client who still has intact membranes Rationale: For internal EFM, the membranes must have ruptured and the cervix must be dilated at least 2 to 3 cm. The presenting part must be low enough to allow placement of the spiral electrode necessary for internal EFM. The accuracy of EFM is not affected by maternal size. However, evaluating fetal well-being using external EFM may be more difficult on an obese client. The client whose cervix is dilated to 4 to 5 cm is indeed a candidate for internal monitoring.

Ovarian function and hormone production decline during which transitional phase? a. Climacteric b. Menarche c. Menopause d. Puberty

Answer: Climacteric Rationale: The climacteric phase is a transitional period during which ovarian function and hormone production decline. Menarche is the term that denotes the first menstruation. Menopause refers only to the last menstrual period. Puberty is a broad term that denotes the entire transitional period between childhood and sexual maturity.

A new mother asks whether she should feed her newborn colostrum, because it is not "real milk." What is the nurse's most appropriate answer? a. Colostrum is high in antibodies, protein, vitamins, and minerals. b. Colostrum is lower in calories than milk and should be supplemented by formula. c. Giving colostrum is important in helping the mother learn how to breastfeed before she goes home. d. Colostrum is unnecessary for newborns.

Answer: Colostrum is high in antibodies, protein, vitamins, and minerals. Rationale: Colostrum is important because it has high levels of the nutrients needed by the neonate and helps protect against infection. Supplementation is not necessary and will decrease stimulation to the breast and decrease the production of milk. It is important for the mother to feel comfortable in this role before discharge; however, the importance of the colostrum to the infant is the top priority. Colostrum provides immunities and enzymes necessary to cleanse the gastrointestinal system, among other things.

A client is concerned that her breasts are engorged and uncomfortable. What is the nurse's explanation for this physiologic change? a. Overproduction of colostrum b. Accumulation of milk in the lactiferous ducts and glands c. Hyperplasia of mammary tissue d. Congestion of veins and lymphatic vessels .

Answer: Congestion of veins and lymphatic vessels Rationale: Breast engorgement is caused by the temporary congestion of veins and lymphatic vessels. An overproduction of colostrum, an accumulation of milk in the lactiferous ducts and glands, and hyperplasia of mammary tissue do not cause breast engorgement.

What are the legal responsibilities of the perinatal nurses? a. Correctly interpreting fetal heartrate (FHR) patterns, initiating appropriate nursing interventions, and documenting the outcomes b. Greeting the client on arrival, assessing her status, and starting an IV line c. Applying the external fetal monitor and notifying the health care provider d. Ensuring that the woman is comfortable

Answer: Correctly interpreting fetal heartrate (FHR) patterns, initiating appropriate nursing interventions, and documenting the outcomes Rationale: Nurses who care for women during childbirth are legally responsible for correctly interpreting FHR patterns, initiating appropriate nursing interventions based on those patterns, and documenting the outcomes of those interventions. Greeting the client on arrival, assessing her, and starting an IV line are activities that should be performed when any client arrives to the maternity unit. The nurse is not the only one legally responsible for performing these functions. Applying the external fetal monitor and notifying the health care provider is a nursing function that is part of the standard of care for all obstetric clients and falls within the registered nurse's scope of practice. Everyone caring for the pregnant woman should ensure that both she and her support partner are comfortable.

A nurse is responsible for teaching new parents regarding the hygienic care of their newborn. Which instruction should the nurse provide regarding bathing? a. Avoid washing the head for at least 1 week to prevent heat loss. b. Sponge bathe the newborn for the first month of life. c. Cleanse the ears and nose with cotton-tipped swabs, such as Q-tips. d. Create a draft-free environment of at least 26° C (75° F) when bathing the infant. ANS: D The temperature of the room should be least 26° C (75° F), and the bathing area should be free of drafts. To prevent heat loss, the infant's head should be bathed before unwrapping and undressing. Tub baths may be initiated from birth. Ensure that the infant is fully immersed. Q-tips should not be used; they may cause injury. A corner of a moistened washcloth should be twisted into shape so that it can be used to cleanse the ears and nose.

Answer: Create a draft-free environment of at least 26° C (75° F) when bathing the infant. Rationale: The temperature of the room should be least 26° C (75° F), and the bathing area should be free of drafts. To prevent heat loss, the infant's head should be bathed before unwrapping and undressing. Tub baths may be initiated from birth. Ensure that the infant is fully immersed. Q-tips should not be used; they may cause injury. A corner of a moistened washcloth should be twisted into shape so that it can be used to cleanse the ears and nose.

Which description of the phases of the second stage of labor is most accurate? a. Latent phase: Feeling sleepy; fetal station 2+ to 4+; duration of 30 to 45 minutes b. Active phase: Overwhelmingly strong contractions; Ferguson reflux activated; duration of 5 to 15 minutes c. Descent phase: Significant increase in contractions; Ferguson reflux activated; average duration varies d. Transitional phase: Woman "laboring down"; fetal station 0; duration of 15 minutes

Answer: Descent phase: Significant increase in contractions; Ferguson reflux activated; average duration varies Rationale: The descent phase begins with a significant increase in contractions; the Ferguson reflex is activated, and the duration varies, depending on several factors. The latent phase is the lull or "laboring down" period at the beginning of the second stage and lasts 10 to 30 minutes on average. The second stage of labor has no active phase. The transition phase is the final phase in the second stage of labor; contractions are strong and painful.

When assessing a woman in the first stage of labor, which clinical finding will alert the nurse that uterine contractions are effective? a. Dilation of the cervix b. Descent of the fetus to -2 station c. Rupture of the amniotic membranes (ROM) d. Increase in bloody show

Answer: Dilation of the cervix Rationale: The vaginal examination reveals whether the woman is in true labor. Cervical change, especially dilation, in the presence of adequate labor, indicates that the woman is in true labor. Engagement and descent of the fetus are not synonymous and may occur before labor. ROM may occur with or without the presence of labor. Bloody show may indicate a slow, progressive cervical change (e.g., effacement) in both true and false labor.

Where is the point of maximal intensity (PMI) of the fetal heart tone (FHR) located? a. Usually directly over the fetal abdomen b. In a vertex position, heard above the mother's umbilicus c. Directly over the fetal back d. In a breech position, heard below the mother's umbilicus

Answer: Directly over the fetal back Rationale: Nurses should be prepared for the shift. The PMI of the FHR is usually directly over the fetal back. In a vertex position, the PMI of the FHR is heard below the mother's umbilicus. In a breech position, it is heard above the mother's umbilicus.

A primigravida at 39 weeks of gestation is observed for 2 hours in the intrapartum unit. The fetal heart rate (FHR) has been normal. Contractions are 5 to 9 minutes apart, 20 to 30 seconds in duration, and of mild intensity. Cervical dilation is 1 to 2 cm and uneffaced (unchanged from admission). Membranes are intact. What disposition would the nurse anticipate? a. Admitted and prepared for a cesarean birth b. Admitted for extended observation c. Discharged home with a sedative d. Discharged home to await the onset of true labor

Answer: Discharged home to await the onset of true labor Rationale: This situation describes a woman with normal assessments who is probably in false labor and will likely not deliver rapidly once true labor begins. No further assessments or observations are indicated; therefore, the client will be discharged along with instructions to return when contractions increase in intensity and frequency. Neither a cesarean birth nor a sedative is required at this time.

The nurse performs a vaginal examination to assess a client's labor progress. Which action should the nurse take next? a. Perform an examination at least once every hour during the active phase of labor. b. Perform the examination with the woman in the supine position. c. Wear two clean gloves for each examination. d. Discuss the findings with the woman and her partner.

Answer: Discuss the findings with the woman and her partner. Rationale: The nurse should discuss the findings of the vaginal examination with the woman and her partner, as well as report the findings to the primary care provider. A vaginal examination should be performed only when indicated by the status of the woman and her fetus. The woman should be positioned so as to avoid supine hypotension. The examiner should wear a sterile glove while performing a vaginal examination for a laboring woman.

While assessing the integument of a 24-hour-old newborn, the nurse notes a pink papular rash with vesicles superimposed on the thorax, back, and abdomen. What action is the highest priority for the nurse to take immediately? a. Immediately notify the health care professional. b. Move the newborn to an isolation nursery. c. Document the finding as erythema toxicum neonatorum. d. Take the newborn's temperature and obtain a culture of one of the vesicles.

Answer: Document the finding as erythema toxicum neonatorum. Rationale: Erythema toxicum neonatorum (or erythema neonatorum) is a newborn rash that resembles flea bites. Documentation of the condition is the priority. Notification of the health care provider, isolation of the newborn, or additional interventions are not necessary when erythema toxicum neonatorum is present.

Which stage of labor varies the most in length? a. First b. Second c. Third d. Fourth

Answer: First Rationale: The first stage of labor is considered to last from the onset of regular uterine contractions to the full dilation of the cervix. The first stage is significantly longer than the second and third stages combined. In a first-time pregnancy, the first stage of labor can take up to 20 hours. The second stage of labor lasts from the time the cervix is fully dilated to the birth of the fetus. The average length is 20 minutes for a multiparous woman and 50 minutes for a nulliparous woman. The third stage of labor lasts from the birth of the fetus until the placenta is delivered. This stage may be as short as 3 minutes or as long as 1 hour. The fourth stage of labor, recovery, lasts approximately 2 hours after the delivery of the placenta.

The uterine contractions of a woman early in the active phase of labor are assessed by an internal uterine pressure catheter (IUPC). The uterine contractions occur every 3 to 4 minutes and last an average of 55 to 60 seconds. They are becoming more regular and are moderate to strong. Based on this information, what would a prudent nurse do next? a. Immediately notify the woman's primary health care provider. b. Prepare to administer an oxytocic to stimulate uterine activity. c. Document the findings because they reflect the expected contraction pattern for the active phase of labor. d. Prepare the woman for the onset of the second stage of labor.

Answer: Document the findings because they reflect the expected contraction pattern for the active phase of labor. Rationale: The nurse is responsible for monitoring the uterine contractions to ascertain whether they are powerful and frequent enough to accomplish the work of expelling the fetus and the placenta. In addition, the nurse documents these findings in the client's medical record. This labor pattern indicates that the client is in the active phase of the first stage of labor. Nothing indicates a need to notify the primary health care provider at this time. Oxytocin augmentation is not needed for this labor pattern; this contraction pattern indicates that the woman is in active labor. Her contractions will eventually become stronger, last longer, and come closer together during the transition phase of the first stage of labor. The transition phase precedes the second stage of labor, or delivery of the fetus.

Which intervention can nurses use to prevent evaporative heat loss in the newborn? a. Drying the baby after birth, and wrapping the baby in a dry blanket b. Keeping the baby out of drafts and away from air conditioners c. Placing the baby away from the outside walls and windows d. Warming the stethoscope and the nurse's hands before touching the baby

Answer: Drying the baby after birth, and wrapping the baby in a dry blanket Rationale: Because the infant is wet with amniotic fluid and blood, heat loss by evaporation quickly occurs. Heat loss by convection occurs when drafts come from open doors and air currents created by people moving around. If the heat loss is caused by placing the baby near cold surfaces or equipment, it is referred to as a radiation heat loss. Conduction heat loss occurs when the baby comes in contact with cold surfaces.

The Valsalva maneuver can be described as the process of making a forceful bearing-down attempt while holding one's breath with a closed glottis and a tightening of the abdominal muscles. When is it appropriate to instruct the client to use this maneuver? a. During the second stage to enhance the movement of the fetus b. During the third stage to help expel the placenta c. During the fourth stage to expel blood clots d. During no stage of labor ANS: D The client should not be instructed to use this maneuver. This process stimulates the parasympathetic division of the autonomic nervous system and produces a vagal response (decrease in heart rate and blood pressure.) An alternative method includes instructing the client to perform open-mouth and open-glottis breathing and pushing.

Answer: During no stage of labor Rationale: The client should not be instructed to use this maneuver. This process stimulates the parasympathetic division of the autonomic nervous system and produces a vagal response (decrease in heart rate and blood pressure.) An alternative method includes instructing the client to perform open-mouth and open-glottis breathing and pushing.

When caring for a newly delivered woman, what is the best measure for the nurse to implement to prevent abdominal distention after a cesarean birth? a. Rectal suppositories b. Early and frequent ambulation c. Tightening and relaxing abdominal muscles d. Carbonated beverages

Answer: Early and frequent ambulation Rationale: Activity will aid the movement of accumulated gas in the gastrointestinal tract that results in abdominal distention. Rectal suppositories can be helpful after distention occurs; however, they do not prevent it. Ambulation is the best prevention. Carbonated beverages may increase distention.

Nurses should be cognizant of what regarding the mechanism of labor? a. Seven critical movements usually progress in a orderly sequence. b. Asynclitism is sometimes achieved by means of the Leopold's maneuver. c. Effects of the forces determining descent are modified by the shape of the woman's pelvis and the size of the fetal head. d. At birth, the baby is said to achieve "restitution"; that is, a return to the C-shape of the womb. A

Answer: Effects of the forces determining descent are modified by the shape of the woman's pelvis and the size of the fetal head. Rationale: The size of the maternal pelvis and the ability of the fetal head to mold also affect the process. The seven identifiable movements of the mechanism of labor simultaneously occur in combinations, not in precise sequences. Asynclitism is the deflection of the baby's head; the Leopold's maneuver is a means of judging descent by palpating the mother's abdomen. Restitution is the rotation of the baby's head after the infant is born.

Maternity nurses often must answer questions about the many, sometimes unusual, ways people have tried to make the birthing experience more comfortable. Which information regarding nonpharmacologic pain relief is accurate? a. Music supplied by the support person must be discouraged because it could disturb others or upset the hospital routine. b. Women in labor can benefit from sitting in a bathtub, but they must limit immersion to no longer than 15 minutes at a time. c. Effleurage is permissible, but counterpressure is almost always counterproductive. d. Electrodes attached to either side of the spine to provide high-intensity electrical impulses facilitate the release of endorphins.

Answer: Electrodes attached to either side of the spine to provide high-intensity electrical impulses facilitate the release of endorphins. Rationale: Transcutaneous electrical nerve stimulation (TENS) may help and is most useful for lower back pain that occurs during the first stage of labor. Music may be very helpful for reducing tension and certainly can be accommodated by the hospital. Women can stay in a bath for as long as they want, although repeated baths with breaks might be more effective than one long bath. Counterpressure can help the woman cope with lower back pain.

Which part of the menstrual cycle includes the stimulated release of gonadotropin-releasing hormone (GnRH) and follicle-stimulating hormone (FSH)? a. Menstrual phase b. Endometrial cycle c. Ovarian cycle d. Hypothalamic-pituitary cycle

Answer: Hypothalamic-pituitary cycle Rationale: The cyclic release of hormones is the function of the hypothalamus and pituitary glands. The menstrual cycle is a complex interplay of events that simultaneously occur in the endometrium, hypothalamus, pituitary glands, and ovaries. The endometrial cycle consists of four phases: menstrual phase, proliferative phase, secretory phase, and ischemic phase. The ovarian cycle remains under the influence of FSH and estrogen.

Preconception and prenatal care have become important components of women's health. What is the guiding principal of preconception care? a. Ensure that pregnancy complications do not occur. b. Identify the woman who should not become pregnant. c. Encourage healthy lifestyles for families desiring pregnancy. d. Ensure that women know about prenatal care.

Answer: Encourage healthy lifestyles for families desiring pregnancy. Rationale: Preconception counseling guides couples in how to avoid unintended pregnancies, how to identify and manage risk factors in their lives and in their environment, and how to identify healthy behaviors that promote the well-being of the woman and her potential fetus. Preconception care does not ensure that pregnancy complications will not occur. In many cases, problems can be identified and treated and may not recur in subsequent pregnancies. For many women, counseling can allow behavior modification before any damage is done, or a woman can make an informed decision about her willingness to accept potential hazards. If a woman is seeking preconception care, then she is likely aware of prenatal care.

Part of the nurse's role is assisting with pushing and positioning. Which guidance should the nurse provide to her client in active labor? a. Encourage the woman's cooperation in avoiding the supine position. b. Advise the woman to avoid the semi-Fowler position. c. Encourage the woman to hold her breath and tighten her abdominal muscles to produce a vaginal response. d. Instruct the woman to open her mouth and close her glottis, letting air escape after the push.

Answer: Encourage the woman's cooperation in avoiding the supine position. Rationale: The woman should maintain a side-lying position. The supine position can cause maternal hypotension, which impairs placental perfusion and fetal oxygenation. The semi-Fowler position is the recommended side-lying position with a lateral tilt to the uterus. Encouraging the woman to hold her breath and tighten her abdominal muscles is the Valsalva maneuver, which should be avoided. Both the mouth and glottis should be open, allowing air to escape during the push.

When managing the care of a woman in the second stage of labor, the nurse uses various measures to enhance the progress of fetal descent. Which instruction best describes these measures? a. Encouraging the woman to try various upright positions, including squatting and standing b. Telling the woman to start pushing as soon as her cervix is fully dilated c. Continuing an epidural anesthetic so pain is reduced and the woman can relax d. Coaching the woman to use sustained, 10- to 15-second, closed-glottis bearing-down efforts with each contraction

Answer: Encouraging the woman to try various upright positions, including squatting and standing Rationale: Both upright and squatting positions may enhance the progress of fetal descent. Many factors dictate when a woman should begin pushing. Complete cervical dilation is necessary, but complete dilation is only one factor. If the fetal head is still in a higher pelvic station, then the physician or midwife may allow the woman to "labor down" if the woman is able (allowing more time for fetal descent and thereby reducing the amount of pushing needed). The epidural may mask the sensations and muscle control needed for the woman to push effectively. Closed glottic breathing may trigger the Valsalva maneuver, which increases intrathoracic and cardiovascular pressures, reducing cardiac output and inhibiting perfusion of the uterus and placenta. In addition, holding her breath for longer than 5 to 7 seconds diminishes the perfusion of oxygen across the placenta and results in fetal hypoxia.

Which statement regarding the care of a client in labor is correct and important to the nurse in the formulation of the plan of care? a. The woman's blood pressure will increase during contractions and fall back to prelabor normal levels between contractions. b. The use of the Valsalva maneuver is encouraged during the second stage of labor to relieve fetal hypoxia. c. Having the woman point her toes will reduce leg cramps. d. Endogenous endorphins released during labor will raise the woman's pain threshold and produce sedation.

Answer: Endogenous endorphins released during labor will raise the woman's pain threshold and produce sedation. Rationale: The endogenous endorphins released during labor will raise the woman's pain threshold and produce sedation. In addition, physiologic anesthesia of the perineal tissues, caused by the pressure of the presenting part, decreases the mother's perception of pain. Blood pressure levels increase during contractions but remain somewhat elevated between them. The use of the Valsalva maneuver is discouraged during the second stage labor because of a number of unhealthy outcomes, including fetal hypoxia. Pointing the toes can cause leg cramps, as can the process of labor itself.

Which action is correct when palpation is used to assess the characteristics and pattern of uterine contractions? a. Placing the hand on the abdomen below the umbilicus and palpating uterine tone with the fingertips b. Determining the frequency by timing from the end of one contraction to the end of the next contraction c. Evaluating the intensity by pressing the fingertips into the uterine fundus d. Assessing uterine contractions every 30 minutes throughout the first stage of labor

Answer: Evaluating the intensity by pressing the fingertips into the uterine fundus Rationale: The nurse or primary health care provider may assess uterine activity by palpating the fundal section of the uterus using the fingertips. Many women may experience labor pain in the lower segment of the uterus, which may be unrelated to the firmness of the contraction detectable in the uterine fundus. The frequency of uterine contractions is determined by palpating from the beginning of one contraction to the beginning of the next contraction. Assessment of uterine activity is performed in intervals based on the stage of labor. As labor progresses, this assessment is performed more frequently.

What physiologic change occurs as the result of increasing the infusion rate of nonadditive IV fluids? a. Maintaining normal maternal temperature b. Preventing normal maternal hypoglycemia c. Increasing the oxygen-carrying capacity of the maternal blood d. Expanding maternal blood volume

Answer: Expanding maternal blood volume Rationale: Filling the mother's vascular system increases the amount of blood available to perfuse the placenta and may correct hypotension. Increasing fluid volume may alter the maternal temperature only if she is dehydrated. Most IV fluids for laboring women are isotonic and do not provide extra glucose. Oxygen-carrying capacity is increased by adding more red blood cells.

What is the correct placement of the tocotransducer for effective electronic fetal monitoring (EFM)? a. Over the uterine fundus b. On the fetal scalp c. Inside the uterus d. Over the mother's lower abdomen

Answer: Over the uterine fundus Rationale: The tocotransducer monitors uterine activity and should be placed over the fundus, where the most intensive uterine contractions occur. The tocotransducer is for external use.

During a health history interview, a woman states that she thinks that she has "bumps" on her labia. She also states that she is not sure how to check herself. The correct response by the nurse would be what? a. Reassure the woman that the examination will reveal any problems. b. Explain the process of vulvar self-examination and reassure the woman that she should become familiar with normal and abnormal findings during the examination. c. Reassure the woman that "bumps" can be treated. d. Reassure her that most women have "bumps" on their labia.

Answer: Explain the process of vulvar self-examination and reassure the woman that she should become familiar with normal and abnormal findings during the examination. Rationale: During the assessment and evaluation, the responsibility for self-care, health promotion, and enhancement of wellness is emphasized. The pelvic examination provides a good opportunity for the practitioner to emphasize the need for regular vulvar self-examination. Providing reassurance to the woman concerning the "bumps" would not be an accurate response.

A 23-year-old African-American woman is pregnant with her first child. Based on current statistics for infant mortality, which intervention is most important for the nurse to include in the client's plan of care? a. Perform a nutrition assessment. b. Refer the woman to a social worker. c. Advise the woman to see an obstetrician, not a midwife. d. Explain to the woman the importance of keeping her prenatal care appointments.

Answer: Explain to the woman the importance of keeping her prenatal care appointments. Rationale: Consistent prenatal care is the best method of preventing or controlling risk factors associated with infant mortality. Nutritional status is an important modifiable risk factor, but it is not the most important action a nurse should take in this situation. The client may need assistance from a social worker at some time during her pregnancy, but a referral to a social worker is not the most important aspect the nurse should address at this time. If the woman has identifiable high-risk problems, then her health care may need to be provided by a physician. However, it cannot be assumed that all African-American women have high-risk issues. In addition, advising the woman to see an obstetrician is not the most important aspect on which the nurse should focus at this time, and it is not appropriate for a nurse to advise or manage the type of care a client is to receive.

The U.S. Department of Health and Human Services has designated Thanksgiving Day as National Family History Day. The U.S. Surgeon General encourages family members to discuss important family health information while sharing in holiday gatherings. Why is this initiative significant to nurses? a. Few genetic tests are available that identify this information. b. Only physicians should obtain this detailed information. c. Clients cannot accurately complete these histories on their own. d. Family history is the single most cost-effective source for genetic information.

Answer: Family history is the single most cost-effective source for genetic information Rationale: Although more than 1000 genetic tests are available, the single most cost-effective piece of genetic information is the family history. Nurses are ideally suited to take the lead in ongoing efforts to recognize the significance of the family history as an important source of genetic information. A computerized tool called My Family Health Portrait is available free of charge (https://familyhistory.hhs.gov/fhh-web/home.action). Other tools designed to help the lay community in completing their family histories are available to the public.

Which statement best exemplifies contemporary maternity nursing? a. Use of midwives for all vaginal deliveries b. Family-centered care c. Free-standing birth clinics d. Physician-driven care

Answer: Family-centered care Rationale: Contemporary maternity nursing focuses on the family's needs and desires. Fathers, partners, grandparents, and siblings may be present for the birth and participate in activities such as cutting the baby's umbilical cord. Both midwives and physicians perform vaginal deliveries. Free-standing clinics are an example of alternative birth options. Contemporary maternity nursing is driven by the relationship between nurses and their clients.

A woman who is gravida 3 para 2 arrives on the intrapartum unit. What is the most important nursing assessment initially? a. Contraction pattern, amount of discomfort, and pregnancy history b. Fetal heart rate, maternal vital signs, and the woman's nearness to birth c. Identification of ruptured membranes, woman's gravida and para, and her support person d. Last food intake, when labor began, and cultural practices the couple desires

Answer: Fetal heart rate, maternal vital signs, and the woman's nearness to birth Rationale: All options describe relevant intrapartum nursing assessments; however, this focused assessment has a priority. If the maternal and fetal conditions are normal and birth is not imminent, then other assessments can be performed in an unhurried manner; these include: gravida, para, support person, pregnancy history, pain assessment, last food intake, and cultural practices.

Certain changes stimulate chemoreceptors in the aorta and carotid bodies to prepare the fetus for initiating respirations immediately after birth. Which change in fetal physiologic activity is not part of this process? a. Fetal lung fluid is cleared from the air passages during labor and vaginal birth. b. Fetal partial pressure of oxygen (PO2) decreases. c. Fetal partial pressure of carbon dioxide in arterial blood (PaCO2) increases. d. Fetal respiratory movements increase during labor.

Answer: Fetal respiratory movements increase during labor. Rationale: Fetal respiratory movements decrease during labor. Fetal lung fluid is cleared from the air passages during labor and vaginal birth. Fetal PO2 decreases, and fetal PaCO2 increases.

Nurses who provide care to victims of intimate partner violence (IPV) should be keenly aware of what? a. Relationship violence usually consists of a single episode that the couple can put behind them. b. Violence often declines or ends with pregnancy. c. Financial coercion is considered part of IPV. d. Battered women are generally poorly educated and come from a deprived social background.

Answer: Financial coercion is considered part of IPV. Rationale: Economic coercion may accompany physical assault and psychologic attacks. IPV almost always follows an escalating pattern. It rarely ends with a single episode of violence. IPV often begins with and escalates during pregnancy. It may include both psychologic attacks and economic coercion. Race, religion, social background, age, and education level are not significant factors in differentiating women at risk.

When the nurse is unsure how to perform a client care procedure that is high risk and low volume, his or her best action in this situation would be what? a. Ask another nurse. b. Discuss the procedure with the client's physician. c. Look up the procedure in a nursing textbook. d. First consult the agency procedure manual

Answer: First consult the agency procedure manual Rationale: Following the agency's policies and procedures manual is always best when seeking information on correct client procedures. These policies should reflect the current standards of care and the individual state's guidelines. Each nurse is responsible for his or her own practice. Relying on another nurse may not always be a safe practice. Each nurse is obligated to follow the standards of care for safe client care delivery. Physicians are responsible for their own client care activity. Nurses may follow safe orders from physicians, but they are also responsible for the activities that they, as nurses, are to carry out. Information provided in a nursing textbook is basic information for general knowledge. Furthermore, the information in a textbook may not reflect the current standard of care or the individual state or hospital policies.

The nurse is aware that the initiation of breastfeeding is most effective during the first 30 minutes after birth. What is the correct term for this phase of alertness? a. Transition period b. First period of reactivity c. Organizational stage d. Second period of reactivity

Answer: First period of reactivity Rationale: The first period of reactivity is the first phase of transition and lasts up to 30 minutes after birth. The infant is highly alert during this phase. The transition period is the phase between intrauterine and extrauterine existence. An organizational stage is not a valid stage. The second period of reactivity occurs approximately between 4 and 8 hours after birth, after a period of sleep.

Through a vaginal examination, the nurse determines that a woman is 4 cm dilated. The external fetal monitor shows uterine contractions every to 4 minutes. The nurse reports this as what stage of labor? a. First stage, latent phase b. First stage, active phase c. First stage, transition phase d. Second stage, latent phase

Answer: First stage, active phase Rationale: This maternal progress indicates that the woman is in the active phase of the first stage of labor. During the latent phase of the first stage of labor, the expected maternal progress is 0 to 3 cm dilation with contractions every 5 to 30 minutes. During the transition phase of the first stage of labor, the expected maternal progress is 8 to 10 cm dilation with contractions every 2 to 3 minutes. During the latent phase of the second stage of labor, the woman is completely dilated and experiences a restful period of "laboring down."

Which description of the four stages of labor is correct for both the definition and the duration? a. First stage: onset of regular uterine contractions to full dilation; less than 1 hour to 20 hours b. Second stage: full effacement to 4 to 5 cm; visible presenting part; 1 to 2 hours c. Third stage: active pushing to birth; 20 minutes (multiparous woman), 50 minutes (nulliparous woman) d. Fourth stage: delivery of the placenta to recovery; 30 minutes to 1 hour

Answer: First stage: onset of regular uterine contractions to full dilation; less than 1 hour to 20 hours Rationale: Full dilation may occur in less than 1 hour, but in first-time pregnancies full dilation can take up to 20 hours. The second stage of labor extends from full dilation to birth and takes an average of 20 to 50 minutes, although 2 hours is still considered normal. The third stage of labor extends from birth to the expulsion of the placenta and usually takes a few minutes. The fourth stage begins after the expulsion of the placenta and lasts until homeostasis is reestablished (approximately 2 hours).

Which collection of risk factors will most likely result in damaging lacerations, including episiotomies? a. Dark-skinned woman who has had more than one pregnancy, who is going through prolonged second-stage labor, and who is attended by a midwife b. Reddish-haired mother of two who is going through a breech birth c. Dark-skinned first-time mother who is going through a long labor d. First-time mother with reddish hair whose rapid labor was overseen by an obstetrician

Answer: First-time mother with reddish hair whose rapid labor was overseen by an obstetrician Rationale: Reddish-haired women have tissue that is less distensible than darker-skinned women and therefore may have less efficient healing. First-time mothers are also at greater risk, especially with breech births, long second-stage labors, or rapid labors during which the time for the perineum to stretch is insufficient. The rate of episiotomies is higher when obstetricians rather than midwives attend the births. The woman in the first scenario (a) is at low risk for either damaging lacerations or an episiotomy. She is multiparous, has dark skin, and is being attended by a midwife, who is less likely to perform an episiotomy. Reddish-haired women have tissue that is less distensible than that of darker-skinned women. Consequently, the client in the second scenario (b) is at increased risk for lacerations; however, she has had two previous deliveries, which result in a lower likelihood of an episiotomy. The fact that the woman in the third scenario (c) is experiencing a prolonged labor might increase her risk for lacerations. Fortunately, she is dark skinned, which indicates that her tissue is more distensible than that of fair-skinned women and therefore less susceptible to injury.

Which statement, related to the reconditioning of the urinary system after childbirth, should the nurse understand? a. Kidney function returns to normal a few days after birth. b. Diastasis recti abdominis is a common condition that alters the voiding reflex. c. Fluid loss through perspiration and increased urinary output accounts for a weight loss of more than 2 kg during the puerperium. d. With adequate emptying of the bladder, bladder tone is usually restored 2 to 3 weeks after childbirth.

Answer: Fluid loss through perspiration and increased urinary output accounts for a weight loss of more than 2 kg during the puerperium. Rationale: Excess fluid loss through other means besides perspiration and increased urinary output occurs as well. Kidney function usually returns to normal in approximately 1 month. Diastasis recti abdominis is the separation of muscles in the abdominal wall and has no effect on the voiding reflex. Bladder tone is usually restored 5 to 7 days after childbirth.

The first hour after birth is sometimes referred to as what? a. Bonding period b. Third stage of labor c. Fourth stage of labor d. Early postpartum period

Answer: Fourth stage of labor Rationale: The fourth stage of labor begins with the expulsion of the placenta and lasts until the woman is stable in the immediate postpartum period, usually within the first hour after birth. Maternal organs undergo their initial readjustment to a nonpregnant state. The third stage of labor lasts from the birth of the baby to the expulsion of the placenta. Bonding will occur over a much longer period, although it may be initiated during the fourth stage of labor.

A woman's position is an important component of the labor progress. Which guidance is important for the nurse to provide to the laboring client? a. The supine position, which is commonly used in the United States, increases blood flow. b. The laboring client positioned on her hands and knees ("all fours" position) is hard on the woman's back. c. Frequent changes in position help relieve fatigue and increase the comfort of the laboring client. d. In a sitting or squatting position, abdominal muscles of the laboring client will have to work harder.

Answer: Frequent changes in position help relieve fatigue and increase the comfort of the laboring client. Rationale: Frequent position changes relieve fatigue, increase comfort, and improve circulation. Blood flow can be compromised in the supine position; any upright position benefits cardiac output. The "all fours" position is used to relieve backache in certain situations. In a sitting or squatting position, the abdominal muscles work in greater harmony with uterine contractions.

The nurse should be cognizant of which statement regarding the unique qualities of human breast milk? a. Frequent feedings during predictable growth spurts stimulate increased milk production. b. Milk of preterm mothers is the same as the milk of mothers who gave birth at term. c. Milk at the beginning of the feeding is the same as the milk at the end of the feeding. d. Colostrum is an early, less concentrated, less rich version of mature milk.

Answer: Frequent feedings during predictable growth spurts stimulate increased milk production. Rationale: Growth spurts (at 10 days, 3 weeks, 6 weeks, and 3 months) usually last 24 to 48 hours, after which the infants resume normal feeding. The milk of mothers of preterm infants is different from that of mothers of full-term infants to meet the needs of these newborns. Milk changes composition during feeding. The fat content of the milk increases as the infant feeds. Colostrum precedes mature milk and is more concentrated and richer in proteins and minerals (but not fat)

Which phase does not belong in Lenore Walker's three-cycle pattern of violence? a. Tension-building state b. Frustration, followed by violence c. Acute battering incident d. Kindness and contrite, loving behavior

Answer: Frustration, followed by violence Rationale: Frustration, followed by violence, is not part of the cycle of violence. The tension-building state is also known as phase I of the cycle. The batterer expresses dissatisfaction and hostility with violent outbursts. The woman senses anger and anxiously tries to placate him. An acute battering incident is phase II of the cycle. It results in the man's uncontrollable discharge of tension toward the woman. Outbursts can last from several hours to several days and may involve kicking, punching, slapping, choking, burns, broken bones, and the use of weapons. Phase III of the cycle is sometimes referred to as the honeymoon, kindness and contrite, and loving behavior phase, during which the batterer feels remorseful and profusely apologizes. He tries to help the woman and often showers her with gifts.

Which documentation on a woman's chart on postpartum day 14 indicates a normal involution process? a. Moderate bright red lochial flow b. Breasts firm and tender c. Fundus below the symphysis and nonpalpable d. Episiotomy slightly red and puffy

Answer: Fundus below the symphysis and nonpalpable Rationale: The fundus descends 1 cm per day; consequently, it is no longer palpable by postpartum day 14. The lochia should be changed by this day to serosa. Breasts are not part of the involution process. The episiotomy should not be red or puffy at this stage.

Which component of the physical examination are Leopold's maneuvers unable to determine? a. Gender of the fetus b. Presenting fetal part c. Fetal lie d. Estimated fetal size

Answer: Gender of the fetus Rationale: Leopold's maneuvers help identify the size of the fetal lie and attitude, and the degree of descent of the presenting part into the pelvis. The gender of the fetus cannot be determined by performing Leopold's maneuvers.

Nurses who elect to practice in the field of obstetrics must have a basic working knowledge of genetics. What is the correct term used to describe an individual's genetic makeup? a. Genotype b. Phenotype c. Karyotype d. Chromotype

Answer: Genotype Rationale: The genotype comprises all the genes the individual can pass on to a future generation. The phenotype is the observable expression of an individual's genotype. The karyotype is a pictorial analysis of the number, form, and size of an individual's chromosomes. Genotype refers to an individual's genetic makeup.

A postpartum woman telephones the provider regarding her 5-day-old infant. The client is not scheduled for another weight check until the infant is 14 days old. The new mother is worried about whether breastfeeding is going well. Which statement indicates that breastfeeding is effective for meeting the infant's nutritional needs? a. Sleeps for 6 hours at a time between feedings b. Has at least one breast milk stool every 24 hours c. Gains 1 to 2 ounces per week d. Has 3 bowel movements and eight wet diapers per day

Answer: Has 3 bowel movements and eight wet diapers per day Rationale: After day 4, when the mother's milk comes in, the infant should have six to eight wet diapers every 24 hours. Typically, infants sleep 2 to 4 hours between feedings, depending on whether they are being fed on a 2- to 3-hour schedule or cluster-fed. The infant's sleep pattern is not an indication whether the infant is breastfeeding well. The infant should have a minimum of three bowel movements in a 24-hour period. Breastfed infants typically gain 15 to 30 g/day.

Which condition, not uncommon in pregnancy, is likely to require careful medical assessment during the puerperium? a. Varicosities of the legs b. Carpal tunnel syndrome c. Periodic numbness and tingling of the fingers d. Headaches ANS: D Headaches are common in the first postpartum week; they are usually bilateral and frontal in location. Headaches in the postpartum period can have a number of causes, some of which deserve medical attention. Total or nearly total regression of varicosities is expected after childbirth. Carpal tunnel syndrome is relieved in childbirth when the compression on the median nerve is lessened. Periodic numbness of the fingers usually disappears after childbirth unless carrying the baby aggravates the condition.

Answer: Headaches

What is the rationale for the use of a blood patch after spinal anesthesia? a. Preventing related hypotension b. Minimizing the risk of a spinal headache c. Eliminating neonatal respiratory depression d. Limiting the loss of movement

Answer: Minimizing the risk of a spinal headache Rationale: The subarachnoid block may cause a post-spinal headache resulting from the loss of cerebrospinal fluid from the puncture in the dura. When blood is injected into the epidural space around the dural puncture, it forms a seal over the hole to stop the leaking of cerebrospinal fluid. Hypotension is prevented by increasing fluid volume before the procedure. Neonatal respiratory depression is not an expected outcome with spinal anesthesia. Loss of movement is an expected outcome of spinal anesthesia.

A woman in the active phase of the first stage of labor is using a shallow pattern of breathing, which is approximately twice the normal adult breathing rate. She starts to report feeling lightheaded and dizzy and states that her fingers are tingling. Which intervention should the nurse immediately initiate? a. Contact the woman's health care provider. b. Tell the woman to slow her pace of her breathing. c. Administer oxygen via a mask or nasal cannula. d. Help her breathe into a paper bag.

Answer: Help her breathe into a paper bag. Rationale: This woman is experiencing the side effects of hyperventilation, which include the symptoms of lightheadedness, dizziness, tingling of the fingers, or circumoral numbness. Having the woman breathe into a paper bag held tightly around her mouth and nose may eliminate respiratory alkalosis and enable her to rebreathe carbon dioxide and replace the bicarbonate ion. It is unlikely the woman will be able to follow instructions to slow her breathing since there is a physiological cause for the hyperventilation. The woman is in need of carbon dioxide not oxygen. The health care provider is notified only if all appropriate interventions fail to help normalize the client's breathing.

What is the primary reason why a woman who is older than 35 years may have difficulty achieving pregnancy? a. Personal risk behaviors influence fertility. b. Mature women have often used contraceptives for an extended time. c. Her ovaries may be affected by the aging process. d. Pre-pregnancy medical attention is lacking.

Answer: Her ovaries may be affected by the aging process. Rationale: Once the mature woman decides to conceive, a delay in becoming pregnant may occur because of the normal aging of the ovaries. Older adults participate in fewer risk behaviors than younger adults. The past use of contraceptives is not the problem. Pre-pregnancy medical care is both available and encouraged and has limited effect on initiating pregnancy.

A recently graduated nurse is attempting to understand the reason for increasing health care spending in the United States. Which information gathered from research best explains the rationale for these higher costs compared with other developed countries? a. Higher rate of obesity among pregnant women b. Limited access to technology c. Increased use of health care services along with lower prices d. Homogeneity of the population

Answer: Higher rate of obesity among pregnant women Rationale: Health care is one of the fastest growing sectors of the U.S. economy. Currently, 17.5% of the gross domestic product is spent on health care. Higher spending in the United States, as compared with 12 other industrialized countries, is related to higher prices and readily accessible technology along with greater obesity rates among women. More than one third of women in the United States are obese. In the population in the United States, 16% are uninsured and have limited access to health care. Maternal morbidity and mortality are directly related to racial disparities.

While examining a newborn, the nurse notes uneven skinfolds on the buttocks and a clunk when performing the Ortolani maneuver. These findings are likely indicative of what? a. Polydactyly b. Clubfoot c. Hip dysplasia d. Webbing

Answer: Hip dysplasia Answer: The Ortolani maneuver is used to detect the presence of hip dysplasia. Polydactyly is the presence of extra digits. Clubfoot (talipes equinovarus) is a deformity in which the foot turns inward and is fixed in a plantar-flexion position. Webbing, or syndactyly, is a fusing of the fingers or toes.

Nurses with an understanding of cultural differences regarding likely reactions to pain may be better able to help their clients. Which clients may initially appear very stoic but then become quite vocal as labor progresses until late in labor, when they become more vocal and request pain relief? a. Chinese b. Arab or Middle Eastern c. Hispanic d. African-American

Answer: Hispanic Rationale: Hispanic women may be stoic early in labor but more vocal and readier for medications later. Chinese women may not show reactions to pain. Medical interventions must be offered more than once. Arab or Middle Eastern women may be vocal in response to labor pain from the start; they may prefer pain medications. African-American women may openly express pain; the use of medications for pain is more likely to vary with the individual.

When weighing the advantages and disadvantages of planning home care for perinatal services, what information should the nurse use in making the decision? a. Home care for perinatal services is more dangerous for vulnerable neonates at risk of acquiring an infection from the nurse. b. Home care for perinatal services is more cost-effective for the nurse than office visits. c. Home care for perinatal services allows the nurse to interact with and include family members in teaching. d. Home care for perinatal services is made possible by the ready supply of nurses with expertise in maternity care.

Answer: Home care for perinatal services allows the nurse to interact with and include family members in teaching. Rationale: Treating the whole family is an advantage of home care. Forcing neonates out in inclement weather and in public is more risky. Office visits are more cost-effective for the providers such as nurses because less travel time is involved. Unfortunately, home care options are limited by the lack of nurses with expertise in maternity care.

What is the primary difference between hospital care and home health care? a. Home care is routinely and continuously delivered by professional staff. b. Home care is delivered on an intermittent basis by professional staff. c. Home care is delivered for emergency conditions. d. Home care is not available 24 hours a day.

Answer: Home care is delivered on an intermittent basis by professional staff Rationale: Home care is generally delivered on an intermittent basis by professional staff members. The primary difference between health care in a hospital and home care is the absence of the continuous presence of professional health care providers in a client's home. In a true emergency, the client should be directed to call 9-1-1 or to report to the nearest hospital's emergency department. Generally, home health care entails intermittent care by a professional who visits the client's home for a particular reason and provides on-site care for periods shorter than 4 hours at a time.

What is a limitation of a home postpartum visit? a. Distractions limit the nurse's ability to teach. b. Identified problems cannot be resolved in the home setting. c. Necessary items for infant care are not available. d. Home visits to different families may require the nurse to travel a great distance.

Answer: Home visits to different families may require the nurse to travel a great distance. One limitation of home health visits is the distance the nurse must travel between clients. Driving directions should be obtained by telephone before the visit. The home care nurse is accustomed to distractions but may request that the television be turned off so that attention can be focused on the client and her family. Problems cannot always be resolved; however, appropriate referrals may be arranged by the nurse. The nurse is required to bring any necessary equipment, such as a thermometer, baby scale, or laptop computer, for documentation.

Nursing follow-up care often includes home visits for the new mother and her infant. Which information related to home visits is correct? a. Ideally, the visit is scheduled within 72 hours after discharge. b. Home visits are available in all areas. c. Visits are completed within a 30-minute time frame. d. Blood draws are not a part of the home visit.

Answer: Ideally, the visit is scheduled within 72 hours after discharge. Rationale: The home visit is ideally scheduled within 72 hours after discharge. This timing allows early assessment and intervention for problems with feedings, jaundice, newborn adaptation, and maternal-infant interaction. Because of geographic distances, home visits are not available in all locales. Visits are usually 60 to 90 minutes in length to allow enough time for assessment and teaching. When jaundice is found, the nurse can discuss the implications and check the transcutaneous bilirubin level or draw blood for testing.

As the nurse assists a new mother with breastfeeding, the client asks, "If formula is prepared to meet the nutritional needs of the newborn, what is in breast milk that makes it better?" What is the nurse's best response? a. More calories b. Essential amino acids c. Important immunoglobulins d. More calcium

Answer: Important immunoglobulins Rationale: Breast milk contains immunoglobulins that protect the newborn against infection. The calorie count of formula and breast milk is approximately the same. All the essential amino acids are in both formula and breast milk; however, the concentrations may differ. Calcium levels are higher in formula than in breast milk, which can cause an excessively high renal solute load if the formula is not properly diluted.

A new father states, "I know nothing about babies"; however, he seems to be interested in learning. How would the nurse best respond to this father? a. Continue to observe his interaction with the newborn. b. Tell him when he does something wrong. c. Show no concern; he will learn on his own. d. Include him in teaching sessions.

Answer: Include him in teaching sessions Rationale: The nurse must be sensitive to the father's needs and include him whenever possible. As fathers take on their new role, the nurse should praise every attempt, even if his early care is awkward. Although noting the bonding process of the mother and the father is important, it does not satisfy the expressed needs of the father. The new father should be encouraged to care for his baby by pointing out the things that he does right. Criticizing him will discourage him.

The nurse expects which maternal cardiovascular finding during labor? a. Increased cardiac output b. Decreased pulse rate c. Decreased white blood cell (WBC) count d. Decreased blood pressure

Answer: Increased cardiac output Rationale: During each contraction, 400 ml of blood is emptied from the uterus into the maternal vascular system, which increases cardiac output by approximately 10% to 15% during the first stage of labor and by approximately 30% to 50% in the second stage of labor. The heart rate increases slightly during labor. The WBC count can increase during labor. During the first stage of labor, uterine contractions cause systolic readings to increase by approximately 10 mm Hg. During the second stage, contractions may cause systolic pressures to increase by 30 mm Hg and diastolic readings to increase by 25 mm Hg.

Which cardiovascular changes cause the foramen ovale to close at birth? a. Increased pressure in the right atrium b. Increased pressure in the left atrium c. Decreased blood flow to the left ventricle d. Changes in the hepatic blood flow

Answer: Increased pressure in the left atrium Rationale: With the increase in the blood flow to the left atrium from the lungs, the pressure is increased, and the foramen ovale is functionally closed. The pressure in the right atrium decreases at birth and is higher during fetal life. Blood flow increases to the left ventricle after birth. The hepatic blood flow changes but is not the reason for the closure of the foramen ovale.

As part of the infant discharge instructions, the nurse is reviewing the use of the infant car safety seat. Which information is the highest priority for the nurse to share? a. Infant carriers are okay to use until an infant car safety seat can be purchased. b. For traveling on airplanes, buses, and trains, infant carriers are satisfactory. c. Infant car safety seats are used for infants only from birth to 15 pounds. d. Infant car seats should be rear facing and placed in the back seat of the car.

Answer: Infant car seats should be rear-facing and placed in the back seat of the car. Rationale: An infant placed in the front seat could be severely injured by an air bag that deploys during an automobile accident. Infants should travel only in federally approved, rear-facing safety seats secured in the rear seat and only in federally approved safety seats even when traveling on a commercial vehicle. Infants should use a rear-facing car seat from birth to 20 pounds and to age 1 year.

In addition to eye contact, other early sensual contacts between the infant and mother involve sound and smell. What other statement regarding the senses is correct? a. High-pitched voices irritate newborns. b. Infants can learn to distinguish their mother's voice from others soon after birth. c. All babies in the hospital smell alike. d. Mother's breast milk has no distinctive odor.

Answer: Infants can learn to distinguish their mother's voice from others soon after birth. Rationale: Infants know the sound of their mother's voice at an early age. Infants positively respond to high-pitched voices. Each infant has a unique odor. Infants quickly learn to distinguish the odor of their mother's breast milk.

A new mother asks the nurse what the "experts say" about the best way to feed her infant. Which recommendation of the American Academy of Pediatrics (AAP) regarding infant nutrition should be shared with this client? a. Infants should be given only human milk for the first 6 months of life. b. Infants fed on formula should be started on solid food sooner than breastfed infants. c. If infants are weaned from breast milk before 12 months, then they should receive cow's milk, not formula. d. After 6 months, mothers should shift from breast milk to cow's milk.

Answer: Infants fed on formula should be started on solid food sooner than breastfed infants. Rationale: Breastfeeding and human milk should also be the sole source of milk for the first 12 months, not for only the first 6 months. Infants should be started on solids when they are ready, usually at 6 months, whether they start on formula or breast milk. If infants are weaned from breast milk before 12 months, then they should receive iron-fortified formula, not cow's milk.

According to professional standards which action cannot be performed by the non-anesthetist registered nurse who is caring for a woman with epidural anesthesia? a. Monitoring the status of the woman and fetus b. Initiating epidural anesthesia c. Replacing empty infusion bags with the same medication and concentrate d. Stopping the infusion, and initiating emergency measures

Answer: Initiating epidural anesthesia Rationale: Only qualified, licensed anesthesia care providers are permitted to insert a catheter, initiate epidural anesthesia, verify catheter placement, inject medication through the catheter, or alter the medication or medications including type, amount, or rate of infusion. The non-anesthetist nurse is permitted to monitor the status of the woman, the fetus, and the progress of labor. Replacement of the empty infusion bags or syringes with the same medication and concentration is permitted. If the need arises, the nurse may stop the infusion, initiate emergency measures, and remove the catheter if properly educated to do so. Complications can require immediate interventions. Nurses must be prepared to provide safe and effective care during an emergency situation.

What is the most critical physiologic change required of the newborn after birth? a. Closure of fetal shunts in the circulatory system b. Full function of the immune defense system c. Maintenance of a stable temperature d. Initiation and maintenance of respirations

Answer: Initiation and maintenance of respirations Rationale: The most critical adjustment of a newborn at birth is the establishment of respirations. The cardiovascular system changes significantly after birth as a result of fetal respirations, which reduce pulmonary vascular resistance to the pulmonary blood flow and initiate a chain of cardiac changes that support the cardiovascular system. After the establishment of respirations, heat regulation is critical to newborn survival. The infant relies on passive immunity received from the mother for the first 3 months of life.

Because a full bladder prevents the uterus from contracting normally, nurses intervene to help the woman spontaneously empty her bladder as soon as possible. If all else fails, what tactic might the nurse use? a. Pouring water from a squeeze bottle over the woman's perineum b. Placing oil of peppermint in a bedpan under the woman c. Asking the healthcare provider to prescribe analgesic agents d. Inserting a sterile catheter

Answer: Inserting a sterile catheter Rationale: Invasive procedures are usually the last to be tried, especially with so many other simple and easy methods available (e.g., water, peppermint vapors, pain pills). Pouring water over the perineum may stimulate voiding. It is easy, noninvasive, and should be tried first. The oil of peppermint releases vapors that may relax the necessary muscles. It, too, is easy, noninvasive, and should be tried early on. If the woman is anticipating pain from voiding, then pain medications may be helpful. Other nonmedical means should be tried first, but medications still come before the insertion of a catheter.

Which condition is likely the biggest risk for the pregnant client? a. Preeclampsia b. Intimate partner violence (IPV) c. Diabetes d. Abnormal Pap test

Answer: Intimate partner violence (IPV) Rationale: The prevalence of IPV during pregnancy is estimated at 6% of all pregnant women. The risk for IPV and even IPV-related homicide is more common than all of the other pregnancy-related conditions. Although preeclampsia poses a risk to the health of the pregnant client, it is less common than IPV. Gestational diabetes continues to be a complication of pregnancy; however, it is less common than IPV during pregnancy. Some women are at risk for an abnormal Pap screening during pregnancy, but this finding is not as common as IPV.

The nurse is caring for a client in early labor. Membranes ruptured approximately 2 hours earlier. This client is at increased risk for which complication? a. Intrauterine infection b. Hemorrhage c. Precipitous labor d. Supine hypotension

Answer: Intrauterine infection Rationale: When the membranes rupture (ROM), microorganisms from the vagina can ascend into the amniotic sac, causing chorioamnionitis and placentitis. ROM is not associated with fetal or maternal bleeding. Although ROM may increase the intensity of the contractions and facilitate active labor, it does not result in precipitous labor. ROM has no correlation with supine hypotension.

Which adaptation of the maternal-fetal exchange of oxygen occurs in response to uterine contraction? a. The maternal-fetal exchange of oxygen and waste products continues except when placental functions are reduced. b. This maternal-fetal exchange increases as the blood pressure decreases. c. It diminishes as the spiral arterioles are compressed. d. This exchange of oxygen and waste products is not significantly affected by contractions.

Answer: It diminishes as the spiral arterioles are compressed. Rationale: Uterine contractions during labor tend to decrease circulation through the spiral arterioles and subsequent perfusion through the intervillous space. The maternal blood supply to the placenta gradually stops with contractions. The exchange of oxygen and waste products decreases. The exchange of oxygen and waste products is affected by contractions.

The nurse is performing a gestational age and physical assessment on the newborn. The infant appears to have an excessive amount of saliva. This clinical finding may be indicative of what? a. Excessive saliva is a normal finding in the newborn. b. Excessive saliva in a neonate indicates that the infant is hungry. c. It may indicate that the infant has a tracheoesophageal fistula or esophageal atresia. d. Excessive saliva may indicate that the infant has a diaphragmatic hernia.

Answer: It may indicate that the infant has a tracheoesophageal fistula or esophageal atresia. Rationale: The presence of excessive saliva in a neonate should alert the nurse to the possibility of a tracheoesophageal fistula or esophageal atresia. Excessive salivation may not be a normal finding and should be further assessed for the possibility that the infant has an esophageal abnormality. The hungry infant reacts by making sucking motions, rooting, or making hand-to-mouth movements. The infant with a diaphragmatic hernia exhibits severe respiratory distress.

The nurse is providing instruction to the newly delivered client regarding postbirth uterine and vaginal discharge, called lochia. Which statement is the most appropriate? a. Lochia is similar to a light menstrual period for the first 6 to 12 hours. b. It is usually greater after cesarean births. c. Lochia will usually decrease with ambulation and breastfeeding. d. It should smell like normal menstrual flow unless an infection is present.

Answer: It should smell like normal menstrual flow unless an infection is present. Rationale: An offensive odor usually indicates an infection. Lochia flow should approximate a heavy menstrual period for the first 2 hours and then steadily decrease. Less lochia is usually seen after cesarean births and usually increases with ambulation and breastfeeding.

What is the most critical nursing action in caring for the newborn immediately after the birth? a. Keeping the airway clear b. Fostering parent-newborn attachment c. Drying the newborn and wrapping the infant in a blanket d. Administering eye drops and vitamin K

Answer: Keeping the airway clear Rationale: The care given immediately after the birth focuses on assessing and stabilizing the newborn. Although fostering parent-newborn attachment is an important task for the nurse, it is not the most critical nursing action in caring for the newborn immediately after birth. The care given immediately after birth focuses on assessing and stabilizing the newborn. The nursing activities are (in order of importance) to maintain a patent airway, to support respiratory effort, and to prevent cold stress by drying the newborn and covering him or her with a warmed blanket or placing the newborn under a radiant warmer. After the newborn has been stabilized, the nurse assesses the newborn's physical condition, weighs and measures the newborn, administers prophylactic eye ointment and a vitamin K injection, affixes an identification bracelet, wraps the newborn in warm blankets, and then gives the newborn to the partner or to the mother of the infant.

Which statement best describes Kegel exercises? a. Kegel exercises were developed to control or reduce incontinent urine loss. b. Kegel exercises produce a pleasurable vaginal sensation. c. Kegel exercises help manage stress. d. Kegel exercises are ineffective without sufficient calcium in the diet.

Answer: Kegel exercises were developed to control or reduce incontinent urine loss. Rationale: Kegel exercises help control the urge to urinate. Although these exercises may be pleasurable for some, the most important factor is the control they provide over incontinence. Kegel exercises help manage urination, not stress. Calcium in the diet is important but not related to Kegel exercises.

A nurse is assessing the knowledge of new parents of a child born with Klinefelter syndrome. Which statement accurately describes this genetic disorder? a. Klinefelter syndrome is a sex chromosome abnormality. b. It affects only female children. c. The disorder is expressed as trisomy XYY. d. The child with this disorder will grow to be infertile.

Answer: Klinefelter syndrome is a sex chromosome abnormality. Rationale: Klinefelter syndrome, also known as trisomy XXY, is a sex chromosomal deviation that is expressed in males. Turner syndrome (monosomy X) is displayed in females. Most males with Klinefelter syndrome are tall, may be infertile, and are slow to learn; however, those who have mosaic Klinefelter syndrome may be fertile as adults.

A nurse is assessing a newborn girl who is 2 hours old. Which finding warrants a call to the health care provider? a. Blood glucose of 45 mg/dl using a Dextrostix screening method b. Heart rate of 160 beats per minute after vigorously crying c. Laceration of the cheek d. Passage of a dark black-green substance from the rectum

Answer: Laceration of the cheek Rationale: Accidental lacerations can be inflicted by a scalpel during a cesarean birth. They are most often found on the scalp or buttocks and may require an adhesive strip for closure. Parents would be overly concerned about a laceration on the cheek. A blood glucose level of 45 mg/dl and a heart rate of 160 beats per minute after crying are both normal findings that do not warrant a call to the physician. The passage of meconium from the rectum is an expected finding in the newborn.

The most serious complication of an infant heelstick is necrotizing osteochondritis resulting from lancet penetration of the bone. What approach should the nurse take when performing the test to prevent this complication? a. Lancet should penetrate at the outer aspect of the heel. b. Lancet should penetrate the walking surface of the heel. c. Lancet should penetrate the ball of the foot. d. Lancet should penetrate the area just below the fifth toe.

Answer: Lancet should penetrate at the outer aspect of the heel. Rationale: The stick should be made at the outer aspect of the heel and should penetrate no deeper than 2.4 mm. Repeated trauma to the walking surface of the heel can cause fibrosis and scarring that can lead to problems with walking later in life. The ball of the foot and the area below the fifth toe are inappropriate sites for a heelstick.

Which statement best describes the transition period between intrauterine and extrauterine existence for the newborn? a. Consists of four phases, two reactive and two of decreased responses b. Lasts from birth to day 28 of life c. Applies to full-term births only d. Varies by socioeconomic status and the mother's age

Answer: Lasts from birth to day 28 of life Rationale: Changes begin immediately after birth; the cutoff time when the transition is considered over (although the baby keeps changing) is 28 days. This transition period has three phases: first reactivity, decreased response, and second reactivity. All newborns experience this transition period, regardless of age or type of birth. Although stress can cause variations in the phases, the mother's age and wealth do not disturb the pattern.

Which fetal heart rate (FHR) finding is the most concerning to the nurse who is providing care to a laboring client? a. Accelerations with fetal movement b. Early decelerations c. Average FHR of 126 beats per minute d. Late decelerations

Answer: Late decelerations Rationale: Late decelerations are caused by uteroplacental insufficiency and are associated with fetal hypoxemia. Late FHR decelerations are considered ominous if they are persistent and left uncorrected. Accelerations with fetal movement are an indication of fetal well-being. Early decelerations in the FHR are associated with head compression as the fetus descends into the maternal pelvic outlet; they are not generally a concern during normal labor. An FHR finding of 126 beats per minute is normal and not a concern.

Which characteristic correctly matches the type of deceleration with its likely cause? a. Early deceleration—umbilical cord compression b. Late deceleration—uteroplacental insufficiency c. Variable deceleration—head compression d. Prolonged deceleration—unknown cause

Answer: Late deceleration—uteroplacental insufficiency Rationale: Late deceleration is caused by uteroplacental insufficiency. Early deceleration is caused by head compression. Variable deceleration is caused by umbilical cord compression. Prolonged deceleration has a variety of either benign or critical causes.

What is the correct term describing the slight overlapping of cranial bones or shaping of the fetal head during labor? a. Lightening b. Molding c. Ferguson reflex d. Valsalva maneuver

Answer: Molding Rationale: Molding also permits adaptation to various diameters of the maternal pelvis. Lightening is the mother's sensation of decreased abdominal distention, which usually occurs the week before labor. The Ferguson reflex is the contraction urge of the uterus after the stimulation of the cervix. The Valsalva maneuver describes conscious pushing during the second stage of labor.

The trend in the United States is for women to remain hospitalized no longer than 1 or 2 days after giving birth. Which scenario is not a contributor to this model of care? a. Wellness orientation model of care rather than a sick-care model b. Desire to reduce health care costs c. Consumer demand for fewer medical interventions and more family-focused experiences d. Less need for nursing time as a result of more medical and technologic advances and devices available at home that can provide information

Answer: Less need for nursing time as a result of more medical and technologic advances and devices available at home that can provide information. Rationale: Nursing time and care are in demand as much as ever; the nurse simply has to do things more quickly. A wellness orientation model of care seems to focus on getting clients out the door sooner. In most cases, less hospitalization results in lower costs. People believe that the family gives more nurturing care than the institution.

Which statement correctly describes the effects of various pain factors? a. Higher prostaglandin levels arising from dysmenorrhea can blunt the pain of childbirth. b. Upright positions in labor increase the pain factor because they cause greater fatigue. c. Women who move around trying different positions experience more pain. d. Levels of pain-mitigating beta-endorphins are higher during a spontaneous, natural childbirth.

Answer: Levels of pain-mitigating beta-endorphins are higher during a spontaneous, natural childbirth. Rationale: Higher endorphin levels help women tolerate pain and reduce anxiety and irritability. Higher prostaglandin levels correspond to more severe labor pains. Upright positions in labor usually result in improved comfort and less pain. Moving freely to find more comfortable positions is important for reducing pain and muscle tension.

A woman who has a history of sexual abuse may have several traumatic memories triggered during labor. She may fight the labor process and react with pain or anger. The nurse can implement appropriate care measures to help her client view the childbirth experience in a positive manner. Which intervention is key for the nurse to use while providing care? a. Tell the client to relax and that it won't hurt much. b. Limit the number of procedures that invade her body. c. Reassure the client that, as the nurse, you know what is best. d. Allow unlimited care providers to be with the client.

Answer: Limit the number of procedures that invade her body. Rationale: The number of invasive procedures such as vaginal examinations, internal monitoring, and IV therapy should be limited as much as possible. The nurse should always avoid words and phrases that may result in the client's recalling the phrases of her abuser (i.e., "Relax, this won't hurt" or "Just open your legs"). The woman's sense of control should always be maintained . The nurse should explain procedures at the client's pace and wait for permission to proceed. Protecting the client's environment by providing privacy and limiting the number of staff who observe the client will help to make her feel safe.

Intervention for the sexual abuse survivor is often not attempted by maternity and women's health nurses because of the concern about increasing the distress of the woman and the lack of expertise in counseling. What initial intervention is appropriate and most important in facilitating the woman's care? a. Initiating a referral to an expert counselor b. Setting limits on what the client discloses c. Listening and encouraging therapeutic communication skills d. Acknowledging the nurse's discomfort to the client as an expression of empathy

Answer: Listening and encouraging therapeutic communication skills Rationale: The survivor needs support on many different levels, and a women's health nurse may be the first person to whom she relates her story. Therapeutic communication skills and listening are initial interventions. Referring this client to a counselor is an appropriate measure but not the most important initial intervention. A client should be allowed to disclose any information she feels the need to discuss. A nurse should provide a safe environment in which she can do so. Either verbal or nonverbal shock and horror reactions from the nurse are particularly devastating. Professional demeanor and professional empathy are essential.

A woman gave birth to a healthy infant boy 5 days ago. What type of lochia does the nurse expect to find when evaluating this client? a. Lochia rubra b. Lochia sangra c. Lochia alba d. Lochia serosa

Answer: Lochia serosa Rationale: Lochia serosa, which consists of blood, serum, leukocytes, and tissue debris, generally occurs around day 4 after childbirth. Lochia rubra consists of blood and decidual and trophoblastic debris. The flow generally lasts 3 to 4 days and pales, becoming pink or brown. Lochia sangra is not a real term. Lochia alba occurs in most women after day 10 and can continue up to 6 weeks after childbirth.

Two days ago a woman gave birth to a full-term infant. Last night she awakened several times to urinate and noted that her gown and bedding were wet from profuse diaphoresis. Which physiologic alteration is the cause for the diaphoresis and diuresis that this client is experiencing? a. Elevated temperature caused by postpartum infection b. Increased basal metabolic rate after giving birth c. Loss of increased blood volume associated with pregnancy d. Increased venous pressure in the lower extremities

Answer: Loss of increased blood volume associated with pregnancy Rationale: Within 12 hours of birth, women begin to lose the excess tissue fluid that has accumulated during pregnancy. One mechanism for reducing these retained fluids is the profuse diaphoresis that often occurs, especially at night, for the first 2 or 3 days after childbirth. Postpartal diuresis is another mechanism by which the body rids itself of excess fluid. An elevated temperature causes chills and possibly dehydration, not diaphoresis and diuresis. Diaphoresis and diuresis are sometimes referred to as reversal of the water metabolism of pregnancy, not as the basal metabolic rate. Postpartal diuresis may be caused by the removal of increased venous pressure in the lower extremities.

A new mother recalls from prenatal class that she should try to feed her newborn daughter when she exhibits feeding readiness cues rather than waiting until the baby is frantically crying. Which feeding cue would indicate that the baby is ready to eat? a. Waves her arms in the air b. Makes sucking motions c. Has the hiccups d. Stretches out her legs straight

Answer: Makes sucking motions Rationale: Sucking motions, rooting, mouthing, and hand-to-mouth motions are examples of feeding readiness cues. Waving her arms in the air, having the hiccups, and stretching out her extremities are not typical feeding readiness cues.

What is the primary theme of the feminist perspective regarding violence against women? a. Role of testosterone as the underlying cause of men's violent behavior b. Basic human instinctual drive toward aggression c. Male dominance and coercive control over women d. Cultural norm of violence in Western society

Answer: Male dominance and coercive control over women Rationale: The contemporary social view of violence is derived from the feminist theory. With the primary theme of male dominance and coercive control, this view enhances an understanding of all forms of violence against women, including wife battering, stranger and acquaintance rape, incest, and sexual harassment in the workplace. The role of testosterone as an underlying cause of men's violent behavior, the basic human instinctual drive toward aggression, and the cultural norm of violence in Western society are not associated with the feminist perspective regarding violence against women.

Nursing care measures are commonly offered to women in labor. Which nursing measure reflects the application of the gate-control theory? a. Massaging the woman's back. b. Changing the woman's position. c. Giving the prescribed medication. d. Encouraging the woman to rest between contractions.

Answer: Massaging the woman's back. Rationale: According to the gate-control theory, pain sensations travel along sensory nerve pathways to the brain, but only a limited number of sensations, or messages, can travel through these nerve pathways at one time. Distraction techniques, such as massage or stroking, music, focal points, and imagery, reduce or completely block the capacity of the nerve pathways to transmit pain. These distractions are thought to work by closing down a hypothetic gate in the spinal cord, thus preventing pain signals from reaching the brain. The perception of pain is thereby diminished. Changing the woman's position, administering pain medication, and resting between contractions do not reduce or block the capacity of the nerve pathways to transmit pain using the gate-control theory.

On examining a woman who gave birth 5 hours ago, the nurse finds that the woman has completely saturated a perineal pad within 15 minutes. What is the nurse's highest priority? a. Beginning an intravenous (IV) infusion of Ringer's lactate solution b. Assessing the woman's vital signs c. Calling the woman's primary health care provider d. Massaging the woman's fundus

Answer: Massaging the woman's fundus Rationale: The nurse should first assess the uterus for atony by massaging the woman's fundus. Uterine tone must be established to prevent excessive blood loss. The nurse may begin an IV infusion to restore circulatory volume, but this would not be the first action. Blood pressure is not a reliable indicator of impending shock from impending hemorrhage; assessing vital signs should not be the nurse's first action. The healthcare provider would be notified after the nurse completes the intervention if the discharge is still excessive.

Which statement is the best rationale for assessing the maternal vital signs between uterine contractions? a. During a contraction, assessing the fetal heart rate is the priority. b. Maternal circulating blood volume temporarily increases during contractions. c. Maternal blood flow to the heart is reduced during contractions. d. Vital signs taken during contractions are not accurate. During uterine contractions, blood flow to the placenta temporarily stops, causing a relative increase in the mother's blood volume, which, in turn, temporarily increases blood pressure and slows the pulse. Monitoring fetal responses to the contractions is important; however, this question concerns the maternal vital signs. Maternal blood flow is increased during a contraction. Vital signs are altered by contractions but are considered accurate for that period.

Answer: Maternal circulating blood volume temporarily increases during contractions. Rationale: During uterine contractions, blood flow to the placenta temporarily stops, causing a relative increase in the mother's blood volume, which, in turn, temporarily increases blood pressure and slows the pulse. Monitoring fetal responses to the contractions is important; however, this question concerns the maternal vital signs. Maternal blood flow is increased during a contraction. Vital signs are altered by contractions but are considered accurate for that period.

Which statement best describes maternity nursing care that is based on knowledge gained through research and clinical trials? a. Maternity nursing care is derived from the Nursing Intervention Classification. b. Maternity nursing care is known as evidence-based practice. c. Maternity nursing care is at odds with the Cochrane School of traditional nursing. d. Maternity nursing care is an outgrowth of telemedicine.

Answer: Maternity nursing care is known as evidence-based practice Rationale: Evidence-based practice is based on knowledge gained from research and clinical trials. The Nursing Intervention Classification is a method of standardizing language and categorizing care. Dr. Cochrane systematically reviewed research trials and is part of the evidence-based practice movement. Telemedicine uses communication technologies to support health care.

The nurse observes a sudden increase in variability on the electronic fetal monitoring (EFM) tracing. Which class of medications may cause this finding? a. Narcotics b. Barbiturates c. Methamphetamines d. Tranquilizers

Answer: Methamphetamines Rationale: Narcotics, barbiturates, and tranquilizers may be causes of decreased variability; whereas methamphetamines may cause increased variability.

As the United States and Canada continue to become more culturally diverse, recognizing a wide range of varying cultural beliefs and practices is increasingly important for the nursing staff. A client is from which country if she requests to have the baby's father in attendance? a. Mexico b. China c. Iran d. India

Answer: Mexico Rationale: Hispanic women routinely have fathers and female relatives in attendance during the second stage of labor. The father of the baby is expected to provide encouragement, support, and reassurance that all will be well. In China, fathers are usually not present. The side-lying position is preferred for labor and birth because it is believed that this will reduce trauma to the infant. In China, the client has a stoic response to pain. In Iran, the father will not be present. Female support persons and female health care providers are preferred. For many, a male caregiver is unacceptable. In India, the father is usually not present, but female relatives are usually in attendance. Natural childbirth methods are preferred.

Which trait is least likely to be displayed by a woman experiencing intimate partner violence (IPV)? a. Social isolation b. Assertive personality c. Reoccurring depression d. Dependent personality

Answer: assertive personality Rationale: Every segment of society is represented among women who are suffering abuse. However, traits of assertiveness, independence, and willingness to take a stand have been documented as more characteristic of women who are in nonviolent relationships. Women who are financially more dependent have fewer resources and support systems, exhibit symptoms of depression, and are more often seen as victims.

An African-American woman noticed some bruises on her newborn daughter's buttocks. The client asks the nurse what causes these. How would the nurse best explain this integumentary finding to the client? a. Lanugo b. Vascular nevus c. Nevus flammeus d. Mongolian spot

Answer: Mongolian spot Rationale: A Mongolian spot is a bluish-black area of pigmentation that may appear over any part of the exterior surface of the infant's body and is more commonly noted on the back and buttocks and most frequently observed on infants whose ethnic origins are Mediterranean, Latin American, Asian, or African. Lanugo is the fine, downy hair observed on a term newborn. A vascular nevus, commonly called a strawberry mark, is a type of capillary hemangioma. A nevus flammeus, commonly called a port wine stain, is most frequently found on the face.

A woman in labor has just received an epidural block. What is the most important nursing intervention at this time? a. Limit parenteral fluids. b. Monitor the fetus for possible tachycardia. c. Monitor the maternal blood pressure for possible hypotension. d. Monitor the maternal pulse for possible bradycardia.

Answer: Monitor the maternal blood pressure for possible hypotension. Rationale: The most important nursing intervention for a woman who has received an epidural block is for the nurse to monitor the maternal blood pressure frequently for signs of hypotension. IV fluids are increased for a woman receiving an epidural to prevent hypotension. The nurse also observes for signs of fetal bradycardia and monitors for signs of maternal tachycardia, secondary to hypotension.

While evaluating the reflexes of a newborn, the nurse notes that with a loud noise the newborn symmetrically abducts and extends his arms, his fingers fan out and form a C with the thumb and forefinger, and he has a slight tremor. The nurse would document this finding as a which positive reflex. a. tonic neck b. glabellar (Myerson) c. Babinski d. Moro

Answer: Moro Rationale: The characteristics displayed by the infant are associated with a positive Moro reflex. The tonic neck reflex occurs when the infant extends the leg on the side to which the infant's head simultaneously turns. The glabellar (Myerson) reflex is elicited by tapping on the infant's head while the eyes are open. A characteristic response is blinking for the first few taps. The Babinski reflex occurs when the sole of the foot is stroked upward along the lateral aspect of the sole and then across the ball of the foot. A positive response occurs when all the toes hyperextend, with dorsiflexion of the big toe.

The nurse should be aware of which important information regarding nerve block analgesia and anesthesia? a. Most local agents are chemically related to cocaine and end in the suffix -caine. b. Local perineal infiltration anesthesia is effective when epinephrine is added, but it can be injected only once. c. Pudendal nerve block is designed to relieve the pain from uterine contractions. d. Pudendal nerve block, if performed correctly, does not significantly lessen the bearing-down reflex.

Answer: Most local agents are chemically related to cocaine and end in the suffix -caine. Rationale: Common agents include lidocaine and chloroprocaine. Injections can be repeated to prolong the anesthesia. A pudendal nerve block relieves pain in the vagina, vulva, and perineum but not the pain from uterine contractions. A pudendal nerve block lessens or shuts down the bearing-down reflex.

Rho immune globulin will be ordered postpartum if which situation occurs? a. Mother Rh-, baby Rh+ b. Mother Rh-, baby Rh- c. Mother Rh+, baby Rh+ d. Mother Rh+, baby Rh-

Answer: Mother Rh-, baby Rh+ Rationale: An Rh- mother delivering an Rh+ baby may develop antibodies to fetal cells that entered her bloodstream when the placenta separated. The Rho immune globulin works to destroy the fetal cells in the maternal circulation before sensitization occurs. If mother and baby are both Rh+ or Rh- the blood types are alike, so no antibody formation would be anticipated. If the Rh+ blood of the mother comes in contact with the Rh- blood of the infant, no antibodies would develop because the antigens are in the mother's blood, not in the infant's.

When the infant's behaviors and characteristics call forth a corresponding set of maternal behaviors and characteristics, what is the correct term for this behavior? a. Mutuality b. Bonding c. Claiming d. Acquaintance

Answer: Mutuality Rationale: Mutuality extends the concept of attachment and includes a shared set of behaviors as part of the bonding process. Bonding is the process during which parents form an emotional attachment to their infant over time. Claiming is the process during which parents identify their new baby in terms of the infant's likeness to other family members and their differences and uniqueness. Like mutuality, acquaintance is part of attachment. It describes how parents get to know their baby during the immediate postpartum period through eye contact, touching, and talking.

How would the physiologic process of the sexual response best be characterized? a. Coitus, masturbation, and fantasy b. Myotonia and vasocongestion c. Erection and orgasm d. Excitement, plateau, and orgasm

Answer: Myotonia and vasocongestion Rationale: Physiologically, sexual response can be analyzed in terms of two processes: vasocongestion and myotonia. Coitus, masturbation, and fantasy are forms of stimulation for the physical manifestation of the sexual response. Erection and orgasm occur in two of the four phases of the sexual response cycle. Excitement, plateau, and orgasm are three of the four phases of the sexual response cycle.

Several delivery changes in the integumentary system that appear during pregnancy disappear after birth, although not always completely. What change is almost certain to be completely reversed? a. Nail brittleness b. Darker pigmentation of the areolae and linea nigra c. Striae gravidarum on the breasts, abdomen, and thighs d. Spider nevi

Answer: Nail brittleness Rationale: The nails return to their prepregnancy consistency and strength. Some women have permanent darker pigmentation of the areolae and linea nigra. Striae gravidarum (stretch marks) usually do not completely disappear. For some women, spider nevi persist indefinitely.

A laboring woman has received meperidine intravenously (IV), 90 minutes before giving birth. Which medication should be available to reduce the postnatal effects of meperidine on the neonate? a. Fentanyl b. Promethazine c. Naloxone d. Nalbuphine

Answer: Naloxone Rationale: An opioid antagonist can be given to the newborn as one part of the treatment for neonatal narcosis, which is a state of central nervous system (CNS) depression in the newborn produced by an opioid. Opioid antagonists, such as naloxone, can promptly reverse the CNS depressant effects, especially respiratory depression. Fentanyl, promethazine, and nalbuphine do not act as opioid antagonists to reduce the postnatal effects of meperidine on the neonate.

A newborn is placed under a radiant heat warmer. The nurse understands that thermoregulation presents a problem for the newborn. What is the rationale for this difficulty? a. The renal function of a newborn is not fully developed, and heat is lost in the urine. b. The small body surface area of a newborn favors more rapid heat loss than does an adult's body surface area. c. Newborns have a relatively thin layer of subcutaneous fat that provides poor insulation. d. Their normal flexed posture favors heat loss through perspiration.

Answer: Newborns have a relatively thin layer of subcutaneous fat that provides poor insulation. Rationale: The newborn has little thermal insulation. Furthermore, the blood vessels are closer to the surface of the skin. Changes in environmental temperature alter the temperature of the blood, thereby influencing temperature regulation centers in the hypothalamus. Heat loss does not occur through urination. Newborns have a higher body surface-to-weight ratio than adults. The flexed position of the newborn helps guard against heat loss, because it NURSINGTB.COM diminishes the amount of body surface exposed to the environment.

A labor and delivery nurse should be cognizant of which information regarding how the fetus moves through the birth canal? a. Fetal attitude describes the angle at which the fetus exits the uterus. b. Of the two primary fetal lies, the horizontal lie is that in which the long axis of the fetus is parallel to the long axis of the mother. c. Normal attitude of the fetus is called general flexion. d. Transverse lie is preferred for vaginal birth.

Answer: Normal attitude of the fetus is called general flexion. Rationale: The normal attitude of the fetus is called general flexion. The fetal attitude is the relationship of the fetal body parts to each one another. The horizontal lie is perpendicular to the mother; in the longitudinal (or vertical) lie, the long axes of the fetus and the mother are parallel. Vaginal birth cannot occur if the fetus stays in a transverse lie.

During labor a fetus displays an average fetal heart rate (FHR) of 135 beats per minute over a 10-minute period. Which statement best describes the status of this fetus? a. Bradycardia b. Normal baseline heart rate c. Tachycardia d. Hypoxia

Answer: Normal baseline heart rate Rationale: The baseline FHR is measured over 10 minutes; a normal range is 110 to 160 beats per minute. Bradycardia is a FHR less than 110 beats per minute for 10 minutes or longer. Tachycardia is a FHR higher than 160 beats per minutes for 10 minutes or longer. Hypoxia is an inadequate supply of oxygen; no indication of hypoxia exists with a baseline FHR in the normal range.

Which traditional family structure is decreasing in numbers and attributable to societal changes? a. Extended family b. Binuclear family c. Nuclear family d. Blended family

Answer: Nuclear family Rationale: The nuclear family has long represented the traditional American family in which husband, wife, and children live as an independent unit. As a result of rapid changes in society, this number is steadily decreasing as other family configurations are socially recognized. Extended families involve additional blood relatives other than the parents. A binuclear family involves two households. A blended family is reconstructed after divorce and involves the merger of two families.

Which statement about the development of cultural competence is inaccurate? a. Local health care workers and community advocates can help extend health care to underserved populations. b. Nursing care is delivered in the context of the client's culture but not in the context of the nurse's culture. c. Nurses must develop an awareness of and a sensitivity to various cultures. d. Culture's economic, religious, and political structures influence practices that affect childbearing.

Answer: Nursing care is delivered in the context of the client's culture but not in the context of the nurse's culture. Rationale: Although the cultural context of the nurse affects the delivery of nursing care and is very important, the work of local health care workers and community advocates, developing sensitivity to various cultures, and the impact of economic, religious, and political structures are all parts of cultural competence.

The woman's family members are present when the nurse arrives for a postpartum and newborn visit. What should the nurse do? a. Observe the family members' interactions with the newborn and one another. b. Ask the woman to meet with her and the baby alone. c. Perform a brief assessment on all family members who are present. d. Reschedule the visit for another time so that the mother and infant can be privately assessed.

Answer: Observe the family members' interactions with the newborn and one another. Rationale: The nurse should introduce her or himself to the client and to the other family members who are present. Family members in the home may be providing care and assistance to the mother and infant. However, this care may not be based on sound health practices. Nurses should take the opportunity to dispel myths while family members are present. The responsibility of the home care maternal-child nurse is to provide care to the new postpartum mother and to her infant, not to all family members. The nurse can politely ask about the other people in the home and their relationships with the mother. Unless an indication is given that the woman would prefer privacy, the visit may continue.

The nurse is preparing to administer a hepatitis B virus (HBV) vaccine to a newborn. Which intervention by the nurse is correct? a. Obtaining a syringe with a 25-gauge, 5/8-inch needle for medication administration b. Confirming that the newborn's mother has been infected with the HBV c. Assessing the dorsogluteal muscle as the preferred site for injection d. Confirming that the newborn is at least 24 hours old

Answer: Obtaining a syringe with a 25-gauge, 5/8-inch needle for medication administration. Rationale: The HBV vaccine should be administered in the vastus lateralis muscle at childbirth with a 25-gauge, 5/8-inch needle and is recommended for all infants. If the infant is born to an infected mother who is a chronic HBV carrier, then the hepatitis vaccine and HBV immunoglobulin should be administered within 12 hours of childbirth.

Under which circumstance should the nurse assist the laboring woman into a hands-and-knees position? a. Occiput of the fetus is in a posterior position. b. Fetus is at or above the ischial spines. c. Fetus is in a vertex presentation. d. Membranes have ruptured.

Answer: Occiput of the fetus is in a posterior position Rationale: The hands-and-knees position is effective in helping to rotate the fetus from a posterior to an anterior position not a vertex position. Many women experience the irresistible urge to push when the fetus is at the level of the ischial spines. In some cases, this urge is felt before the woman is fully dilated. The woman should be instructed not to push until complete cervical dilation has occurred. No one position is correct for childbirth. The two most common positions assumed by women are the sitting and side-lying positions. The woman may be encouraged into a hands-and-knees position if the umbilical cord prolapsed when the membranes ruptured.

Which statement regarding postpartum depression (PPD) is essential for the nurse to be aware of when attempting to formulate a nursing diagnosis? a. PPD symptoms are consistently severe. b. This syndrome affects only new mothers. c. PPD can easily go undetected. d. Only mental health professionals should teach new parents about this condition.

Answer: PPD can easily go undetected. Rationale: PPD can go undetected because parents do not voluntarily admit to this type of emotional distress out of embarrassment, fear, or guilt. PPD symptoms range from mild to severe, with women having both good and bad days. PPD may also affect new fathers. Therefore, both mothers and fathers should be screened. The nurse should include information on PPD and how to differentiate it from the baby blues for all clients before discharge. Nurses can also urge new parents to report symptoms and to seek follow-up care promptly if symptoms occur.

The microscopic examination of scrapings from the cervix, endocervix, or other mucous membranes to detect premalignant or malignant cells is called what? a. Bimanual palpation b. Rectovaginal palpation c. Papanicolaou (Pap) test d. Four As procedure

Answer: Papanicolaou (Pap) test Rationale: The Pap test is a microscopic examination for cancer that should be regularly performed, depending on the client's age. Bimanual palpation is a physical examination of the vagina. Rectovaginal palpation is a physical examination performed through the rectum. The four As procedure is an intervention to help a client stop smoking.

In follow-up appointments or visits with parents and their new baby, it may be useful if the nurse can identify parental behaviors that can either facilitate or inhibit attachment. Which is a facilitating behavior? a. Parents have difficulty naming the infant. b. Parents hover around the infant, directing attention to and pointing at the infant. c. Parents make no effort to interpret the actions or needs of the infant. d. Parents do not move from fingertip touch to palmar contact and holding.

Answer: Parents hover around the infant, directing attention to and pointing at the infant. Rationale: Hovering over the infant and obviously paying attention to the baby are facilitating behaviors. Inhibiting behaviors include difficulty naming the infant, making no effort to interpret the actions or needs of the infant, and not moving from fingertip touch to palmar contact and holding.

The postpartum nurse should be cognizant of what about the adaptation of other family members (primarily siblings and grandparents) to the newborn? a. Sibling rivalry cannot be dismissed as overblown psychobabble; negative feelings and behaviors can take a long time to blow over. b. Participation in preparation classes helps both siblings and grandparents. c. In the United States, paternal and maternal grandparents consider themselves of equal importance and status. d. Since 1990, the number of grandparents providing permanent care to their grandchildren has been declining.

Answer: Participation in preparation classes helps both siblings and grandparents. Rationale: Preparing older siblings, as well as grandparents, helps with everyone's adaptation. Sibling rivalry should be initially expected, but the negative behaviors associated with it have been overemphasized and stop in a comparatively short time. In the United States, in contrast to other cultures, paternal grandparents frequently consider themselves secondary to maternal grandparents. The number of grandparents providing permanent child care has been rising.

How would the nurse differentiate a meconium stool from a transitional stool in the healthy newborn? a. Observed at age 3 days b. Is residue of a milk curd c. Passes in the first 12 to 24 hours of life d. Is lighter in color and looser in consistency

Answer: Passes in the first 12 to 24 hours of life Rationale: Meconium stool is usually passed in the first 12 hours of life, and 99% of newborns have their first stool within 48 hours. If meconium is not passed by 48 hours, then obstruction is suspected. Meconium stool is the first stool of the newborn and is made up of matter remaining in the intestines during intrauterine life. Meconium is dark and stick.

A nurse caring for a woman in labor should understand that absent or minimal variability is classified as either abnormal or indeterminate. Which condition related to decreased variability is considered benign? a. Periodic fetal sleep state b. Extreme prematurity c. Fetal hypoxemia d. Preexisting neurologic injury

Answer: Periodic fetal sleep state Rationale: When the fetus is temporarily in a sleep state, minimal variability is present. Periodic fetal sleep states usually last no longer than 30 minutes. A woman in labor with extreme prematurity may display a FHR pattern of minimal or absent variability. Abnormal variability may also be related to fetal hypoxemia and metabolic acidemia. Congenital anomalies or a preexisting neurologic injury may also result in absent or minimal variability. Other possible causes might be central nervous system (CNS) depressant medications, narcotics, or general anesthesia.

A 3.8-kg infant was vaginally delivered at 39 weeks of gestation after a 30-minute second stage. A nuchal cord occurred. After the birth, the infant is noted to have petechiae over the face and upper back. Based on the nurse's knowledge, which information regarding petechiae should be shared with the parents? a. Petechiae are benign if they disappear within 48 hours of childbirth. b. These hemorrhagic areas may result from increased blood volume. c. Petechiae should always be further investigated. d. Petechiae usually occur with a forceps delivery.

Answer: Petechiae are benign if they disappear within 48 hours of childbirth. Rationale: Petechiae that are acquired during birth may extend over the upper portion of the trunk and face. These lesions are benign if they disappear within 2 or 3 days of birth and no new lesions appear. Petechiae may result from decreased platelet formation. In this infant, the presence of petechiae is more likely a soft-tissue injury resulting from the nuchal cord at birth. Unless the lesions do not dissipate in 2 days, no reason exists to alarm the family. Petechiae usually occur with a breech presentation vaginal birth.

What marks on a baby's skin may indicate an underlying problem that requires notification of a physician? a. Mongolian spots on the back b. Telangiectatic nevi on the nose or nape of the neck c. Petechiae scattered over the infant's body d. Erythema toxicum neonatorum anywhere on the body

Answer: Petechiae scattered over the infant's body Rationale: Petechiae (bruises) scattered over the infant's body should be reported to the pediatrician because they may indicate underlying problems such as infection or low platelet count. Mongolian spots are bluish-black spots that resemble bruises but gradually fade over months and have no clinical significance. Telangiectatic nevi (stork bites, angel kisses) fade by the second year and have no clinical significance. Erythema toxicum neonatorum is an appalling-looking rash; however, it has no clinical significance and requires no treatment.

What is the nurse's initial action while caring for an infant with a slightly decreased temperature? a. Immediately notify the physician. b. Place a cap on the infant's head, and have the mother perform kangaroo care. c. Tell the mother that the infant must be kept in the nursery and observed for the next 4 hours. d. Change the formula; a decreased body temperature is a sign of formula intolerance.

Answer: Place a cap on the infant's head, and have the mother perform kangaroo care. Rationale: Keeping the head well covered with a cap prevents further heat loss from the head and placing the infant skin-to-skin against the mother should increase the infant's temperature. Nursing actions are needed first to correct the problem. If the problem persists after the interventions, physician notification may then be necessary. A slightly decreased temperature can be treated in the mother's room, offering an excellent time for parent teaching on the prevention of cold stress. Mild temperature instability is an expected deviation from normal during the first days after childbirth as the infant adapts to external life.

As part of their teaching function at discharge, nurses should educate parents regarding safe sleep. Based on the most recent evidence, which information is incorrect and should be discussed with parents? a. Prevent exposure to people with upper respiratory tract infections. b. Keep the infant away from secondhand smoke. c. Avoid loose bedding, water beds, and beanbag chairs. d. Place the infant on his or her abdomen to sleep.

Answer: Place the infant on his or her abdomen to sleep. Rationale: The infant should be laid down to sleep on his or her back for better breathing and to prevent sudden infant death syndrome (SIDS). Grandmothers may encourage the new parents to place the infant on the abdomen; however, evidence shows "back to sleep" reduces SIDS. Infants are vulnerable to respiratory infections; therefore, infected people must be kept away. Secondhand smoke can damage lungs. Infants can suffocate in loose bedding and in furniture that can trap them. Per AAP guidelines, infants should always be placed "back to sleep" and allowed tummy time to play to prevent plagiocephaly.

A newborn is jaundiced and is receiving phototherapy via ultraviolet bank lights. What is the most appropriate nursing intervention when caring for an infant with hyperbilirubinemia and receiving phototherapy? a. Applying an oil-based lotion to the newborn's skin to prevent dying and cracking b. Limiting the newborn's intake of milk to prevent nausea, vomiting, and diarrhea c. Placing eye shields over the newborn's closed eyes d. Changing the newborn's position every 4 hours

Answer: Placing eye shields over the newborn's closed eyes Rationale: The infant's eyes must be protected by an opaque mask to prevent overexposure to the light. Eye shields should completely cover the eyes but not occlude the nares. Lotions and ointments should not be applied to the infant because they absorb heat and can cause burns. The lights increase insensible water loss, placing the infant at risk for fluid loss and dehydration. Therefore, adequate hydration is important for the infant. The infant should be turned every 2 hours to expose all body surfaces to the light.

Which action by the mother will initiate the milk ejection reflex (MER)? a a. Wearing a firm-fitting bra b. Drinking plenty of fluids c. Placing the infant to the breast d. Applying cool packs to her breast

Answer: Placing the infant to the breast Rationale : Oxytocin, which causes the MER reflex, increases in response to nipple stimulation. A firm bra is important to support the breast; however, it will not initiate the MER reflex. Drinking plenty of fluids is necessary for adequate milk production, but adequate intake of water alone will not initiate the MER reflex. Cool packs to the breast will decrease the MER reflex.

Which basic type of pelvis includes the correct description and percentage of occurrence in women? a. Gynecoid: classic female pelvis; heart shaped; 75% b. Android: resembling the male pelvis; wide oval; 15% c. Anthropoid: resembling the pelvis of the ape; narrow; 10% d. Platypelloid: flattened, wide, and shallow pelvis; 3%

Answer: Platypelloid: flattened, wide, and shallow pelvis; 3% Rationale: A platypelloid pelvis is flattened, wide, and shallow; approximately 3% of women have this shape. The gynecoid pelvis is the classic female shape, slightly ovoid and rounded; approximately 50% of women have this shape. An android or malelike pelvis is heart shaped; approximately 23% of women have this shape. An anthropoid or apelike pelvis is oval and wide; approximately 24% of women have this shape.

Which statement by the nurse can assist a new father in his transition to parenthood? a. Pointing out that the infant turned at the sound of his voice b. Encouraging him to go home to get some sleep c. Telling him to tape the infant's diaper a different way d. Suggesting that he let the infant sleep in the bassinet

Answer: Pointing out that the infant turned at the sound of his voice Rationale: Infants respond to the sound of voices. Because attachment involves a reciprocal interchange, observing the interaction between parent and infant is very important. Separation of the parent and infant does not encourage parent-infant attachment. Educating the parent in infant care techniques is important, but the way a diaper is taped is not relevant and does not enhance parent-infant interactions. Parent-infant attachment involves touching, holding, and cuddling. It is appropriate for a father to want to hold the infant as the baby sleeps.

Which instruction should the nurse provide to reduce the risk of nipple trauma? a. Limit the feeding time to less than 5 minutes. b. Position the infant so the nipple is far back in the mouth. c. Assess the nipples before each feeding. d. Wash the nipples daily with mild soap and water.

Answer: Position the infant so the nipple is far back in the mouth. Rationale: If the infant's mouth does not cover as much of the areola as possible, the pressure during sucking will be applied to the nipple, thus causing trauma to the area. Stimulating the breast for less than 5 minutes will not produce the extra milk the infant may need and will also limit access to the higher-fat hindmilk. Assessing the nipples for trauma is important; however, this action alone will not prevent sore nipples. Soap can be drying to the nipples and should be avoided during breastfeeding.

A nurse is observing a family. The mother is holding the baby she delivered less than 24 hours ago. Her husband is watching his wife and asking questions about newborn care. The 4-year-old brother is punching his mother on the back. How should the nurse react to this situation? a. Report the incident to the social services department. b. Advise the parents that the toddler needs to be reprimanded. c. Report to oncoming staff that the mother is probably not a good disciplinarian. d. Realize that this is a normal family unit adjusting to a major family change.

Answer: Realize that this is a normal family unit adjusting to a major family Rationale: The observed behaviors are normal variations of a family adjusting to change. Reporting this one incident is not needed. Offering advice at this point would make the parents feel inadequate.

. During a telephone follow-up conversation with a woman who is 4 days postpartum, the woman tells the nurse, "I don't know what's wrong. I love my son, but I feel so let down. I seem to cry for no reason!" Which condition might this new mother be experiencing? a. Letting-go b. Postpartum depression (PPD) c. Postpartum blues d. Attachment difficulty

Answer: Postpartum blues Rationale: During the postpartum blues, women are emotionally labile, often crying easily and for no apparent reason. This lability seems to peak around the fifth postpartum day. The letting-go the phase is the period that occurs several weeks after childbirth. During this phase the woman wants to move forward as a family unit with all members, appropriately interacting to their new roles. PPD is an intense, pervasive sadness marked by severe, labile mood swings; it is more serious and persistent than the postpartum blues. Crying is not a maladaptive attachment response; it indicates postpartum blues.

The nurse should be aware of which physiologic effect of labor pain? a. Predominant pain of the first stage of labor is visceral pain that is in the lower portion of the abdomen. b. Referred pain is the extreme discomfort experienced between contractions. c. Somatic pain of the second stage of labor is more generalized and related to fatigue. d. Pain during the third stage is a somewhat milder version of the pain experienced during the second stage.

Answer: Predominant pain of the first stage of labor is visceral pain that is in the lower portion of the abdomen. Rationale: Predominant pain comes from cervical changes, the distention of the lower uterine segment, and uterine ischemia. Referred pain occurs when the pain that originates in the uterus radiates to the abdominal wall, lumbosacral area of the back, iliac crests, and gluteal area. Second-stage labor pain is intense, sharp, burning, and localized. Third-stage labor pain is like that of the first stage.

Parents have been asked by the neonatologist to provide breast milk for their newborn son, who was born prematurely at 32 weeks of gestation. The nurse who instructs them regarding pumping, storing, and transporting the milk needs to assess their knowledge of lactation. Which statement is a valid basis for the instructions? a. Premature infants more easily digest breast milk than formula. b. A glass of wine just before pumping will help reduce stress and anxiety. c. The mother should only pump as much milk as the infant can drink. d. The mother should pump every 2 to 3 hours, including during the night.

Answer: Premature infants more easily digest breast milk than formula. Rationale: Human milk is the ideal food for preterm infants, with benefits that are unique, in addition to those benefits received by full-term, healthy infants. Greater physiologic stability occurs with breastfeeding, compared with formula feeding. Consumption of alcohol during lactation is approached with caution. Excessive amounts can have serious effects on the infant and can adversely affect the mother's milk ejection reflex. To establish an optimal milk supply, the most appropriate instruction for the mother should be to pump 8 to 10 times a day for 10 to 15 minutes on each breast.

What is the nurse's understanding of the appropriate role of primary and secondary powers? a. Primary powers are responsible for the effacement and dilation of the cervix. b. Effacement is generally well ahead of dilation in women giving birth for the first time; they are closer together in subsequent pregnancies. c. Scarring of the cervix caused by a previous infection or surgery may make the delivery a bit more painful, but it should not slow or inhibit dilation. d. Pushing in the second stage of labor is more effective if the woman can breathe deeply and control some of her involuntary needs to push, as the nurse directs.

Answer: Primary powers are responsible for the effacement and dilation of the cervix. Rationale: The primary powers are responsible for dilation and effacement; secondary powers are concerned with expulsion of the fetus. Effacement is generally well ahead of dilation in first-time pregnancies; they are closer together in subsequent pregnancies. Scarring of the cervix may slow dilation. Pushing is more effective and less fatiguing when the woman begins to push only after she has the urge to do so.

In which clinical situation would the nurse most likely anticipate a fetal bradycardia? a. Intraamniotic infection b. Fetal anemia c. Prolonged umbilical cord compression d. Tocolytic treatment using terbutaline

Answer: Prolonged umbilical cord compression Rationale: Fetal bradycardia can be considered a later sign of fetal hypoxia and is known to occur before fetal death. Bradycardia can result from placental transfer of drugs, prolonged compression of the umbilical cord, maternal hypothermia, and maternal hypotension. Intraamniotic infection, fetal anemia, and tocolytic treatment using terbutaline would most likely result in fetal tachycardia.

What fatty acids (classified as hormones) are found in many body tissues with complex roles in many reproductive functions? a. GnRH b. Prostaglandins (PGs) c. Follicle-stimulating hormone (FSH) d. Luteinizing hormone (LH)

Answer: Prostaglandins (PGs) Rationale: PGs affect smooth muscle contraction and changes in the cervix. GnRH is part of the hypothalamic-pituitary cycle, which responds to the rise and fall of estrogen and progesterone. FSH is part of the hypothalamic-pituitary cycle, which responds to the rise and fall of estrogen and progesterone. LH is part of the hypothalamic-pituitary cycle, which responds to the rise and fall of estrogen and progesterone.

When attempting to communicate with a client who speaks a different language, which action is the most appropriate? a. Promptly and positively respond to project authority. b. Never use a family member as an interpreter. c. Talk to the interpreter to avoid confusing the client. d. Provide as much privacy as possible.

Answer: Provide as much privacy as possible. Rationale: Providing privacy creates an atmosphere of respect and puts the client at ease. The nurse should not rush to judgment and should ensure she or he clearly understands the client's message. In crisis situations, the nurse may need to use a family member or neighbor as a translator. The nurse should speak directly to the client to create an atmosphere of respect.

Which breathing pattern should the nurse support for the woman and her coach during the latent phase of the first stage of labor if the couple has attended childbirth preparation classes? a. Slow-paced breathing b. Deep abdominal breathing c. Modified-paced breathing d. Patterned-paced breathing

Answer: Slow-paced breathing Rationale: Slow-paced breathing is approximately one half the woman's normal breathing rate and is used during the early stages of labor when a woman can no longer walk or talk through her contractions. No such pattern called deep abdominal breathing exists in childbirth preparation. Modified-paced breathing is shallow breathing that is twice the woman's normal breathing rate. It is used when labor progresses, and the woman can no longer maintain relaxation through paced breathing. Patterned-pace breathing is a fast, 4:1 breathe, breathe, breathe, blow pattern that is used during the transitional phase of labor just before pushing and delivery.

After giving birth to a healthy infant boy, a primiparous client, 16 years of age, is admitted to the postpartum unit. An appropriate nursing diagnosis for her is "Lack of understanding of infant care." What should the nurse be certain to include in the plan of care as he or she prepares the client for discharge? a. Teach the client how to feed and bathe her infant. b. Give the client written information on bathing her infant. c. Advise the client that all mothers instinctively know how to care for their infants. d. Provide time for the client to bathe her infant after she views a demonstration of infant bathing.

Answer: Provide time for the client to bathe her infant after she views a demonstration of infant bathing. Rationale: Having the mother demonstrate infant care is a valuable method of assessing the client's understanding of her newly acquired knowledge, especially in this age group, because she may inadvertently neglect her child. Although verbalizing how to care for the infant is a form of client education or providing written information might be useful, neither is the most developmentally appropriate teaching method for a teenage mother. Advising the young woman that all mothers instinctively know how to care for their infants is inappropriate; it is belittling and false.

Which statement regarding genetic health care is considered a priority to the nurse practicing in this specialty? a. Genetic disorders equally affect people of all socioeconomic backgrounds, races, and ethnic groups. b. Genetic health care is more concerned with populations than individuals. c. Providing emotional support to the family during counseling is the most important of all nursing functions. d. Taking genetic histories is the province of large universities and medical centers.

Answer: Providing emotional support to the family during counseling is the most important of all nursing functions Rationale: Perhaps the most important of all nursing functions is the ability to provide emotional support. Nurses should be prepared to help with various stress reactions from a couple facing the possibility of a genetic disorder. Although anyone may have a genetic disorder, certain disorders appear more often in certain ethnic and racial groups. Genetic health care is highly individualized because treatments are based on the phenotypic responses of the individual. Individual nurses at any facility can take a genetic history, although larger facilities may have better support services.

What is the correct terminology for the nerve block that provides anesthesia to the lower vagina and perineum? a. Epidural b. Pudendal c. Local d. Spinal block

Answer: Pudendal Rationale: A pudendal block anesthetizes the lower vagina and perineum to provide anesthesia for an episiotomy and the use of low forceps, if needed. An epidural provides anesthesia for the uterus, perineum, and legs. A local provides anesthesia for the perineum at the site of the episiotomy. A spinal block provides anesthesia for the uterus, perineum, and down the legs.

Which term best describes the interval between the birth of the newborn and the return of the reproductive organs to their normal nonpregnant state? a. Involutionary period because of changes to the uterus b. Lochia period because of the nature of the vaginal discharge c. Mini-tri period because it lasts only 3 to 6 weeks d. Puerperium, or fourth trimester of pregnancy

Answer: Puerperium, or fourth trimester of pregnancy Rationale: The puerperium, also called the fourth trimester or the postpartum period of pregnancy, is the final period of pregnancy and lasts approximately 3 to 6 weeks. Involution marks the end of the puerperium. Lochia refers to the various vaginal discharges during the puerperium. Mini-tri period is not a commonly used term.

When assessing the fetus using Leopold's maneuvers, the nurse feels a round, firm, and movable fetal part in the fundal portion of the uterus and a long, smooth surface in the mother's right side close to midline. What is the position of the fetus? a. ROA b. LSP c. RSA d. LOA

Answer: RSA Rationale: Fetal position is denoted with a three-letter abbreviation. The first letter indicates the presenting part in either the right or the left side of the maternal pelvis. The second letter indicates the anatomic presenting part of the fetus. The third letter stands for the location of the presenting part in relationship to the anterior, posterior, or transverse portion of the maternal pelvis. Palpation of a round, firm fetal part in the fundal portion of the uterus would be the fetal head, indicating that the fetus is in a breech position with the sacrum as the presenting part in the maternal pelvis. Palpation of the fetal spine along the mother's right side denotes the location of the presenting part in the mother's pelvis. The ability to palpate the fetal spine indicates that the fetus is anteriorly positioned in the maternal pelvis. This fetus is anteriorly positioned in the right side of the maternal pelvis with the sacrum as the presenting part. RSA is the correct three-letter abbreviation to indicate this fetal position. ROA denotes a fetus that is anteriorly positioned in the right side of the maternal pelvis with the occiput as the presenting part. LSP describes a fetus that is posteriorly positioned in the left side of the pelvis with the sacrum as the presenting part. A fetus that is LOA would be anteriorly positioned in the left side of the pelvis with the occiput as the presenting part.

A 30-year-old multiparous woman has a boy who is years old and has recently delivered an infant girl. She tells the nurse, "I don't know how I'll ever manage both children when I get home." Which suggestion would assist this new mother in alleviating sibling rivalry? a. Tell the older child that he is a big boy now and should love his new sister. b. Let the older child stay with his grandparents for the first 6 weeks to allow him to adjust to the newborn. c. Ask friends and relatives not to bring gifts to the older sibling because you do not want to spoil him. d. Realize that the regression in habits and behaviors in the older child is a typical reaction and that he needs extra love and attention at this time.

Answer: Realize that the regression in habits and behaviors in the older child is a typical reaction and that he needs extra love and attention at this time. Rationale: The older child may regress in habits or behaviors (e.g., toileting, sleep habits) as a method of seeking attention. Parents need to distribute their attention in an equitable manner. Telling the older child that he should love his new sister is a negative approach to facilitating sibling acceptance of the new infant. Reactions of siblings may result from temporary separation from the mother. Removing the older child from the home when the new infant arrives may enhance negative behaviors from the older child caused by a separation from the mother. Providing small gifts from the infant to the older child is a strategy for facilitating sibling acceptance of the new infant.

An 18-year-old pregnant woman, gravida 1, para 0, is admitted to the labor and birth unit with moderate contractions every 5 minutes that last 40 seconds. The client states, "My contractions are so strong, I don't know what to do." What should the nurse's first action be? a. Assess for fetal well-being. b. Encourage the woman to lie on her side. c. Disturb the woman as little as possible. d. Recognize that pain is personalized

Answer: Recognize that pain is personalized Rationale: Each woman's pain during childbirth is unique and is influenced by a variety of physiologic, psychosocial, and environmental factors. A critical issue for the nurse is how support can make a difference in the pain of the woman during labor and birth. This scenario includes no information that would indicate fetal distress or a logical reason to be overly concerned about the well-being of the fetus. The left lateral position is used to alleviate fetal distress, not maternal stress. The nurse has an obligation to provide physical, emotional, and psychosocial care and support to the laboring woman. This client clearly needs support.

What should the nurse's next action be if the client's white blood cell (WBC) count is 25,000/mm3on her second postpartum day? a. Immediately inform the health care provider. b. Have the laboratory draw blood for reanalysis. c. Recognize that this count is an acceptable range at this point postpartum. d. Immediately begin antibiotic therapy.

Answer: Recognize that this count is an acceptable range at this point postpartum. Rationale: During the first 10 to 12 days after childbirth, WBC values up to 30,000/mm3are common. Because a WBC count of 25,000/mm3on her second postpartum day is normal, alerting the health care provider is not warranted nor is reassessment or antibiotics needed; the WBC count is not elevated and so no reanalysis is required.

In recovery, if a woman is asked to either raise her legs (knees extended) off the bed or flex her knees, and then place her feet flat on the bed and raise her buttocks well off the bed, the purpose of this exercise is to assess what? a. Recovery from epidural or spinal anesthesia b. Hidden bleeding underneath her c. Flexibility d. Whether the woman is a candidate to go home after 6 hours

Answer: Recovery from epidural or spinal anesthesia Rationale: If the numb or prickly sensations are gone from her legs after these movements, then she has likely recovered from the epidural or spinal anesthesia. Assessing the client for bleeding beneath her buttocks before discharge from the recovery is always important; however, she should be rolled to her side for this assessment. The nurse is not required to assess the woman for flexibility. This assessment is performed to evaluate whether the client has recovered from spinal anesthesia, not to determine if she is a candidate for early discharge.

What is the primary rationale for the thorough drying of the infant immediately after birth? a. Stimulates crying and lung expansion b. Removes maternal blood from the skin surface c. Reduces heat loss from evaporation d. Increases blood supply to the hands and feet

Answer: Reduces heat loss from evaporation Rationale: Infants are wet with amniotic fluid and blood at birth, and this accelerates evaporative heat loss. The primary purpose of drying the infant is to prevent heat loss. Although rubbing the infant stimulates crying, it is not the main reason for drying the infant. This intervention would have only minimal effect of circulation to the hands and feet. This process does not remove all the maternal blood.

What information should the nurse be aware of regarding telephonic nursing care such as warm lines? a. Were developed as a reaction to impersonal telephonic nursing care b. Were set up to take complaints concerning health maintenance organizations (HMOs) c. Are the second option when 9-1-1 hotlines are busy d. Refer to community service telephone lines designed to provide new parents with encouragement and basic information

Answer: Refer to community service telephone lines designed to provide new parents with encouragement and basic information Rationale: Warm lines are one aspect of telephonic nursing care specifically designed to provide new parents with encouragement and basic information. Warm lines and similar services sometimes are set up by HMOs to provide new parents with encouragement and basic information. The name, warm lines, may have been suggested by the term hotlines, but these are not emergency numbers but are designed to provide new parents with encouragement and basic information.

The nurse should be cognizant of which important information regarding the gastrointestinal (GI) system of the newborn? a. The newborn's cheeks are full because of normal fluid retention. b. The nipple of the bottle or breast must be placed well inside the baby's mouth because teeth have been developing in utero, and one or more may even be through. c. Regurgitation during the first day or two can be reduced by burping the infant and slightly elevating the baby's head. d. Bacteria are already present in the infant's GI tract at birth because they traveled through the placenta.

Answer: Regurgitation during the first day or two can be reduced by burping the infant and slightly elevating the baby's head. Rationale: Avoiding overfeeding can also reduce regurgitation. The newborn's cheeks are full because of well-developed sucking pads. Teeth do develop in utero, but the nipple is placed deep because the baby cannot move food from the lips to the pharynx. Bacteria are not present at birth, but they soon enter through various orifices.

The client has delivered by urgent caesarean birth for fetal compromise. Umbilical cord gases were obtained for acid-base determination. The pH is 6.9, partial pressure of carbon dioxide (PCO2) is elevated, and the base deficit is 11 mmol/L. What type of acidemia is displayed by the infant? a. Respiratory b. Metabolic c. Mixed d. Turbulent

Answer: Respiratory Rationale: These findings are evidence of respiratory acidemia. Metabolic acidemia is expressed by a pH <7.20, normal carbon dioxide pressure, and a base excess of ≥12 mmol/L. Mixed acidemia is evidenced by a pH <7.20, elevated carbon dioxide pressure, and a base excess of ≥12 mmol/L. There is no such finding as turbulent acidemia.

Which statement by the client would lead the nurse to believe that labor has been established? a. "I passed some thick, pink mucus when I urinated this morning." b. "My bag of waters just broke." c. "The contractions in my uterus are getting stronger and closer together." d. "My baby dropped, and I have to urinate more frequently now."

Answer: The contractions in my uterus are getting stronger and closer together." Rationale: Regular, strong contractions with the presence of cervical change indicate that the woman is experiencing true labor. Although the loss of the mucous plug (operculum) often occurs during the first stage of labor or before the onset of labor, it is not the indicator of true labor. Spontaneous rupture of membranes often occurs during the first stage of labor; however, it is not an indicator of true labor. The presenting part of the fetus typically becomes engaged in the pelvis at the onset of labor but is not the indicator of true labor.

The nurse should be cognizant of which postpartum physiologic alteration? a. Cardiac output, pulse rate, and stroke volume all return to prepregnancy normal values within 48 hours of childbirth. b. Respiratory function returns to nonpregnant levels by 6 to 8 weeks after childbirth. c. Lowered white blood cell count after pregnancy can lead to false-positive results on tests for infections. d. Hypercoagulable state protects the new mother from thromboembolism, especially after a cesarean birth.

Answer: Respiratory function returns to nonpregnant levels by 6 to 8 weeks after childbirth. Rationale: Pulse, along with stroke volume and cardiac output, remains elevated for the first hour or so after birth. It gradually decreases over the first 48 hours postpartum. Respirations should decrease to within the woman's normal prepregnancy range soon after childbirth. Leukocytosis increases 10 to 12 days after childbirth, which can obscure the diagnosis of acute infections, producing false-negative test results. The hypercoagulable state increases the risk of thromboembolism, especially after a cesarean birth.

What is the goal of a long-term treatment plan for an adolescent with an eating disorder? a. Managing the effects of malnutrition b. Establishing sufficient caloric intake c. Improving family dynamics d. Restructuring client perception of body image

Answer: Restructuring client perception of body image Rationale: The treatment of eating disorders is initially focused on reestablishing physiologic homeostasis. Once body systems are stabilized, the next goal of treatment for eating disorders is maintaining adequate caloric intake. Although family therapy is indicated when dysfunctional family relationships exist, the primary focus of therapy for eating disorders is to help the adolescent cope with complex issues. The focus of treatment in individual therapy for an eating disorder involves restructuring cognitive perceptions about the individual's body image.

Which technique is an adequate means of controlling the birth of the fetal head during delivery in a vertex presentation? a. Ritgen maneuver b. Fundal pressure c. Lithotomy position d. De Lee apparatus

Answer: Ritgen maneuver Rationale: The Ritgen maneuver extends the head during the actual birth and protects the perineum. Gentle, steady pressure against the fundus of the uterus facilitates vaginal birth. The lithotomy position has been commonly used in Western cultures, partly because it is convenient for the health care provider. The De Lee apparatus is used to suction fluid from the infant's mouth.

A new father has just been told that his infant has trisomy 18. Which identifying physical feature is unique to an infant with this genetic disorder? a. Microcephaly and capillary hemangiomas b. Epicanthal folds and a simian crease c. Oblique palpebral fissures and Cri du chat syndrome d. Rocker-bottom feet and clenched hands with overlapping fingers

Answer: Rocker-bottom feet and clenched hands with overlapping fingers Rationale: Rocker-bottom feet and clenched hands with overlapping fingers are associated with trisomy 18. Microcephaly and capillary hemangiomas are associated with trisomy 13. Epicanthal folds and a simian crease are associated with trisomy 21 (Down syndrome). Deletion of the short arm of chromosome number 5 is manifested by Cri du chat syndrome.

The laboratory results for a postpartum woman are as follows: blood type, A; Rh status, NURSINGTB.COM positive; rubella titer, 1:8 (enzyme immunoassay [EIA] 0.8); hematocrit, 30%. How should the nurse best interpret these data? a. Rubella vaccine should be administered. b. Blood transfusion is necessary. c. Rh immune globulin is necessary within 72 hours of childbirth. d. Kleihauer-Betke test should be performed. ANS: A This client's rubella titer indicates that she is not immune and needs to receive a vaccine. These data do not indicate that the client needs a blood transfusion. Rh immune globulin is indicated only if the client has an Rh-negative status and the infant has an Rh-positive status. A Kleihauer-Betke test should be performed if a large fetomaternal transfusion is suspected, especially if the mother is Rh negative. However, the data provided do not indicate a need for performing this test.

Answer: Rubella vaccine should be administered.

Which questionnaire would be best for the nurse to use when screening an adolescent client for an eating disorder? a. Four Cs b. Dietary Guidelines for Americans c. SCOFF screening tool d. Dual-energy x-ray absorptiometry (DEXA) scan

Answer: SCOFF screening tool Rationale: A screening tool specifically developed to identify eating disorders uses the acronym SCOFF. Each question scores 1 point. A score of 2 or more indicates that the client may have anorexia nervosa or bulimia. The letters represent the following questions: • Do you make yourself Sick because you feel too full? • Do you worry about loss of Control over the amount that you eat? • Have you recently lost more than One stone (14 pounds) in a 3-month period? • Do you think that you are too Fat, even if others think you are thin? • Does Food dominate your life? The 4 Cs are used to determine cultural competence. Dietary Guidelines for Americans provide nutritional guidance for all, not only for those with eating disorders. The DEXA scan is used to determine bone density.

Which phase of the endometrial cycle best describes a heavy, velvety soft, fully matured endometrium? a. Menstrual b. Proliferative c. Secretory d. Ischemic

Answer: Secretory Rationale: The secretory phase extends from the day of ovulation to approximately 3 days before the next menstrual cycle. During this secretory phase, the endometrium becomes fully mature again. During the menstrual phase, the endometrium is shed. The proliferative phase is a period of rapid growth. During the ischemic phase, the blood supply is blocked, and necrosis develops.

Which alterations in the perception of pain by a laboring client should the nurse understand? a. Sensory pain for nulliparous women is often greater than for multiparous women during early labor. b. Affective pain for nulliparous women is usually less than for multiparous women throughout the first stage of labor. c. Women with a history of substance abuse experience more pain during labor. d. Multiparous women have more fatigue from labor and therefore experience more pain.

Answer: Sensory pain for nulliparous women is often greater than for multiparous women during early labor. Rationale: Sensory pain is greater for nulliparous women because their reproductive tract structures are less supple. Affective pain is greater for nulliparous women during the first stage but decreases for both nulliparous and multiparous during the second stage. Women with a history of substance abuse experience the same amount of pain as those without such a history. Nulliparous women have longer labors and therefore experience more fatigue.

Anxiety is commonly associated with pain during labor. Which statement regarding anxiety is correct? a. Even mild anxiety must be treated. b. Severe anxiety increases tension, increases pain, and then, in turn, increases fear and anxiety, and so on. c. Anxiety may increase the perception of pain, but it does not affect the mechanism of labor. d. Women who have had a painful labor will have learned from the experience and have less anxiety the second time because of increased familiarity.

Answer: Severe anxiety increases tension, increases pain, and then, in turn, increases fear and anxiety, and so on. Rationale: Anxiety and pain reinforce each other in a negative cycle that will slow the progress of labor. Mild anxiety is normal for a woman in labor and likely needs no special treatment other than the standard reassurances. Anxiety increases muscle tension and ultimately can sufficiently build to slow the progress of labor. Unfortunately, an anxious, painful first labor is likely to carry over, through expectations and memories, into an anxious and painful experience in the second pregnancy.

A woman has requested an epidural for her pain. She is 5 cm dilated and 100% effaced. The baby is in a vertex position and is engaged. The nurse increases the woman's IV fluid for a preprocedural bolus. The nurse reviews her laboratory values and notes that the woman's hemoglobin is 12 g/dl, hematocrit is 38%, platelets are 67,000, and white blood cells (WBCs) are 12,000/mm3. Which factor would contraindicate an epidural for this woman? a. She is too far dilated. b. She is anemic. c. She has thrombocytopenia. d. She is septic.

Answer: She has thromocytopenia. Rationale: The platelet count indicates a coagulopathy, specifically, thrombocytopenia (low platelets), which is a contraindication to epidural analgesia and anesthesia. Typically, epidural analgesia and anesthesia are used in the laboring woman when a regular labor pattern has been achieved, as evidenced by progressive cervical change. The laboratory values show that the woman's hemoglobin and hematocrit levels are in the normal range and show a slight increase in the WBC count that is not uncommon in laboring women.

The nurse observes several interactions between a postpartum woman and her new son. What behavior, if exhibited by this woman, would the nurse identify as a possible maladaptive behavior regarding parent-infant attachment? a. The postpartum woman talks and coos to her son. b. She seldom makes eye contact with her son. c. The mother cuddles her son close to her. d. She tells visitors how well her son is feeding.

Answer: She seldom makes eye contact with her son. Rationale: The mother should be encouraged to hold her infant in the face position and make eye contact with the infant. Normal infant-parent interactions include talking and cooing to her son, cuddling her son close to her, and telling visitors how well her son is feeding.

Which information should the nurse provide to a breastfeeding mother regarding optimal self-care? a. She will need an extra 1000 calories a day to maintain energy and produce milk. b. She can return to prepregnancy consumption patterns of any drinks as long as she gets enough calcium. c. She should avoid trying to lose large amounts of weight. d. She must avoid exercising because it is too fatiguing.

Answer: She should avoid trying to lose large amounts of weight. Rationale: Large weight loss releases fat-stored contaminants into her breast milk, and it also involves eating too little and/or exercising too much. A breastfeeding mother needs to add only 200 to 500 extra calories to her diet to provide the extra nutrients for her infant. However, this is true only if she does not drink alcohol, limits coffee to no more than two cups (including caffeine in chocolate, tea, and some sodas, too), and carefully reads the herbal tea ingredients. Although she needs her rest, moderate exercise is healthy.

In the 1970s, the rape-trauma syndrome (RTS) was identified as a cluster of symptoms and related behaviors observed in the weeks and months after an episode of rape. Researchers identified three phases related to this condition. Which phase is not displayed in a client with RTS? a. Acute Phase: Disorganization b. Outward Adjustment Phase c. Shock/Disbelief: Disorientation Phase d. Long-Term Process: Reorganization Phase

Answer: Shock/Disbelief: Disorientation Phase Rationale: Shock, disbelief, or disorientation is a component of the Acute Phase. The rape survivor feels embarrassed, degraded, fearful, and angry. She may feel unclean and want to bathe and douche repeatedly, even though doing so may destroy evidence. The victim relives the scene over and over in her mind, thinking of things she "should have done." During the Outward Adjustment Phase, the victim may appear to have resolved her crisis and return to activities of daily living and work. Other women may move, leave their job, and buy a weapon to protect themselves. Disorientation is a reaction during which the victim may feel disoriented, have difficulty concentrating, or have poor recall. The Long-Term Process is the reorganization phase. This recovery phase may take years and may be difficult and painful.

The nurse observes that a first-time mother appears to ignore her newborn. Which strategy should the nurse use to facilitate mother-infant attachment? a. Tell the mother she must pay attention to her infant. b. Show the mother how the infant initiates interaction and attends to her. c. Demonstrate for the mother different positions for holding her infant while feeding. d. Arrange for the mother to watch a video on parent-infant interaction.

Answer: Show the mother how the infant initiates interaction and attends to her. Rationale: Pointing out the responsiveness of the infant is a positive strategy for facilitating parent-infant attachment. Telling the mother that she must pay attention to her infant may be perceived as derogatory and is not appropriate. Educating the young mother in infant care is important but pointing out the responsiveness of her baby is a better tool for facilitating mother-infant attachment. Videos are an educational tool that can demonstrate parent-infant attachment but encouraging the mother to recognize the infant's responsiveness is more appropriate.

In what form do families tend to be the most socially vulnerable? a. Married-blended family b. Extended family c. Nuclear family d. Single-parent family

Answer: Single-parent family Rationale: The single-parent family tends to be economically and socially vulnerable, creating an unstable and deprived environment for the growth potential of children. The married-blended family, the extended family, and the nuclear family are not the most socially vulnerable.

What is the correct term for the cheeselike, white substance that fuses with the epidermis and serves as a protective coating? a. Vernix caseosa b. Surfactant c. Caput succedaneum d. Acrocyanosis

Answer: vernix caseosa Rationale: The protection provided by vernix caseosa is needed because the infant's skin is so thin. Surfactant is a protein that lines the alveoli of the infant's lungs. Caput succedaneum is the swelling of the tissue over the presenting part of the fetal head. Acrocyanosis is cyanosis of the hands and feet, resulting in a blue coloring.

A 38-year-old Hispanic woman vaginally delivered a 9-pound, 6-ounce baby girl after being in labor for 43 hours. The baby died 3 days later from sepsis. On what grounds could the woman have a legitimate legal case for negligence? a. Inexperienced maternity nurse was assigned to care for the client. b. Client was past her due date by 3 days. c. Standard of care was not met. d. Client refused electronic fetal monitoring.

Answer: Standard of care was not met. Rationale: Not meeting the standard of care is a legitimate factor for a case of negligence. An inexperienced maternity nurse would need to display competency before being assigned to care for clients on his or her own. This client may have been past her due date; however, a term pregnancy often goes beyond 40 weeks of gestation. Although fetal monitoring is the standard of care, the client has the right to refuse treatment. This refusal is not a case for negligence, but informed consent should be properly obtained, and the client should have signed an against medical advice form when refusing any treatment that is within the standard of care.

A nurse is discussing the storage of breast milk with a mother whose infant is preterm and in the special care nursery. Which storage practice indicates that the mother requires additional teaching? a. Keeping breast milk in the deep freezer for up to 6 months. b. Freezing breast milk for up to 3 months." c. Allowing breast milk to sit at room temperature for up to 4 hours. d. Storing breast milk in the refrigerator for up to 10 days."

Answer: Storing breast milk in the refrigerator for up to 10 days." Rationale: Breast milk for the hospitalized infant can be stored in the refrigerator for only 8 days, not for 3 months. Breast milk can be stored in the freezer for 3 months, in a deep freezer for 6 months, or at room temperature for 4 hours. Human milk for the healthy or preterm hospitalized infant can be kept in the refrigerator for up to 8 days or in the freezer for up to 3 months, but only for 4 hours or less at room temperature.

If the newborn has excess secretions, the mouth and nasal passages can be easily cleared with a bulb syringe. How should the nurse instruct the parents on the use of this instrument? a. Avoid suctioning the nares. b. Insert the compressed bulb into the center of the mouth. c. Suction the mouth first. d. Remove the bulb syringe from the crib when finished.

Answer: Suction the mouth first. Rationale: The mouth should always be suctioned first to prevent the infant from inhaling pharyngeal secretions by gasping as the nares are suctioned. After compressing the bulb, the syringe should be inserted into one side of the mouth. If it is inserted into the center of the mouth, then the gag reflex is likely to be initiated. When the infant's cry no longer sounds as though it is through mucus or a bubble, suctioning can be stopped. The nasal passages should be suctioned one nostril at a time. The bulb syringe should remain in the crib so that it is easily accessible if needed again.

A pregnant woman is at 38 weeks of gestation. She wants to know whether there are any signs that "labor is getting close to starting." Which finding is an indication that labor may begin soon? a. Weight gain of 1.5 to 2 kg (3 to 4 lb) b. Increase in fundal height c. Urinary retention d. Surge of energy

Answer: Surge of energy Rationale: Women speak of having a burst of energy before labor. The woman may lose 0.5 to 1.5 kg, as a result of water loss caused by electrolyte shifts that, in turn, are caused by changes in the estrogen and progesterone levels. When the fetus descends into the true pelvis (called lightening), the fundal height may decrease. Urinary frequency may return before labor.

A client is 5 months pregnant. On a routine ultrasound scan, the physician discovers that the fetus has a diaphragmatic hernia. The woman becomes distraught and asks the nurse what she should do. Which response would be most suitable? a. Talk to the client and refer her to a genetic counselor. b. Suggest that the client travel to a fetal treatment center for intrauterine surgery. c. Tell her that everything is going to be fine. d. Sit with the client, and calmly suggest that she consider terminating this pregnancy.

Answer: Talk to the client and refer her to a genetic counselor Rationale: Before the client makes any decisions, she should discuss this newly discovered information with a genetic counselor. Genetic counselors can help with the diagnosis and management of families affected by genetic conditions. The discussion of potential surgery should be pursuant to genetic counseling. Telling the woman that everything is going to be fine may give her false hope and is not accurate. All options should be discussed with the genetic counselor. Furthermore, the guiding principle for genetic counseling is nondirection, which respects the right of the individual or family who are being counseled to make autonomous decisions.

A woman gave birth to a 7-pound, 6-ounce infant girl 1 hour ago. The birth was vaginal and the estimated blood loss (EBL) was 1500 ml. When evaluating the woman's vital signs, which finding would be of greatest concern to the nurse? a. Temperature 37.9° C, heart rate 120 beats per minute (bpm), respirations 20 breaths per minute, and blood pressure 90/50 mm Hg b. Temperature 37.4° C, heart rate 88 bpm, respirations 36 breaths per minute, and blood pressure 126/68 mm Hg c. Temperature 38° C, heart rate 80 bpm, respirations 16 breaths per minute, and blood pressure 110/80 mm Hg d. Temperature 36.8° C, heart rate 60 bpm, respirations 18 breaths per minute, and blood pressure 140/90 mm Hg

Answer: Temperature 37.9° C, heart rate 120 beats per minute (bpm), respirations 20 breaths per minute, and blood pressure 90/50 mm Hg Rationale: An EBL of 1500 ml with tachycardia and hypotension suggests hypovolemia caused by excessive blood loss. Temperature 37.4° C, heart rate 88 bpm, respirations 36 breaths per minute, and blood pressure 126/68 mm Hg are normal vital signs except for an increased respiratory rate, which may be secondary to pain from the birth. Temperature 38° C, heart rate 80 bpm, respirations 16 breaths per minute, and blood pressure 110/80 mm Hg are normal vital signs except for the temperature, which may increase to 38° C during the first 24 hours as a result of the dehydrating effects of labor. Temperature 36.8° C, heart rate 60 bpm, respirations 18 breaths per minute, and blood pressure 140/90 mm Hg are normal vital signs, although the blood pressure is slightly elevated, which may be attributable to the use of oxytocic medications.

Which nursing assessment indicates that a woman who is in second-stage labor is almost ready to give birth? a. Fetal head is felt at 0 station during vaginal examination. b. Bloody mucous discharge increases. c. Vulva bulges and encircles the fetal head. d. Membranes rupture during a contraction.

Answer: Vulva bulges and encircles the fetal head. Rationale: A bulging vulva that encircles the fetal head describes crowning, which occurs shortly before birth. Birth of the head occurs when the station is +4. A 0 station indicates engagement. Bloody show occurs throughout the labor process and is not an indication of an imminent birth. ROM can occur at any time during the labor process and does not indicate an imminent birth.

Which explanation concerning postpartum ovary function is most accurate? a. Almost 75% of women who do not breastfeed resume menstruating within 1 month after birth. b. Ovulation occurs slightly earlier for breastfeeding women. c. Because of menstruation and ovulation schedules, contraception considerations can be postponed until after the puerperium. d. The first menstrual flow after childbirth usually is heavier than normal.

Answer: The first menstrual flow after childbirth usually is heavier than normal. Rationale: The first flow is heavier, but within three or four cycles, the flow is back to normal. Ovulation can occur within the first month, but for 70% of nonlactating women, it returns in approximately 3 months. Women who are breastfeeding take longer to resume ovulation. Because many women ovulate before their first postpartum menstrual period, contraceptive options need to be discussed early in the puerperium.

How should the nurse interpret an Apgar score of 10 at 1 minute after birth? a. The infant is having no difficulty adjusting to extrauterine life and needs no further testing. b. The infant is in severe distress and needs resuscitation. c. The nurse predicts a future free of neurologic problems. d. The infant is having no difficulty adjusting to extrauterine life but should be assessed again at 5 minutes after birth.

Answer: The infant is having no difficulty adjusting to extrauterine life but should be assessed again at 5 minutes after birth. Rationale: An initial Apgar score of 10 is a good sign of healthy adaptation; however, the test must be repeated at the 5-minute mark.

In follow-up appointments or visits with parents and their new baby, it is useful if the nurse can identify infant behaviors that can either facilitate or inhibit attachment. What is an inhibiting behavior? a. The infant cries only when hungry or wet. b. The infant's activity is somewhat predictable. c. The infant clings to the parents. d. The infant seeks attention from any adult in the room.\

Answer: The infant seeks attention from any adult in the room Rationale: Parents want to be the focus of the infant's existence, just as the infant is the focus of their existence. Facilitating and inhibiting behaviors build or discourage bonding (attitudes); they do not reflect any value judgments on what might be healthy or unhealthy. The infant who shows no preference for his or her parents over other adults is exhibiting an inhibiting behavior. An infant who cries only when hungry or wet is exhibiting a facilitating behavior. An infant who has a predictable attention span is exhibiting a facilitating behavior. The infant who clings to his or her parents, enjoys being cuddled and held, and is easily consoled is displaying facilitating behaviors.

A primiparous woman is delighted with her newborn son and wants to begin breastfeeding as soon as possible. How should the client be instructed to position the infant to facilitate correct latch-on? a. The infant should be positioned with his or her arms folded together over the chest. b. The infant should be curled up in a fetal position. c. The woman should cup the infant's head in her hand. d. The infant's head and body should be in alignment with the mother.

Answer: The infant's head and body should be in alignment with the mother. Rationale: The infant's head and body should be in correct alignment with the mother and the breast during latch-on and feeding. The infant should be facing the mother with his arms hugging the breast. The baby's body should be held in correct alignment hips in a straight line) during feedings. The mother should support the baby's neck and shoulders with her hand and not push on the occiput.

The nurse caring for a newborn checks the record to note clinical findings that occurred last shift. Which finding related to the renal system would be of increased significance and require further action? a. The newborn has not voided in 24 hours. b. The breastfed infant voided more often than a formula feed infant. c. Brick dust was noted on several diapers. d. Weight loss from fluid loss and other normal factors has yet to be regained.

Answer: The newborn has not voided in 24 hours. Rationale: A newborn who has not voided in 24 hours may have any of a number of problems, some of which deserve the attention of the pediatrician. Formula-fed infants tend to void more frequently in the first 3 days; breastfed infants will void less during this time because the mother's breast milk has not yet come in. Brick dust may be uric acid crystals; blood spotting could be attributable to the withdrawal of maternal hormones (pseudomenstruation) or a circumcision. The physician must be notified only if the cause of bleeding is not apparent. Weight loss from fluid loss might take 14 days to regain.

The nurse is completing a physical examination of the newborn 24 hours after birth. Which component of the evaluation is correct? a. The parents are excused to reduce their normal anxiety. b. The nurse can gauge the neonate's maturity level by assessing his or her general appearance. c. Once often neglected, blood pressure is now routinely checked. d. When the nurse listens to the neonate's heart, the S1 and S2 sounds can be heard; the S1 sound is somewhat higher in pitch and sharper than the S2 sound.

Answer: The nurse can gauge the neonate's maturity level by assessing his or her general appearance. Rationale: The nurse is looking at skin color, alertness, cry, head size, and other features. The parents' presence actively involves them in child care and gives the nurse the chance to observe their interactions. Blood pressure is not usually taken unless cardiac problems are suspected. The S2 sound is higher and sharper than the S1 sound.

When assessing a multiparous woman who has just given birth to an 8-pound boy, the nurse notes that the woman's fundus is firm and has become globular in shape. A gush of dark red blood comes from her vagina. What is the nurse's assessment of the situation? a. The placenta has separated. b. A cervical tear occurred during the birth. c. The woman is beginning to hemorrhage. d. Clots have formed in the upper uterine segment.

Answer: The placenta has separated Rationale: Placental separation is indicated by a firmly contracting uterus, a change in the uterus from a discoid to a globular ovoid shape, a sudden gush of dark red blood from the introitus, an apparent lengthening of the umbilical cord, and a finding of vaginal fullness. Cervical tears that do not extend to the vagina result in minimal blood loss. Signs of hemorrhage are a boggy uterus, bright red vaginal bleeding, alterations in vital signs, pallor, lightheadedness, restlessness, decreased urinary output, and alteration in the level of consciousness. If clots have formed in the upper uterine segment, then the nurse would expect to find the uterus boggy and displaced to the side.

When assessing a multiparous woman who has just given birth to an 8-pound boy, the nurse notes that the woman's fundus is firm and has become globular in shape. A gush of dark red blood comes from her vagina. What is the nurse's assessment of the situation? a. The placenta has separated. b. A cervical tear occurred during the birth. c. The woman is beginning to hemorrhage. d. Clots have formed in the upper uterine segment.

Answer: The placenta has separated. Rationale: Placental separation is indicated by a firmly contracting uterus, a change in the uterus from a discoid to a globular ovoid shape, a sudden gush of dark red blood from the introitus, an apparent lengthening of the umbilical cord, and a finding of vaginal fullness. Cervical tears that do not extend to the vagina result in minimal blood loss. Signs of hemorrhage are a boggy uterus, bright red vaginal bleeding, alterations in vital signs, pallor, lightheadedness, restlessness, decreased urinary output, and alteration in the level of consciousness. If clots have formed in the upper uterine segment, then the nurse would expect to find the uterus boggy and displaced to the side.

Which information related to the newborn's developing cardiovascular system should the nurse fully comprehend? a. The heart rate of a crying infant may rise to 120 beats per minute. b. Heart murmurs heard after the first few hours are a cause for concern. c. The point of maximal impulse (PMI) is often visible on the chest wall. d. Persistent bradycardia may indicate respiratory distress syndrome (RDS).

Answer: The point of maximal impulse (PMI) is often visible on the chest wall. Rationale: The newborn's thin chest wall often allows the PMI to be observed. The normal heart rate for infants who are not sleeping is 120 to 160 beats per minute. However, a crying infant could temporarily have a heart rate of 180 beats per minute. Heart murmurs during the first few days of life have no pathologic significance; however, an irregular heart rate beyond the first few hours should be further evaluated. Persistent tachycardia may indicate RDS; bradycardia may be a sign of congenital heart blockage.

Which statement most accurately describes dominant genetic disorders? a. With a dominant disorder, the likelihood of the second child also having the condition is 100%. b. An autosomal recessive disease carries a one-in-eight risk of the second child also having the disorder. c. Disorders involving maternal ingestion of drugs carry a one-in-four chance of being repeated in the second child. d. The risk factor remains the same no matter how many affected children are already in the family.

Answer: The risk factor remains the same no matter how many affected children are already in the family. Rationale: Each pregnancy is an independent event. The risk factor (e.g., one-in-two, one-in-four) remains the same for each child, no matter how many children are born to the family. In a dominant disorder, the likelihood of recurrence in subsequent children is 50% (i.e., one-in-two). An autosomal recessive disease carries a one-in-four chance of recurrence. In disorders involving maternal ingestion of drugs, subsequent children would be at risk only if the mother continued to take the drugs; the rate of risk would be difficult to calculate.

The nurse should be cognizant of which important statement regarding care of the umbilical cord? a. The stump can become easily infected. b. If bleeding occurs from the vessels of the cord, then the nurse should immediately call for assistance. c. The cord clamp is removed at cord separation. d. The average cord separation time is 5 to 7 days.

Answer: The stump can become easily infected. Rationale: The cord stump is an excellent medium for bacterial growth. The nurse should first check the clamp (or tie) and apply a second one. If bleeding occurs and does not stop, then the nurse should call for assistance. The cord clamp is removed after 24 hours when it is dry. The average cord separation time is 10 to 14 days.

What is a distinct advantage of external electronic fetal monitoring (EFM)? a. The ultrasound transducer can accurately measure short-term variability and beat-to-beat changes in the fetal heart rate (FHR). b. The toco transducer can measure and record the frequency, regularity, intensity, and approximate duration of uterine contractions. c. The toco transducer is especially valuable for measuring uterine activity during the first stage of labor. d. Once correctly applied by the nurse, the transducer need not be repositioned even when the woman changes positions.

Answer: The toco transducer is especially valuable for measuring uterine activity during the first stage of labor. Rationale: The toco transducer is valuable for measuring uterine activity during the first stage of labor and is especially true when the membranes are intact. Short-term variability and beat-to-beat changes cannot be measured with this technology. The toco transducer cannot measure and record the intensity of uterine contractions. The transducer must be repositioned when the woman or the fetus changes position.

Nurses are often the first health care professional with whom a woman comes into contact after being sexually assaulted. Which statement best describes the initial care of a rape victim? a. All legal evidence is preserved during the physical examination. b. The victim appreciates the legal information; however, decides not to pursue legal proceedings. c. The victim states that she is going to advocate against sexual violence. d. The victim leaves the health care facility without feeling re-victimized.

Answer: The victim leaves the health care facility without feeling re-victimized. Rationale: Nurses can assist clients through an examination that is as nontraumatic as possible with kindness, skill, and empathy. The initial care of the victim affects her recovery and decision to receive follow-up care. Preservation of all legal evidence is very important; however, this may not be the best measure in terms of evaluating the care of a rape victim. Offering legal information is not the best measure of evaluating the care that this victim received. The victim may well decide not to pursue legal proceedings. Advocating against sexual violence may be extremely therapeutic for the client after her initial recovery but not a measure of evaluating her care.

A woman who is 6 months pregnant has sought medical attention, saying she fell down the stairs. What scenario would cause an emergency department nurse to suspect that the woman has been a victim of intimate partner violence (IPV)? a. The woman and her partner are having an argument that is loud and hostile. b. The woman has injuries on various parts of her body that are in different stages of healing. c. Examination reveals a fractured arm and fresh bruises. d. She avoids making eye contact and is hesitant to answer questions.

Answer: The woman has injuries on various parts of her body that are in different stages of healing. Rationale: The client may have multiple injuries in various stages of healing that indicates a pattern of violence. An argument is not always an indication of battering. A fractured arm and fresh bruises could be caused by the reported fall and do not necessarily indicate IPV. It may be normal for the woman to be reticent and have a dull affect.

A primiparous woman is to be discharged from the hospital the following day with her infant girl. Which behavior indicates a need for further intervention by the nurse before the woman can be discharged? a. The woman is weepy and asks to postpone learning about infant care. b. The woman continues to hold and cuddle her infant after she has fed her. c. The woman asks for several magazines to read while her infant sleeps. d. The woman changes her infant's diaper and then shows the nurse the contents of the diaper.

Answer: The woman is weepy and asks to postpone learning about infant care. Rationale: The client should be excited, happy, and interested or involved in infant care. A woman who is sad, tearful, or disinterested in caring for her infant may be exhibiting signs of depression or postpartum blues and may require further intervention. Holding and cuddling her infant after feeding is an appropriate parent-infant interaction. Taking time for herself while the infant is sleeping is an appropriate maternal action. Showing the nurse the contents of the diaper is appropriate because the mother is seeking approval from the nurse and notifying the nurse of the infant's elimination patterns.

A new father wants to know what medication was put into his infant's eyes and why it is needed. How does the nurse explain the purpose of the erythromycin ophthalmic ointment? a. Erythromycin ophthalmic ointment destroys an infectious exudate caused by Staphylococcus that could make the infant blind. b. This ophthalmic ointment prevents gonorrheal infection of the infant's eyes, potentially acquired from the birth canal. c. Erythromycin prevents potentially harmful exudate from invading the tear ducts of the infant's eyes, leading to dry eyes. d. This ointment prevents the infant's eyelids from sticking together and helps the infant see.

Answer: This ophthalmic ointment prevents gonorrheal infection of the infant's eyes, potentially acquired from the birth canal. Rationale: The nurse should explain that prophylactic erythromycin ophthalmic ointment is instilled in the eyes of all neonates to prevent gonorrheal infection that potentially could have been acquired from the birth canal. This prophylactic ophthalmic ointment is not instilled to prevent dry eyes and has no bearing on vision other than to protect against infection that may lead to vision problems.

A hospital has several different perineal pads available for use. A nurse is observed soaking several of them and writing down what is observed. What goal is the nurse attempting to achieve by performing this practice? a. To improve the accuracy of blood loss estimation, which usually is a subjective assessment b. To determine which pad is best c. To demonstrate that other nurses usually underestimate blood loss d. To reveal which brand of pad is more absorbent ANS: A

Answer: To improve the accuracy of blood loss estimation, which usually is a subjective assessment Rationale : Saturation of perineal pads is a critical indicator of excessive blood loss; anything done to help in the assessment is valuable. The nurse is noting the saturation volumes and soaking appearances. Instead of determining which pad is best, the nurse is more likely noting saturation volumes and soaking appearances to improve the accuracy of estimated blood loss. Nurses usually overestimate blood loss.

The nurse is circulating during a cesarean birth of a preterm infant. The obstetrician requests that cord clamping be delayed. What is the rationale for this directive? a. To reduce the risk for jaundice b. To reduce the risk of intraventricular hemorrhage c. To decrease total blood volume d. To improve the ability to fight infection

Answer: To reduce the risk of intraventricular hemorrhage Rationale: Delayed cord clamping provides the greatest benefits to the preterm infant. These benefits include a significant reduction in intraventricular hemorrhage, a reduced need for a blood transfusion, and improved blood cell volume. The risk of jaundice can increase, requiring phototherapy. Although no difference in the newborn's infection fighting ability occurs, iron status is improved, which can provide benefits for 6 months.

The nurse should be aware of which information related to a woman's intake and output during labor? a. Traditionally, restricting the laboring woman to clear liquids and ice chips is being challenged because regional anesthesia is used more often than general anesthesia. b. Intravenous (IV) fluids are usually necessary to ensure that the laboring woman stays hydrated. c. Routine use of an enema empties the rectum and is very helpful for producing a clean, clear delivery. d. When a nulliparous woman experiences the urge to defecate, it often means birth will quickly follow.

Answer: Traditionally, restricting the laboring woman to clear liquids and ice chips is being challenged because regional anesthesia is used more often than general anesthesia. Rationale: Women are awake with regional anesthesia and are able to protect their own airway, which reduces the worry over aspiration. Routine IV fluids during labor are unlikely to be beneficial and may be harmful. The routine use of an enema is, at best, ineffective and may be harmful. Having the urge to defecate followed by the birth of her fetus is true for a multiparous woman but not for a nulliparous woman.

When the services of an interpreter are needed, which is the most important factor for the nurse to consider? a. Using a family member who is fluent in both languages b. Using an interpreter who is certified, and documenting the person's name in the nursing notes c. Directing questions only to the interpreter d. Using an interpreter only in an emergency

Answer: Using an interpreter who is certified, and documenting the person's name in the nursing notes Rationale: Using a certified interpreter ensures that the standards of care are met and that the information exchanged is reliable and unaltered. The name of the interpreter should be documented for legal purposes. Asking a family member to interpret may not be appropriate, although many health care personnel must adopt this approach in an emergency. Furthermore, most states require that certified interpreters be used when possible. When using an interpreter, the nurse should direct questions to the client. The interpreter is simply a means by which the nurse communicates with the client. Every attempt should be made to contact an interpreter whenever one is needed. During an emergency, health care workers often rely on information interpreted by family members. This information may be private and should be protected under the rules established by the Health Insurance Portability and Accountability Act (HIPAA). Furthermore, family members may skew information or may not be able to interpret the exact information the nurse is trying to obtain.

A 25-year-old gravida 3, para 2 client gave birth to a 9-pound, 7-ounce boy, 4 hours ago after augmentation of labor with oxytocin. She presses her call light, and asks for her nurse right away, stating "I'm bleeding a lot." What is the most likely cause of postpartum hemorrhaging in this client? a. Retained placental fragments b. Unrepaired vaginal lacerations c. Uterine atony d. Puerperal infection

Answer: Uterine atony Rationale: This woman gave birth to a macrosomic infant after oxytocin augmentation. Combined with these risk factors, uterine atony is the most likely cause of bleeding 4 hours after delivery. Although retained placental fragments may cause postpartum hemorrhaging, it is typically detected within the first hour after delivery of the placenta and is not the most likely cause of the hemorrhaging in this woman. Although unrepaired vaginal lacerations may also cause bleeding, it typically occurs in the period immediately after birth. Puerperal infection can cause subinvolution and subsequent bleeding that is, however, typically detected 24 hours postpartum.

The nurse providing care for a high-risk laboring woman is alert for late fetal heart rate (FHR) decelerations. Which clinical finding might be the cause for these late decelerations? a. Altered cerebral blood flow b. Umbilical cord compression c. Uteroplacental insufficiency d. Meconium fluid

Answer: Uteroplacental insufficiency Rationale: Uteroplacental insufficiency results in late FHR decelerations. Altered fetal cerebral blood flow results in early FHR decelerations. Umbilical cord compression results in variable FHR decelerations. Meconium-stained fluid may or may not produce changes in the FHR, depending on the gestational age of the fetus and whether other causative factors associated with fetal distress are present.

The nurse who provides care to clients in labor must have a thorough understanding of the physiologic processes of maternal hypotension. Which outcome might occur if the interventions for maternal hypotension are inadequate? a. Early fetal heart rate (FHR) decelerations b. Fetal arrhythmias c. Uteroplacental insufficiency d. Spontaneous rupture of membranes

Answer: Uteroplacental insufficiency Rationale: Low maternal blood pressure reduces placental blood flow during uterine contractions, resulting in fetal hypoxemia. Maternal hypotension does not result in early FHR decelerations nor is it associated with fetal arrhythmias. Spontaneous rupture of membranes is not a result of maternal hypotension.

Which female reproductive organ is responsible for cyclic menstruation? a. Uterus b. Ovary c. Vaginal vestibule d. Urethra

Answer: Uterus Rationale: The uterus is responsible for cyclic menstruation while also housing and nourishing the fertilized ovum and the fetus. The ovaries are responsible for ovulation and the production of estrogen. The vaginal vestibule is an external organ that has openings to the urethra and vagina. The urethra is not a reproductive organ, although it is found in the area.

Which description of postpartum restoration or healing times is accurate? a. The cervix shortens, becomes firm, and returns to form within a month postpartum. b. Vaginal rugae reappear by 3 weeks postpartum. c. Most episiotomies heal within a week. d. Hemorrhoids usually decrease in size within 2 weeks of childbirth.

Answer: Vagial rugae reappear by 3 weeks postpartum Rationale: Vaginal rugae reappear by 3 weeks postpartum; however, they are never as prominent as in nulliparous women. The cervix regains its form within days; the cervical os may take longer. Most episiotomies take 2 to 3 weeks to heal. Hemorrhoids can take 6 weeks to decrease in size.

Individual irregularities in the ovarian (menstrual) cycle are most often caused by what? a. Variations in the follicular (preovulatory) phase b. Intact hypothalamic-pituitary feedback mechanism c. Functioning corpus luteum d. Prolonged ischemic phase

Answer: Variations in the follicular (preovulatory) phase Rationale: Almost all variations in the length of the ovarian cycle are the result of variations in the length of the follicular phase. This information discounts the other options as being correct. An intact hypothalamic-pituitary feedback mechanism would be regular, not irregular. The luteal phase begins after ovulation. The corpus luteum is dependent on the ovulatory phase and fertilization. During the ischemic phase, the blood supply to the functional endometrium is blocked, and necrosis develops. The functional layer separates from the basal layer, and menstrual bleeding begins.

Using social media technology, nurses can link with other nurses who may share similar interests, insights about practice, and advocate for clients. Which factor is the most concerning pitfall for nurses using this technology? a. Violation of client privacy and confidentiality b. Institutions and colleagues who may be cast in an unfavorable light c. Unintended negative consequences for using social media d. Lack of institutional policy governing online contact

Answer: Violation of client privacy and confidentiality Rationale: Guidelines for standards of care are published by various professional nursing organizations. Risk management identifies risks and establishes preventive practices, but it does not define the standard of care. Sentinel events are unexpected negative occurrences. They do not establish the standard of care. Failure to rescue is an evaluative process for nursing, but it does not define the standard of care.

Which component of the sensory system is the least mature at birth? a. Vision b. Hearing c. Smell d. Taste

Answer: Vision Rationale: The visual system continues to develop for the first 6 months after childbirth. As soon as the amniotic fluid drains from the ear (in minutes), the infant's hearing is like that of an adult. Newborns have a highly developed sense of smell and can distinguish and react to various tastes.

The nurse is cognizant of which information related to the administration of vitamin K? a. Vitamin K is important in the production of red blood cells. b. Vitamin K is necessary in the production of platelets. c. Vitamin K is not initially synthesized because of a sterile bowel at birth. d. Vitamin K is responsible for the breakdown of bilirubin and the prevention of jaundice.

Answer: Vitamin K is not initially synthesized because of a sterile bowel at birth. Rationale: The bowel is initially sterile in the newborn, and vitamin K cannot be synthesized until food is introduced into the bowel. Vitamin K is necessary to activate blood-clotting factors. The platelet count in term newborns is near adult levels. Vitamin K is necessary to activate prothrombin and other blood-clotting factors.

Which nursing assessment indicates that a woman who is in second-stage labor is almost ready to give birth? a. Fetal head is felt at 0 station during the vaginal examination. b. Bloody mucous discharge increases. c. Vulva bulges and encircles the fetal head. d. Membranes rupture during a contraction.

Answer: Vulva bulges and encircles the fetal head. During the active pushing (descent) phase, the woman has strong urges to bear down as the presenting part of the fetus descends and presses on the stretch receptors of the pelvic floor. The vulva stretches and begins to bulge, encircling the fetal head. Birth of the head occurs when the station is +4. A 0 station indicates engagement. Bloody show occurs throughout the labor process and is not an indication of an imminent birth. Rupture of membranes can occur at any time during the labor process and does not indicate an imminent birth.

In assisting the breastfeeding mother to position the baby, which information regarding positioning is important for the nurse to keep in mind? a. The cradle position is usually preferred by mothers who had a cesarean birth. b. Women with perineal pain and swelling prefer the modified cradle position. c. Whatever the position used, the infant is "skin to skin" with the mother. d. While supporting the head, the mother should push gently on the occiput.

Answer: Whatever the position used, the infant is "skin to skin" with the mother. Rationale: The infant naturally faces the mother, belly to belly or skin to skin. The football position is usually preferred after a cesarean birth. Women with perineal pain and swelling prefer the side-lying position because they can rest while breastfeeding. The mother should never push on the back of the head. It may cause the baby to bite, hyperextend the neck, or develop an aversion to being brought near the breast.

Under which circumstance would it be unnecessary for the nurse to perform a vaginal examination? a. Admission to the hospital at the start of labor b. When accelerations of the fetal heart rate (FHR) are noted c. On maternal perception of perineal pressure or the urge to bear down d. When membranes rupture

Answer: When accelerations of the fetal heart rate (FHR) are noted Rationale: An accelerated FHR is a positive sign; therefore, a vaginal examination would not be necessary. A vaginal examination should be performed when the woman is admitted to the hospital, when she perceives perineal pressure or the urge to bear down, when her membranes rupture, when a significant change in her uterine activity has occurred, or when variable decelerations of the FHR are noted.

Early this morning, an infant boy was circumcised using the PlastiBell method. Based on the nurse's evaluation, when will the infant be ready for discharge? a. When the bleeding completely stops b. When yellow exudate forms over the glans c. When the PlastiBell plastic rim (bell) falls off d. When the infant voids

Answer: When the infant voids Rationale: The infant should be observed for urination after the circumcision. Bleeding is a common complication after circumcision, and the nurse will check the penis for 12 hours after a circumcision to assess and provide appropriate interventions for the prevention and treatment of bleeding. Yellow exudate covers the glans penis in 24 hours after the circumcision and is part of normal healing; yellow exudate is not an infective process. The PlastiBell plastic rim (bell) remains in place for approximately a week and falls off when healing has taken place

Which clinical finding indicates that the client has reached the second stage of labor? a. Amniotic membranes rupture (ROM). b. Cervix cannot be felt during a vaginal examination. c. Woman experiences a strong urge to bear down. d. Presenting part of the fetus is below the ischial spines.

Answer: Woman experiences a strong urge to bear down Rationale: During the descent phase of the second stage of labor, the woman may experience an increase in the urge to bear down. The ROM has no significance in determining the stage of labor. The second stage of labor begins with full cervical dilation. Many women may have an urge to bear down when the presenting fetal part is below the level of the ischial spines. This urge can occur during the first stage of labor, as early as with 5 cm dilation.

A woman gave birth vaginally to a 9-pound, 12-ounce girl yesterday. Her primary health care provider has written orders for perineal ice packs, use of a sitz bath three times daily, and a stool softener. Which information regarding the client's condition is most closely correlated with these orders? a. Woman is a gravida 2, para 2. b. Woman had a vacuum-assisted birth. c. Woman received epidural anesthesia. d. Woman has an episiotomy.

Answer: Woman has an episiotomy. Rationale: These orders are typical interventions for a woman who has had an episiotomy, lacerations, and hemorrhoids. A multiparous classification is not an indication for these orders. A vacuum-assisted birth may be used in conjunction with an episiotomy, which would indicate these interventions. The use of an epidural anesthesia has no correlation with these orders.

What information should the nurse understand fully regarding rubella and Rh status? a. Breastfeeding mothers cannot be vaccinated with the live attenuated rubella virus. b. Women should be warned that the rubella vaccination is teratogenic and that they must avoid pregnancy for at least 1 month after vaccination. c. Rh immunoglobulin is safely administered intravenously because it cannot harm a nursing infant. d. Rh immunoglobulin boosts the immune system and thereby enhances the effectiveness of vaccinations.

Answer: Women should be warned that the rubella vaccination is teratogenic and that they must avoid pregnancy for at least 1 month after vaccination. Rationale: Women should understand that they must practice contraception for at least 1 month after being vaccinated. Because the live attenuated rubella virus is not communicable in breast milk, breastfeeding mothers can be vaccinated. Rh immunoglobulin is administered intramuscular (IM); it should never be administered to an infant. Rh immunoglobulin suppresses the immune system and therefore might thwart the rubella vaccination.

Which condition or treatment reduces the risk of morbidity in women with the inherited factor V Leiden disorder? a. Anticoagulant therapy b. Pregnancy c. Oral contraceptives d. Hormone replacement therapy

Answer: anticoagulant therapy Rationale: Factor V Leiden is the most common inherited risk factor for primary or recurrent venous thromboembolism. It is an autosomal recessive disorder that increases an individual's risk for blood clots in the legs and pulmonary emboli making anticoagulant therapy a primary management intervention. This risk significantly increases if the woman is pregnant or is taking oral contraceptives or hormone replacement therapy. Prophylactic anticoagulation therapy decreases the risk of comorbidities.

Which key factors play the most powerful role in the behaviors of individuals and families? a. Rituals and customs b. Beliefs and values c. Boundaries and channels d. Socialization processes

Answer: beliefs and values Rationale: Beliefs and values are the most prevalent factors in the decision-making and problem-solving behaviors of individuals and families. This prevalence is particularly true during times of stress and illness. Although culture may play a part in the decision-making process of a family, ultimately, values and beliefs dictate the course of action taken by family members. Boundaries and channels affect the relationship between the family members and the health care team, not the decisions within the family. Socialization processes may help families with interactions within the community, but they are not the criteria used for decision making within the family.

Cancer is now recognized as a genetic disorder that begins with one or more genetic mutations. Which type of cancer is specifically being investigated in this regard? a. Lung cancer b. Liver cancer c. Colorectal cancer d. Oral cancer

Answer: colorectal cancer Rationale: Colorectal cancer usually results from one or more predisposing genes and is the third leading cause of cancer deaths in women. Although tobacco smoke is a known causative factor for lung cancer, an acquired mutation of an oncogene may also be present. Liver cancer is not being investigated in this regard. Oral cancer may be caused by an inherited mutation of one or more oncogenes.

Which pictorial tool can assist the nurse in assessing the aspects of family life related to health care? a. Genogram b. Ecomap c. Life-cycle model d. Human development wheel

Answer: genogram Rationale: A genogram depicts the relationships of the family members over generations. An ecomap is a graphic portrayal of the social relationships of the woman and her family. The life-cycle model, in no way, illustrates a family genogram; rather, it focuses on the stages that a person reaches throughout life. The human development wheel describes various stages of growth and development rather than the family members' relationships to each other.

A married couple lives in a single-family house with their newborn son and the husband's daughter from a previous marriage. Based on this information, what family form best describes this family? a. Married-blended family b. Extended family c. Nuclear family d. Same-sex family

Answer: married-blended family Rationale: Married-blended families are formed as the result of divorce and remarriage. Unrelated family members join to create a new household. Members of an extended family are kin or family members related by blood, such as grandparents, aunts, and uncles. A nuclear family is a traditional family with male and female partners along with the children resulting from that union. A same-sex family is a family with homosexual partners who cohabit with or without children.

A client's household consists of her husband, his mother, and another child. To which family configuration does this client belong? a. Multigenerational family b. Single-parent family c. Married-blended family d. Nuclear family

Answer: multigenerational family Rationale: A multigenerational family includes three or more generations living together. Both parents and a grandparent are living in this extended family. Single-parent families comprise an unmarried biologic or adoptive parent who may or may not be living with other adults. Married-blended families refer to those who are reconstructed after divorce. A nuclear family comprises male and female partners and their children living together as an independent unit.

The nurses working at a newly established birthing center have begun to compare their performance in providing maternal-newborn care against clinical standards. This comparison process is most commonly known as what? a. Best practices network b. Clinical benchmarking c. Outcomes-oriented practice d. Evidence-based practice

Answer: outcomes-oriented practice Rationale: Outcomes-oriented practice measures the effectiveness of the interventions and quality of care against benchmarks or standards. The term best practice refers to a program or service that has been recognized for its excellence. Clinical benchmarking is a process used to compare one's own performance against the performance of the best in an area of service. The term evidence-based practice refers to the provision of care based on evidence gained through research and clinical trials.

A 32-year-old woman is pregnant for the third time. One child was born with cystic fibrosis, and the other child is healthy. The client and her partner wonder what chance this child has of having cystic fibrosis. This type of testing is most commonly known as what? a. Occurrence risk b. Recurrence risk c. Predictive testing d. Predisposition testing

Answer: recurrence risk Rationale: The couple already has a child with a genetic disease; therefore, this couple will be given a recurrence risk test. If a couple has not yet had a child but is known to be at risk for having a child with a genetic disease, then an occurrence risk test is administered. Predictive testing clarifies the genetic status of an asymptomatic family member. Predisposition testing differs from pre-symptomatic testing in that a positive result does not indicate 100% risk of a condition developing.

Which type of genetic tests in clinical practice are most often offered to clients with a family history of disease? a. Single-gene disorders b. Carrier screening c. Predictive value testing d. Pre-dispositional testing

Answer: single-gene disorders Rationale: Most tests now offered are tests for single-gene disorders in clients with clinical symptoms or clients who have a family history of a genetic disease. Carrier screening is used to identify individuals who have a gene mutation for a genetic condition but do not display symptoms. Predictive value testing is used only to clarify the genetic status of asymptomatic family members. Pre-dispositional testing differs from the other types of genetic screening in that a positive result does not indicate a 100% chance of developing the condition.

What is the minimum level of practice that a reasonably prudent nurse is expected to provide? a. Standard of care b. Risk management c. Sentinel event d. Failure to rescue

Answer: standard of care Rationale: Guidelines for standards of care are published by various professional nursing organizations. Risk management identifies risks and establishes preventive practices, but it does not define the standard of care. Sentinel events are unexpected negative occurrences. They do not establish the standard of care. Failure to rescue is an evaluative process for nursing, but it does not define the standard of care.

hich concerns regarding parenthood are often expressed by visually impaired mothers? (Select all that apply.) a. Infant safety b. Transportation c. Ability to care for the infant d. Visually missing out e. Needing extra time for parenting activities to accommodate the visual limitations

Answers a. Infant safety b. Transportation d. Visually missing out e. Needing extra time for parenting activities to accommodate the visual limitations Rationale: Concerns expressed by visually impaired mothers include infant safety, extra time needed for parenting activities, transportation, handling other people's reactions, providing proper discipline, and missing out visually. Blind people sense a reluctance on the part of others to acknowledge that they have a right to be parents. However, blind parents are fully capable of caring for their infants.

The physiologic changes that occur during the reversal of the processes of pregnancy are distinctive; however, they are normal. To provide care during this recovery period, the nurse must synthesize knowledge regarding anticipated maternal changes and deviations from normal. Match the vital signs finding that the postpartum nurse may encounter with its probable cause. 1. Puerperal sepsis 2. Unusually high epidural or spinal block 3. Dehydrating effects of labor 4. Hypovolemia resulting from hemorrhage 5. Excessive use of oxytocin a. Elevated temperature within the first 24 hours b. Rapid pulse c. Elevated temperature at 36 hours postpartum d. Hypertension e. Hypoventilation

Answers: 1. Puerperal sepsis = C 2. Unusually high epidural or spinal block = E 3. Dehydrating effects of labor = A 4. Hypovolemia resulting from hemorrhage = B 5. Excessive use of oxytocin = D

The healthy infant must accomplish both behavioral and biologic tasks to develop normally. Behavioral characteristics form the basis of the social capabilities of the infant. Newborns pass through a hierarchy of developmental challenges as they adapt to their environment and caregivers. This progression in behavior is the basis for the Brazelton Neonatal Behavioral Assessment (NBAS). Match the cluster of neonatal behaviors with the correct level on the NBAS scale. [MATCHING] a. Habituation b. Orientation c. Range of state d. Autonomic stability e. Regulation of state 1. Signs of stress related to homeostatic adjustment 2. Ability to respond to discrete stimuli while asleep 3. Measure of general arousability 4. How the infant responds when aroused 5. Ability to attend to visual and auditory stimuli while alert

Answers: 1. Signs of stress related to homeostatic adjustment = D 2. Ability to respond to discrete stimuli while asleep = A 3. Measure of general arousability = C 4. How the infant responds when aroused = E 5. Ability to attend to visual and auditory stimuli while alert = B

The Patient Protection and Affordable Care Act (ACA) was signed into law by President Obama in early 2010. The Act provides some immediate benefits, and other provisions will take place over the next several years. The practicing nurse should have a thorough understanding of how these changes will benefit his or her clients. Which outcomes are goals of the ACA? (Select all that apply.) a. Insurance affordability b. Improve public health c. Treatment of illness d. Elimination of Medicare and Medicaid e. Cost containment

Answers: A. Insurance affordability B. Improve public health E. Cost containment Rationale: The ACA goals are to make insurance more affordable, contain costs, and strengthen Medicare and Medicaid. The Act contains provisions that promote the prevention of illness and improve access to public health. The ultimate goal of the Act is to improve the quality of care for all Americans while reducing waste, fraud, and abuse of the current system.

Greater than one third of women in the United States are now obese (body mass index [BMI] of 30 or greater). Less than one quarter of women in Canada exhibit the same BMI. Obesity in the pregnant woman increases both maternal medical risk factors and negative outcomes for the infant. The nurse is about to perform an assessment on a client who is 28 weeks pregnant and has a BMI of 35. What are the most frequently reported complications for which the nurse must be alert while assessing this client? (Select all that apply.) a. Potential miscarriage b. Diabetes c. Fetal death in utero d. Decreased fertility e. Hypertension

Answers: B. Diabetes E. Hypertension Rationale: The two most frequently reported maternal medical risk factors associated with obesity are hypertension associated with pregnancy and diabetes. Decreased fertility, miscarriage, fetal death, and congenital anomalies are also associated with obesity. These clients often experience longer hospital stays and increased use of health services.

What are the two primary functions of the ovary? (Select all that apply.) a. Normal female development b. Ovulation c. Sexual response d. Hormone production e. Sex hormone release

Answers: B. Ovulation D. Hormone production Rationale: The two functions of the ovaries are ovulation and hormone production. The presence of ovaries does not guarantee normal female development. The ovaries produce estrogen, progesterone, and androgen. Ovulation is the release of a mature ovum from the ovary. Sexual response is a feedback mechanism involving the hypothalamus, anterior pituitary gland, and ovaries.

Which societal factors have a strong influence on parental response to their infant? (Select all that apply.) a. An adolescent mother's egocentricity and unmet developmental needs interfere with her ability to parent effectively. b. An adolescent mother is likely to use less verbal instruction, be less responsive, and interact less positively than other mothers. c. Adolescent mothers have a higher documented incidence of child abuse. d. Mothers older than 35 years of age often deal with more stress related to work and career issues, as well as decreasing libido. e. Relationships between adolescent mothers and fathers are more stable than older adults.

Answers: a. An adolescent mother's egocentricity and unmet developmental needs interfere with her ability to parent effectively. b. An adolescent mother is likely to use less verbal instruction, be less responsive, and interact less positively than other mothers. d. Mothers older than 35 years of age often deal with more stress related to work and career issues, as well as decreasing libido. Rationale: Adolescent mothers are more inclined to have a number of parenting difficulties that can benefit from counseling, but a higher incidence of child abuse is not one of them. As adolescent mothers move through the transition to parenthood, they can feel different from their peers, excluded from fun activities, and prematurely forced to enter the adult role. The conflict between their own desires and the infant's demands further contribute to the normal psychosocial stress of childbirth and parenting. Adolescent mothers provide warm and attentive physical care; however, they use less verbal interaction than older parents, and adolescents tend to be less responsive and to interact less positively with their infants than older mothers. Midlife mothers have many competencies; however, they are more likely to have to deal with career and sexual issues than are younger mothers. Relationships between adolescent parents tend to be less stable than among adults.

Which activities are included in the role of a nurse practicing in the field of genetics? (Select all that apply.) a. Assessing the responses of family members to a genetic disorder b. Performing genetic testing, such as amniocentesis c. Constructing a family pedigree of three or more generations d. Advising a pregnant mother whose fetus has a genetic disorder to have an abortion e. Offering parents information about genetics

Answers: a. Assessing the responses of family members to a genetic disorder c. Constructing a family pedigree of three or more generations e. Offering parents information about genetics Rationale: Assessing the responses of family members, constructing a family pedigree, and offering parents information about genetics are activities that a genetics nurse would carry out in caring for a family undergoing genetic counseling. Physicians perform amniocentesis, but the nurse may assist in this procedure. Being aware of their own values and beliefs and refraining from attempting to influence the family are important responsibilities for nurses. The nurse must respect the right of the individual or family to make autonomous decisions.

Which statements concerning the benefits or limitations of breastfeeding are accurate? (Select all that apply.) a. Breast milk changes over time to meet the changing needs as infants grow. b. Breastfeeding increases the risk of childhood obesity. c. Breast milk and breastfeeding may enhance cognitive development. d. Long-term studies have shown that the benefits of breast milk continue after the infant is weaned. e. Benefits to the infant include a reduced incidence of SIDS.

Answers: a. Breast milk changes over time to meet the changing needs as infants grow. c. Breast milk and breastfeeding may enhance cognitive development. d. Long-term studies have shown that the benefits of breast milk continue after the infant is weaned. e. Benefits to the infant include a reduced incidence of SIDS. Rationale: Breastfeeding actually decreases the risk of childhood obesity. Human milk is the perfect food for human infants. Breast milk changes over time to meet the demands of the growing infant. Scientific evidence is clear that human milk provides the best nutrients for infants with continued benefits long after weaning. Fatty acids in breast milk promote brain growth and development and may lead to enhanced cognition. Infants who are breastfed experience a reduced incidence of SIDS.

During life in utero, oxygenation of the fetus occurs through transplacental gas exchange. When birth occurs, four factors combine to stimulate the respiratory center in the medulla. The initiation of respiration then follows. What are these four essential factors? (Select all that apply.) a. Chemical b. Mechanical c. Thermal d. Psychologic e. Sensory

Answers: a. Chemical b. Mechanical c. Thermal e. Sensory Rationale: Chemical factors are essential to initiate breathing. During labor, decreased levels of oxygen and increased levels of carbon dioxide seem to have a cumulative effect that is involved in the initiation of breathing. Clamping of the cord may also contribute to the start of respirations and results in a drop in the level of prostaglandins, which are known to inhibit breathing. Mechanical factors are also necessary to initiate respirations. As the infant passes through the birth canal, the chest is compressed. After the birth, the chest is relaxed, which allows for negative intrathoracic pressure that encourages air to flow into the lungs. The profound change in temperature between intrauterine and extrauterine life stimulates receptors in the skin to communicate with the receptors in the medulla. The stimulation of these receptors also contributes to the initiation of breathing. Sensory factors include handling by the health care provider, drying by the nurse, lights, smells, and sounds. Psychologic factors do not contribute to the initiation of respirations.

Which changes take place in the woman's reproductive system, days or even weeks before the commencement of labor? (Select all that apply.) a. Lightening b. Exhaustion c. Bloody show d. Rupture of membranes e. Decreased fetal movement

Answers: a. Lightening c. Bloody show d. Rupture of membranes Rationale: Signs that precede labor may include lightening, urinary frequency, backache, weight loss, surge of energy, bloody show, and rupture of membranes. Many women experience a burst of energy before labor. A decrease in fetal movement is an ominous sign that does not always correlate with labor.

In assessing the immediate condition of the newborn after birth, a sample of cord blood may be a useful adjunct to the Apgar score. Cord blood is then tested for pH, carbon dioxide, oxygen, and base deficit or excess. Which clinical situation warrants this additional testing? (Select all that apply.) a. Low 5-minute Apgar score b. Intrauterine growth restriction (IUGR) c. Maternal thyroid disease d. Intrapartum fever e. Vacuum extraction

Answers: a. Low 5-minute Apgar score b. Intrauterine growth restriction (IUGR) c. Maternal thyroid disease d. Intrapartum fever Rationale: The American College of Obstetricians and Gynecologists (ACOG) suggests obtaining cord blood values in all of these clinical situations except for vacuum extractions deliveries. Cord blood gases should also be performed for multifetal pregnancies or abnormal FHR tracings. Samples can be drawn from both the umbilical artery and the umbilical vein. Results may indicate that fetal compromise has occurred.

Because of its size and rigidity, the fetal head has a major effect on the birth process. Which bones comprise the structure of the fetal skull? (Select all that apply.) a. Parietal b. Temporal c. Fontanel d. Occipital e. Femoral

Answers: a. Parietal b. Temporal d. Occipital Rationale: The fetal skull has two parietal bones, two temporal bones, an occipital bone, and a frontal bone. The fontanels are membrane-filled spaces. Femoral is a term used for identifying the area of the thigh.

Women who have participated in childbirth education classes often bring a birth plan with them to the hospital. Which items might this plan include? (Select all that apply.) a. Presence of companions b. Clothing to be worn c. Care and handling of the newborn d. Medical interventions e. Date of delivery

Answers: a. Presence of companions b. Clothing to be worn c. Care and handling of the newborn d. Medical interventions Rationale: The presence of companions, clothing to be worn, care and handling of the newborn, medical interventions, and environmental modifications all might be included in the couple's birth plan. Other items include the presence of nonessential medical personnel (students), labor activities such as the tub or ambulation, preferred comfort and relaxation methods, and any cultural or religious requirements. The expected date of delivery would not be part of a birth plan unless the client is scheduled for an elective cesarean birth.

Which methods help alleviate the problems associated with access to health care for the maternity client? (Select all that apply.) a. Provide transportation to prenatal visits. b. Provide child care to enable a pregnant woman to keep prenatal visits. c. Increase the number of providers that will care for Medicaid clients. d. Provide low-cost or no-cost health care insurance. e. Provide job training.

Answers: a. Provide transportation to prenatal visits. b. Provide child care to enable a pregnant woman to keep prenatal visits. c. Increase the number of providers that will care for Medicaid clients. d. Provide low-cost or no-cost health care insurance. Rationale: Lack of transportation to prenatal visits, child care, access to skilled obstetric providers, and affordable health insurance are prohibitive factors associated with the lack of prenatal care. Although job training may result in employment and income, the likelihood of significant changes during the time frame of the pregnancy is remote.

Which physiologic factors are reliable indicators of impending shock from postpartum hemorrhage? (Select all that apply.) a. Respirations b. Skin condition c. Blood pressure d. Level of consciousness e. Urinary output

Answers: a. Respirations b. Skin condition d. Level of consciousness e. Urinary output Rationale: Blood pressure is not a reliable indicator; several more sensitive signs are available. Blood pressure does not drop until 30% to 40% of blood volume is lost. Respirations, pulse, skin condition, urinary output, and level of consciousness are more sensitive means of identifying hypovolemic shock.

The class of drugs known as opioid analgesics (butorphanol, nalbuphine) is not suitable for administration to women with known opioid dependence. The antagonistic activity could precipitate withdrawal symptoms (abstinence syndrome) in both mothers and newborns. Which signs would indicate opioid withdrawal in the mother? (Select all that apply.) a. Yawning, runny nose b. Increase in appetite c. Chills or hot flashes d. Constipation e. Irritability, restlessness

Answers: a. Yawning, runny nose c. Chills or hot flashes e. Irritability, restlessness Rationale: The woman experiencing maternal opioid withdrawal syndrome will exhibit yawning, runny nose, sneezing, anorexia, chills or hot flashes, vomiting, diarrhea, abdominal pain, irritability, restlessness, muscle spasms, weakness, and drowsiness. Assessing both the mother and the newborn and planning the care accordingly are important steps for the nurse to take. Neither an increase in appetite nor constipation are associated with opioid withdrawal.

The breast-feeding mother should be taught to expect which changes to the condition of the breasts? (Select all that apply.) a. Breast tenderness is likely to persist for approximately 1 week after the start of lactation. b. As lactation is established, a mass may form that can be distinguished from cancer by its positional shift from day to day. c. In nonlactating mothers, colostrum is present for the first few days after childbirth. d. If suckling is never begun or is discontinued, then lactation ceases within a few days to a week. e. Little change occurs to the breasts in the first 48 hours.

Answers: b. As lactation is established, a mass may form that can be distinguished from cancer by its positional shift from day to day. c. In nonlactating mothers, colostrum is present for the first few days after childbirth. d. If suckling is never begun or is discontinued, then lactation ceases within a few days to a week. Rationale: Breasts become fuller and heavier as colostrum transitions to milk; this fullness should last 72 to 96 hours. The movable, noncancerous mass is a filled milk sac. Colostrum is present for a few days whether or not the mother breastfeeds. A mother who does not want to breastfeed should also avoid stimulating her nipples. Little change to the breasts occurs in the first 24 hours of childbirth.

One of the most promising clinical applications of the Human Genome Project has been pharmacogenomic testing (the use of genetic information to guide a client's drug therapy). Which conditions are potential candidates for pharmacogenomic application? (Select all that apply.) a. Fragile X syndrome b. Deep vein thrombosis (DVT) c. Breast cancer d. Myocardial infarction e. Hemophilia

Answers: b. Deep vein thrombosis (DVT) c. Breast cancer d. Myocardial infarction Rationale: Associations between genetic variation and drug effect have been observed for a number of commonly used drugs. The conditions for which these are applicable include: DVT, breast cancer, and myocardial infarction. Gene therapy has been unsuccessfully used in hemophilia treatment. Fragile X syndrome is the leading cause of intellectual disability and lacks effective treatment of any kind.

Which fetal heart rate (FHR) decelerations would require the nurse to change the maternal position? (Select all that apply.) a. Early decelerations b. Late decelerations c. Variable decelerations d. Moderate decelerations e. Prolonged decelerations

Answers: b. Late decelerations c. Variable decelerations e. Prolonged decelerations Rationale: Early decelerations (and accelerations) do not generally need any nursing intervention. Late decelerations suggest that the nurse should change the maternal position (lateral). Variable decelerations also require a maternal position change (side to side). Moderate decelerations are not an accepted category. Prolonged decelerations are late or variable decelerations that last for a prolonged period (longer than 2 minutes) and require intervention.

What are some common characteristics of a potential male batterer? (Select all that apply.) a. High level of assertiveness b. Low frustration tolerance c. Substance abuse problems d. High level of empathy e. Personality disorders

Answers: b. Low frustration tolerance c. Substance abuse problems e. Personality disorders Rationale: Substance abuse and personality disorders are often observed in batterers. Typically, the batterer has deficits in assertiveness towards others with exception of the battered female. Batterers usually have a low frustration level (i.e., they easily lose their temper). Batterers characteristically demonstrate an inability to demonstrate empathy (the ability to understand and share the feelings of another).

Women of all ages will receive substantial and immediate benefits from smoking cessation. The process is not easy, and most people have attempted to quit numerous times before achieving success. Which organizations provide self-help and smoking cessation materials? (Select all that apply.) a. Leukemia and Lymphoma Society b. March of Dimes c. American Cancer Society d. American Lung Association e. Easter Seals

Answers: b. March of Dimes c. American Cancer Society d. American Lung Association Rationale: The March of Dimes, the American Lung Association, and the American Cancer Society have self-help materials available. The Leukemia and Lymphoma Society support research for these two types of cancer. Easter Seals is best known for its work with disabled children.

Maternal hypotension is a potential side effect of regional anesthesia and analgesia. What nursing interventions could the nurse use to increase the client's blood pressure? (Select all that apply.) a. Place the woman in a supine position. b. Place the woman in a lateral position. c. Increase IV fluids. d. Administer oxygen. e. Perform a vaginal examination.

Answers: b. Place the woman in a lateral position. c. Increase IV fluids. d. Administer oxygen. Rationale: Nursing interventions for maternal hypotension arising from analgesia or anesthesia include turning the woman to a lateral position, increasing IV fluids, administering oxygen via face mask, elevating the woman's legs, notifying the physician, administering an IV vasopressor, and monitoring the maternal and fetal status at least every 5 minutes until the woman is stable. Placing the client in a supine position causes venous compression, thereby limiting blood flow to and oxygenation of the placenta and fetus. A sterile vaginal examination has no bearing on maternal blood pressure.

According to the National Institute of Child Health and Human Development (NICHD) Three-Tier System of FHR Classification, category III tracings include all FHR tracings not categorized as category I or II. Which characteristics of the fetal heart rate (FHR) belong in category III? (Select all that apply.) a. Baseline rate of 110 to 160 beats per minute b. Tachycardia c. Absent baseline variability not accompanied by recurrent decelerations d. Variable decelerations with other characteristics such as shoulders or overshoots e. Absent baseline variability with recurrent variable decelerations f. Bradycardia

Answers: b. Tachycardia d. Variable decelerations with other characteristics such as shoulders or overshoots e. Absent baseline variability with recurrent variable decelerations f. Bradycardia Rationale: Tachycardia, variable decelerations with other characteristics, absent baseline variability with recurrent variable decelerations, and bradycardia are characteristics that are considered non-reassuring or abnormal and belong in category III. A FHR of 110 to 160 beats per minute is considered normal and belongs in category I. Absent baseline variability not accompanied by recurrent decelerations is a category II characteristic.

Changes in blood volume after childbirth depend on several factors such as blood loss during childbirth and the amount of extravascular water (physiologic edema) mobilized and excreted. What amount of blood loss does the postpartum nurse anticipate? (Select all that apply.) a. 100 ml b. 250 ml or less c. 300 to 500 ml d. 500 to 1000 ml e. 1500 ml or greater

Answers: c. 300 to 500 ml d. 500 to 1000 ml Rationale: The average blood loss for a vaginal birth of a single fetus ranges from 300 to 500 ml (10% of blood volume). The typical blood loss for women who gave birth by cesarean is 500 to 1000 ml (15% to 30% of blood volume). During the first few days after childbirth, the plasma volume further decreases as a result of diuresis. Pregnancy-induced hypervolemia (i.e., an increase in blood volume of at least 35%) allows most women to tolerate considerable blood loss during childbirth.

Fetal well-being in labor can be measured by the response of the fetal heart rate (FHR) to uterine contractions. Match the characteristic of normal uterine activity during labor with the correct description. 1. Commonly 45 seconds or more in the second stage of labor 2. Generally ranging from two to five contractions per 10 minutes of labor 3. Average of 10 mm Hg 4. Peaking at 40 to 70 mm Hg in the first stage of labor 5. Remaining fairly stable throughout the first and second stages a. Frequency b. Duration c. Strength d. Resting tone e. Relaxation time

Answers: 1. Commonly 45 seconds or more in the second stage of labor --> e. Relaxation time 2. Generally ranging from two to five contractions per 10 minutes of labor ---> a. Frequency 3. Average of 10 mm Hg ---> d. resting tone 4. Peaking at 40 to 70 mm Hg in the first stage of labor ---> c. strength 5. Remaining fairly stable throughout the first and second stages ---> b. duration

While completing an assessment of a homeless woman, the nurse should be aware of which of the following ailments this client is at a higher risk to develop? (Select all that apply.) a. Infectious diseases b. Chronic illness c. Anemia d. Hyperthermia e. Substance abuse

Answers: a. Infectious diseases b. chronic illness c. anemia e. substance abuse Rationale: Poor living conditions contribute to higher rates of infectious disease. Many homeless individuals engage in sexual favors, which may expose them to sexually transmitted infections (STIs). Poor nutrition can lead to anemia. Lifestyle factors also contribute to chronic illness. Exposure to cold temperatures and harsh environmental surroundings may lead to hypothermia. Many homeless people turn to alcohol and other substances as coping mechanisms.

Pain should be regularly assessed in all newborns. If the infant is displaying physiologic or behavioral cues that indicate pain, then measures should be taken to manage the pain. Which interventions are examples of nonpharmacologic pain management techniques? (Select all that apply.) a. Swaddling b. Nonnutritive sucking c. Skin-to-skin contact with the mother d. Sucrose e. Acetaminophen

Answers: a. Swaddling b. Nonnutritive sucking c. Skin-to-skin contact with the mother d. Sucrose Rationale: Swaddling, nonnutritive sucking, skin-to-skin contact with the mother, and sucrose are all appropriate nonpharmacologic techniques used to manage pain in neonates. Acetaminophen is a pharmacologic method of treating pain.

To promote wellness and prevent illness throughout the life span, it is important for the nurse to be cognizant of immunization recommendations for women older than 18 years. Match each immunization with the correct schedule. 1. Three injections for girls between the ages 9 to 26 years 2. Primary series of three injections 3. Annually 4. Once and then a booster every 10 year 5. One dose after age 65 years 6. Once if born after 1956 a. Tetanus-diphtheria-pertussis (Tdap) b. Measles, mumps, rubella c. Herpes zoster d. Hepatitis B e. Influenza f. Human papillomavirus (HPV)

Answers: 1. Three injections for girls between the ages 9 to 26 years ----> F. Human papillomavirus 2. Primary series of three injections ----> D. Hepatits B 3. Annually -----> E. Influenza 4. Once and then a booster every 10 year ----> A. Tetanus-diphteria-pertussis (Tdap) 5. One dose after age 65 years ----> C. Herpes zoster 6. Once if born after 1956 ----> B. Measles, mumps, rubella

Which statements indicate that the nurse is practicing appropriate family-centered care techniques? (Select all that apply.) a. The nurse commands the pregnant woman to do as she is told. b. The nurse allows time for the partner to ask questions. c. The nurse allows the mother and father to make choices when possible. d. The nurse informs the family about what is going to happen. e. The nurse tells the client's sister, who is a nurse, that she cannot be in the room during the delivery.

Answers: B. The nurse allows time for the partner to ask questions C. The nurse allows the mother and father to make choices when possible Rationale: Including the partner in the care process and allowing the couple to make choices are important elements of family-centered care. The nurse should never tell the client what to do. Family-centered care involves collaboration between the health care team and the client. Unless an institutional policy limits the number of attendants at a delivery, the client should be allowed to have whomever she wants present (except when the situation is an emergency and guests are asked to leave).

A woman has requested an epidural block for her pain. She is 5 cm dilated and 100% effaced. The baby is in a vertex position and is engaged. The nurse increases the woman's IV fluid for a preprocedural bolus. Before the initiation of the epidural, the woman should be informed regarding the disadvantages of an epidural block. Which concerns should the nurse share with this client? (Select all that apply.) a. Ability to move freely is limited. b. Orthostatic hypotension and dizziness may occur. c. Gastric emptying is not delayed. d. Higher body temperature may occur. e. Blood loss is not excessive.

Answers: a. Ability to move freely is limited. b. Orthostatic hypotension and dizziness may occur. d. Higher body temperature may occur. Rationale: The woman's ability to move freely and to maintain control of her labor is limited, related to the use of numerous medical interventions (IV lines and electronic fetal monitoring [EFM]). Significant disadvantages of an epidural block include the occurrence of orthostatic hypotension, dizziness, sedation, and leg weakness. Women who receive an epidural block have a higher body temperature (38° C or higher), especially when labor lasts longer than 12 hours, and may result in an unnecessary neonatal workup for sepsis. An advantage of an epidural block is that blood loss is not excessive. Other advantages include the following: the woman remains alert and able to participate, good relaxation is achieved, airway reflexes remain intact, and only partial motor paralysis develops.

Which alternative approaches to relaxation have proven successful when working with the client in labor? (Select all that apply.) a. Aromatherapy b. Massage c. Hypnosis d. Cesarean birth e. Biofeedback

Answers: a. Aromatherapy b. Massage c. Hypnosis e. Biofeedback Rationale: Approaches to relaxation can include neuromuscular relaxation, aromatherapy, music, massage, imagery, hypnosis, or touch relaxation. Cesarean birth is a method of delivery, not a method of relaxation.

A nurse is discussing the signs and symptoms of mastitis with a mother who is breastfeeding. Which findings should the nurse include in the discussion? (Select all that apply.) a. Breast tenderness b. Warmth in the breast c. Area of redness on the breast often resembling the shape of a pie wedge d. Small white blister on the tip of the nipple e. Fever and flulike symptoms

Answers: a. Breast tenderness b. Warmth in the breast c. Area of redness on the breast often resembling the shape of a pie wedge e. Fever and flulike symptoms Rationale: Breast tenderness, warmth in the breast, redness on the breast, and fever and flulike symptoms are commonly associated with mastitis and should be included in the nurse's discussion of mastitis. A small white blister on the tip of the nipple generally is not associated with mastitis but is commonly seen in women who have a plugged milk duct.

Which congenital malformations result from multifactorial inheritance? (Select all that apply.) a. Cleft lip b. Congenital heart disease c. Cri du chat syndrome d. Anencephaly e. Pyloric stenosis

Answers: a. Cleft lip b. Congenital heart disease d. Anencephaly e. Pyloric stenosis Rationale : Cleft lip, congenital heart disease, anencephaly, and pyloric stenosis are associated with multifactorial inheritance. Cri du chat syndrome is related to a chromosomal deletion.

While developing an intrapartum care plan for the client in early labor, which psychosocial factors would the nurse recognize upon the client's pain experience? (Select all that apply.) a. Culture b. Anxiety and fear c. Previous experiences with pain d. Intervention of caregivers e. Support systems

Answers: a. Culture b. Anxiety and fear c. Previous experiences with pain e. Support systems Rationale: Culture: A woman's sociocultural roots influence how she perceives, interprets, and responds to pain during childbirth. Some cultures encourage loud and vigorous expressions of pain, whereas others value self-control. The nurse should avoid praising some behaviors (stoicism) while belittling others (noisy expression). Anxiety and fear: Extreme anxiety and fear magnify the sensitivity to pain and impair a woman's ability to tolerate it. Anxiety and fear increase muscle tension in the pelvic area, which counters the expulsive forces of uterine contractions and pushing efforts. Previous experiences with pain: Fear and withdrawal are a natural response to pain during labor. Learning about these normal sensations ahead of time helps a woman suppress her natural reactions of fear regarding the impending birth. If a woman previously had a long and difficult labor, she is likely to be anxious. She may also have learned ways to cope and may use these skills to adapt to the present labor experience. Support systems: An anxious partner is less able to provide help and support to a woman during labor. A woman's family and friends can be an important source of support if they convey realistic and positive information about labor and delivery. Although the intervention of caregivers may be necessary for the well-being of the woman and her fetus, some interventions add discomfort to the natural pain of labor (i.e., fetal monitor straps, IV lines).

Many pregnant teenagers wait until the second or third trimester to seek prenatal care. What should the nurse recognize as reasons for this delay? (Select all that apply.) a. Lack of realization that they are pregnant b. Uncertainty as to where to go for care c. Continuing to deny the pregnancy d. Desire to gain control over their situation e. Wanting to hide the pregnancy as long as possible

Answers: a. Lack of realization that they are pregnant b. Uncertainty as to where to go for care c. Continuing to deny the pregnancy e. Wanting to hide the pregnancy as long as possible Rationale: These reasons are all valid explanations why teens delay seeking prenatal care. An adolescent often has little to no understanding of the increased physiologic needs that a pregnancy places on her body. Once care is sought, it is often sporadic, and many appointments are usually missed. The nurse should formulate a diagnosis that assists the pregnant teen to receive adequate prenatal care. Planning for her pregnancy and impending birth actually provides some sense of control for the teen and increases her feelings of competency. Receiving praise from the nurse when she attends her prenatal appointments will reinforce the teen's positive self-image.

Which statements regarding menstruation (periodic uterine bleeding) are accurate? (Select all that apply.) a. Menstruation occurs every 28 days. b. During menstruation, the entire uterine lining is shed. c. Menstruation begins 7 to 10 days after ovulation. d. Menstruation leads to fertilization. e. Average blood loss during menstruation is 50 ml.

Answers: a. Menstruation occurs every 28 days. b. During menstruation, the entire uterine lining is shed. e. Average blood loss during menstruation is 50 ml. Rationale: Menstruation is the periodic uterine bleeding that is controlled by a feedback system involving three cycles: the endometrial cycle, the hypothalamic-pituitary cycle, and the ovarian cycle. The average length of a menstrual cycle is 28 days; however, variations are normal. During the endometrial cycle, the functional two thirds of the endometrium is shed. The average blood loss is 50 ml with a normal range of 20 to 80 ml. Menstruation occurs 14 days after ovulation. The lack of fertilization leads to menstruation.

Which statements regarding physiologic jaundice are accurate? (Select all that apply.) a. Neonatal jaundice is common; however, kernicterus is rare. b. Appearance of jaundice during the first 24 hours or beyond day 7 indicates a pathologic process. c. Because jaundice may not appear before discharge, parents need instruction on how to assess for jaundice and when to call for medical help. d. Jaundice is caused by reduced levels of serum bilirubin. e. Breastfed babies have a lower incidence of jaundice.

Answers: a. Neonatal jaundice is common; however, kernicterus is rare. b. Appearance of jaundice during the first 24 hours or beyond day 7 indicates a pathologic process. c. Because jaundice may not appear before discharge, parents need instruction on how to assess for jaundice and when to call for medical help. Rationale: Neonatal jaundice occurs in 60% of term newborns and in 80% of preterm infants. The complication called kernicterus is rare. Jaundice in the first 24 hours or that persists past day 7 is cause for medical concern. Parents need to be taught how to evaluate their infant for signs of jaundice. Jaundice is caused by elevated levels of serum bilirubin. Breastfeeding is associated with an increased incidence of jaundice.

At least five factors affect the process of labor and birth. These are easily remembered as the five Ps. Which factors are included in this process? (Select all that apply.) a. Passenger b. Passageway c. Powers d. Pressure e. Psychologic response

Answers: a. Passenger b. Passageway c. Powers e. Psychologic response Rationale: The five Ps are passenger (fetus and placenta), passageway (birth canal), powers (contractions), position of the mother, and psychologic response. Pressure is not one of the five Ps.

If a woman is at risk for thrombus but is not ready to ambulate, which nursing intervention would the nurse include in the plan of care? (Select all that apply.) a. Putting her in antiembolic stockings b. Having her flex, extend, and rotate her feet, ankles, and legs c. Having her sit in a chair for at 30 uninterrupted minutes d. Immediately notifying the healthcare provider when a positive Homans sign occurs e. Applying sequential compression devices as prescribed

Answers: a. Putting her in antiembolic stockings b. Having her flex, extend, and rotate her feet, ankles, and legs d. Immediately notifying the healthcare provider when a positive Homans sign occurs e. Applying sequential compression devices as prescribed Rationale: Sitting immobile in a chair does not help; bed exercise and prophylactic footwear might. TED hose and SCD boots are recommended. The client should be encouraged to ambulate with assistance, not remain in bed. Bed exercises are useful. A positive Homans sign (calf muscle pain or warmth, redness, tenderness) requires the healthcare provider's immediate attention.

Which factors influence cervical dilation? (Select all that apply.) a. Strong uterine contractions b. Force of the presenting fetal part against the cervix c. Size of the woman d. Pressure applied by the amniotic sac e. Scarring of the cervix

Answers: a. Strong uterine contractions b. Force of the presenting fetal part against the cervix d. Pressure applied by the amniotic sac e. Scarring of the cervix Rationale: Dilation of the cervix occurs by the drawing upward of the musculofibrous components of the cervix, which is caused by strong uterine contractions. Pressure exerted by the amniotic fluid while the membranes are intact or by the force applied by the presenting part can also promote cervical dilation. Scarring of the cervix because of a previous infection or surgery may slow cervical dilation. Neither pelvic size nor the physical size of the woman affect cervical dilation.

Which actions are examples of appropriate techniques to wake a sleepy infant for breastfeeding? (Select all that apply.) a. Unwrapping the infant b. Changing the diaper c. Talking to the infant d. Slapping the infant's hands and feet e. Applying a cold towel to the infant's abdomen

Answers: a. Unwrapping the infant b. Changing the diaper c. Talking to the infant Rationale: Unwrapping the infant, changing the diaper, and talking to the infant are appropriate techniques to use when trying to wake a sleepy infant. The parent can rub, never slap, the infant's hands or feet to wake the infant. Applying a cold towel to the infant's abdomen may lead to cold stress in the infant. The parent may want to apply a cool cloth to the infant's face to wake the infant.

The transition to parenting for same-sex couples can present unique challenges. How can the nurse foster adjustment to parenting for these clients? (Select all that apply.) a. Use a supplemental feeding device to simulate breastfeeding. b. Allow the partner to cut the cord. c. Gay fathers should meet their new infant soon after the birth mother has recovered. d. Understand that strong social sanctions remain. e. Provide information regarding support groups.

Answers: a. Use a supplemental feeding device to simulate breastfeeding. b. Allow the partner to cut the cord. d. Understand that strong social sanctions remain. e. Provide information regarding support groups. Rationale: In a lesbian couple, the nonchildbearing partner may have a desire to breastfeed. This can be achieved using a supplemental nursing device. The female partner should be offered the same right as a heterosexual partner including cutting the cord. A gay couple may adopt a baby or use a surrogate. If the latter method is chosen, then they should be present at the birth if at all possible. The nurse can refer these men to available support groups. Same-sex couples continue to face strong social sanction in their efforts to parent.

A parent who has a hearing impairment is presented with several challenges in parenting. Which nursing approaches are appropriate for working with hearing-impaired new parents? (Select all that apply.) a. Using devices that transform sound into light b. Assuming that the client knows sign language c. Speaking quickly and loudly d. Ascertaining whether the client can read lips before teaching e. Writing messages that aid in communication

Answers: a. Using devices that transform sound into light d. Ascertaining whether the client can read lips before teaching e. Writing messages that aid in communication Rationale: Section 504 of the Rehabilitation Act of 1973 requires that hospitals use various communication techniques and resources with the deaf and hard of hearing client. These resources include devices such as door alarms, cry alarms, and amplifiers. Before initiating communication, the nurse needs to be aware of the parents' preferences for communication. Not all hearing-impaired clients know sign language. Do they wear a hearing aid? Do they read lips? Do they wish to have a sign language interpreter? If the parent relies on lip reading, then the nurse should sit close enough to enable the parent to visualize lip movements. The nurse should speak clearly in a regular voice volume, in short, simple sentences. Written messages such as on a black or white erasable board can be useful.

A tiered system of categorizing fetal heart rate (FHR) has been recommended by professional organizations. Nurses, midwives, and physicians who care for women in labor must have a working knowledge of fetal monitoring standards and understand the significance of each category. What is the correct nomenclature for these categories? (Select all that apply.) a. Reassuring b. Category I c. Category II d. Nonreassuring e. Category III

Answers: b. Category I c. Category II e. Category III Rationale: The three-tiered system of FHR tracings include category I, II, and III. Category I is a normal tracing requiring no action. Category II FHR tracings are indeterminate and includes tracings that do not meet category I or III criteria. Category III tracings are abnormal and require immediate intervention.

Which information related to a prolonged deceleration is important for the labor nurse to understand? a. Prolonged decelerations present a continuing pattern of benign decelerations that do not require intervention. b. Prolonged decelerations constitute a baseline change when they last longer than 5 minutes. c. A disruption to the fetal oxygen supply causes prolonged decelerations. d. Prolonged decelerations require the customary fetal monitoring by the nurse.

Answers: A disruption to the fetal oxygen supply causes prolonged decelerations. Rationale: Prolonged decelerations are caused by a disruption in the fetal oxygen supply. They usually begin as a reflex response to hypoxia. If the disruption continues, then the fetal cardiac tissue, itself, will become hypoxic, resulting in direct myocardial depression of the FHR. Prolonged decelerations can be caused by prolonged cord compression, uteroplacental insufficiency, or perhaps sustained head compression. Prolonged decelerations lasting longer than 10 minutes are considered a baseline change that may require intervention. A prolonged deceleration is a visually apparent decrease (may be either gradual or abrupt) in the FHR of at least 15 beats per minute below the baseline and lasting longer than 2 minutes but shorter than 10 minutes. Nurses should immediately notify the physician or nurse-midwife and initiate appropriate treatment of abnormal patterns when they see prolonged decelerations.

At a 2-month well-baby examination, it was discovered that an exclusively breastfed infant had only gained 10 ounces in the past 4 weeks. The mother and the nurse develop a feeding plan for the infant to increase his weight gain. Which change in dietary management will assist the client in meeting this goal? a. Begin solid foods. b. Have a bottle of formula after every feeding. c. Have one extra breastfeeding session every 24 hours. d. Start iron supplements.

C. Have one extra breastfeeding session every 24 hours Rationale: Usually the solution to slow weight gain is to improve the feeding technique which includes adding an extra session every day . Position and the latch-on technique are evaluated, and adjustments are made. Adding a feeding or two within a 24-hour period might help. Solid foods should not be introduced to an infant for at least 4 to 6 months. Bottle feeding may cause nipple confusion and may limit the supply of milk. Iron supplements have no bearing on weight gain.

The baseline fetal heart rate (FHR) is the average rate during a 10-minute segment. Changes in FHR are categorized as periodic or episodic. These patterns include both accelerations and decelerations. The labor nurse is evaluating the client's most recent 10-minute segment on the monitor strip and notes a late deceleration. Which is likely to have caused this change? (Select all that apply.)

C. Placental abruption E. Maternal supine hypotension Rationale: Late decelerations are almost always caused by uteroplacental insufficiency. Insufficiency is caused by uterine tachysystole, maternal hypotension, epidural or spinal anesthesia, IUGR, intraamniotic infection, or placental abruption. Spontaneous fetal movement, vaginal examination, fetal scalp stimulation, fetal reaction to external sounds, uterine contractions, fundal pressure, and abdominal palpation are all likely to cause accelerations of the FHR. Early decelerations are most often the result of fetal head compression and may be caused by uterine contractions, fundal pressure, vaginal examination, and the placement of an internal electrode. A variable deceleration is likely caused by umbilical cord compression, which may happen when the umbilical cord is around the baby's neck, arm, leg, or other body part or when a short cord, a knot in the cord, or a prolapsed cord is present.

A multiparous woman has been in labor for 8 hours. Her membranes have just ruptured. What is the nurse's highest priority in this situation? a. Prepare the woman for imminent birth. b. Notify the woman's primary health care provider. c. Document the characteristics of the fluid. d. Assess the fetal heart rate (FHR) and pattern.

D. Assess the fetal heart rate (FHR) and pattern. Rationale: The umbilical cord may prolapse when the membranes rupture. The FHR and pattern should be closely monitored for several minutes immediately after the rupture of membranes (ROM) to ascertain fetal well-being, and the findings should be documented. The ROM may increase the intensity and frequency of the uterine contractions, but it does not indicate that birth is imminent. The nurse may notify the primary health care provider after ROM occurs and the fetal well-being and response to ROM have been assessed. The nurse's priority is to assess fetal well-being. The nurse should document the characteristics of the amniotic fluid, but the initial response is to assess fetal well-being and the response to ROM.

The nurse is evaluating the electronic feta monitoring (EFM) tracing of the client who is in active labor. Suddenly, the fetal heart rate (FHR) drops from its baseline of 125 down to 80 beats per minute. The mother is repositioned, and the nurse provides oxygen, increased IV fluids, and performs a vaginal examination. The cervix has not changed. Five minutes have passed, and the FHR remains in the 80s. What additional nursing measures should the nurse take next? a. Call for help. b. Insert a Foley catheter. c. Start administering Pitocin. d. Immediately notify the care provider.

D. Immediately notify the care provider. Rationale: To relieve an FHR deceleration, the nurse can reposition the mother, increase IV fluids, and provide oxygen. If oxytocin is infusing, then it should be discontinued. If the FHR does not resolve, then the primary care provider should be immediately notified. Inserting a Foley catheter is an inappropriate nursing action. If the FHR were to continue in a non-reassuring pattern, then a cesarean section could be warranted, which would require a Foley catheter. However, the physician must make that determination. The administration of Pitocin may place additional stress on the fetus.

The AAP recommends pasteurized donor milk for preterm infants if the mother's own milk in not available. Which statements regarding donor milk and milk banking are important for the nurse to understand and communicate to her client? (Select all that apply.) a. All milk bank donors are screened for communicable diseases. b. Internet milk sharing is an acceptable source for donor milk. c. Donor milk may be given to transplant clients. d. Donor milk is used in neonatal intensive care units (NICUs) for severely low-birth-weight infants only. e. Donor milk may be used for children with immunoglobulin A (IgA) deficiencies.

a. All milk bank donors are screened for communicable diseases. c. Donor milk may be given to transplant clients. e. Donor milk may be used for children with immunoglobulin A (IgA) deficiencies. Rationale: Because of the antiinfective and growth promotion properties for donor milk, donor milk is highly recommended for preterm and sick infants, as well as for term newborns. Human donor milk has also been used for older children with short gut syndrome, immunodeficiencies, metabolic disorders, or congenital anomalies. Human donor milk has also been used in the adult population—posttransplant clients and for those with colitis, ulcers, or cirrhosis of the liver. Some mothers acquire milk through Internet-based or community-based milk sharing. The U.S. Food and Drug Administration (FDA) has issued a warning regarding this practice. Samples of milk from these sources are higher in contaminants and infectious disease. A milk bank that belongs to the Human Milk Banking Association of North America should always be used for donor milk. All donors are scrupulously screened, and the milk is tested to determine its safety for use.

Which practices are ideal for role modeling when attempting to prevent sudden infant death syndrome (SIDS)? (Select all that apply.) a. Fully supine position for all sleep b. Side-sleeping position as an acceptable alternative c. "Tummy time" for play d. Infant sleep sacks or buntings e. Soft mattress

a. Fully supine position for all sleep c. "Tummy time" for play d. Infant sleep sacks or buntings Rationale: The "back to sleep" position is now recommended as the only position for every sleep period. To prevent positional plagiocephaly (flattening of the head) the infant should spend time on his or her abdomen while awake and for play. Loose sheets and blankets may be dangerous because they could easily cover the baby's head. The parents should be instructed to tuck any bedding securely around the mattress or use sleep sacks or bunting bags instead. The side-sleeping position is no longer an acceptable alternative position, according to the AAP. Infants should always sleep on a firm surface, ideally a firm crib mattress covered by a sheet only. Quilts and sheepskins, among other bedding, should not be placed under the infant.

Emergency conditions during labor that would require immediate nursing intervention can arise with startling speed. Which situations are examples of such an emergency? (Select all that apply.) a. Nonreassuring or abnormal fetal heart rate (FHR) pattern b. Inadequate uterine relaxation c. Vaginal bleeding d. Prolonged second stage e. Prolapse of the cord

a. Nonreassuring or abnormal fetal heart rate (FHR) pattern b. Inadequate uterine relaxation c. Vaginal bleeding e. Prolapse of the cord Rationale: A nonreassuring or abnormal FHR pattern, inadequate uterine relaxation, vaginal bleeding, infection, and cord prolapse all constitute an emergency during labor that requires immediate nursing intervention. A prolonged second stage of labor after the upper limits for duration is reached. This is 3 hours for nulliparous women and 2 hours for multiparous women.

In the United States, the en face position is preferred immediately after birth. Which actions by the nurse can facilitate this process? (Select all that apply.) a. Washing both the infant's face and the mother's face b. Placing the infant on the mother's abdomen or breast with their heads on the same plane c. Dimming the lights d. Delaying the instillation of prophylactic antibiotic ointment in the infant's eyes e. Placing the infant in the grandmother's arms

b. Placing the infant on the mother's abdomen or breast with their heads on the same plane c. Dimming the lights d. Delaying the instillation of prophylactic antibiotic ointment in the Rationale: As newborns become functionally able to sustain eye contact with their parents, they spend time in mutual gazing, often in the en face position, a position in which the faces of the parent and infant are approximately 20 cm apart and on the same plane. Washing the faces of the infant or mother is not necessary at this time and would interrupt the process. Nurses and physicians or midwives can facilitate eye contact immediately after birth by placing the infant on the mother's abdomen or breasts with the mother and the infant's faces on the same plane. Dimming the lights encourages the infant's eyes to stay open. To promote eye contact, the instillation of prophylactic antibiotic ointment into the infant's eyes can be delayed until after the infant and parents have had some time together during the first hour after birth. Having the grandmother hold the infant is important; however, it will not necessarily promote eye contact between the parent and infant.

What are the responsibilities of the nurse who suspects or confirms any type of violence against a woman? (Select all that apply.) a. Report the incident to legal authorities. b. Provide resources for domestic violence shelters. c. Call a client advocate who can assist in the client's decision about what actions to take. d. Accurately and concisely document the incident (or findings) in the client's record. e. Reassure and support the client.

b. Provide resources for domestic violence shelters. c. Call a client advocate who can assist in the client's decision about what actions to take. d. Accurately and concisely document the incident (or findings) in the client's record. e. Reassure and support the client. Rationale: Domestic violence is considered a crime in all states; however, mandatory reporting remains controversial. Nurses must become knowledgeable on the laws that apply in the state in which they practice. Caring for a client who may be a victim of domestic abuse is an ideal opportunity to provide the woman with information for safe houses or support groups for herself and her children. The nurse may assist in reaching out to a client advocate, which often occurs when potential legal action is taken or if the woman is seeking shelter. Documentation must be accurate and timely to be useful to the client later in court if she chooses to press charges. The primary functions for the nurse are to reassure the client and to provide her with emotional support.


Conjuntos de estudio relacionados

Exam 1 ATI Test Questions: NUR3000

View Set

Chapter 4 Family And Patient Centered Care

View Set

I've given up Exam 6 NCLEX Questions

View Set

Lecture 3 Learning Outcomes - BIOL2085 Cell Biology - Professor Maria Torres

View Set

Ch 29: Nonmalignant Hematologic Disorders

View Set

HESI Cardiovascular defect- PEDS

View Set

Abeka 5th grade History, Quiz 12

View Set